Sunteți pe pagina 1din 198

1

Esophagus

Scenario #1 - Pediatric Esophageal Burn


8 year old child drinks strong alkali solution accidentally. Issues: a.) E.R. management - airway management, lines, resuscitation. b.) Role of antibiotics and/or steroids? c.) Endoscopy - when and how far should scope be passed? d.) Barium Swallow - if so, when? e.) How is stricture managed? f.) If stricture persists and esophageal replacement is needed, what is the best conduit and when should it be performed?

Answer:
Background - alkali is more destructive than acids, producing liquifactive necrosis which almost ensures deep penetration. Liquid (as opposed to solid) allows contact to the esophagus and stomach with concomitant damage, which can extend to the adjacent structures. Reflex antral and pyloric spasm cause the alkali to pool in the stomach and reflux into the esophagus. May cause relatively little oropharyngeal burn and still have severe injury to the esophagus and stomach. Initial management Airway - oral intubation or tracheostomy is indicated if there is laryngeal edema or laryngeal destruction from ingestion. Breathing - 02 Circ. - oral ingestion of liquids should be held, as this has little benefit. IV fluids should be instituted to correct hypovolemia. Antibiotics - broad spectrum antibiotics should be started ASAP to diminish risk of aspiration as well as direct bacterial invasion through the damaged wall of the esophagus. ?steroids - NO. Have been advocated to prevent stricture but efficacy has not been established. May also mask sepsis and injury while slowing healing. Contrast Esophagram - best way to evaluate an esophageal perforation and should be used if a perforation is suspected either initially or in follow-up. Initial esophagram should be performed with water soluble gastrograffin. However, dilute barium should be used if perforation is

suspected and not seen with water soluble contrast. Diagnosis of perforation and identifying site of perforation is important to plan operative intervention. Esophagoscopy - should be used ASAP after admission to establish diagnosis and grade the severity of the injury. Should use pediatric flexible endoscope and sedation to minimize the risk of perforation. It is acceptable to advance the scope past the area of the first burn, though conservative answer is to pass the scope to the level of the first burn. Grading: First degree - mucosal hyperemia and edema. Second degree - mucosal ulceration with vesicles and exudates, pseudomembranes present. Third Degree - deep ulceration with charring and eschar formation: severe edema obliterating lumen. TREATMENT: FIRST DEGREE - no specific treatment for 24-48 hrs. May begin liquids and treated as an outpatient. SECOND DEGREE/THIRD DEGREE - must be more closely observed for evidence of gastric or esophageal perforation during the acute phase of injury. Problem - full thickness necrosis requires resection but it is very difficult to tell which have full thickness necrosis. Therefore, those not showing free air but with persistent back pain (mediastinitis), peritonitis, or acidosis (visceral injury) should be explored. Operative intervention - should be done through the abdomen to assess the abdominal organs. Though only the distal esophagus is seen through the abdomen, if an esophageal resection is needed, a transhiatal esophagectomy should be performed. (So prep the neck, chest abdomen to axillary lines.) Rare that gastric resection is needed without esophageal resection, but if this happens, the esophagus should be brought out as an esophagostomy. NONOPERATIVE TREATMENT - as a rule second and third degree burns will cause stricture formation. Dilatation has been the traditional therapy. Important to wait 6-8 weeks for dilation to allow re-epithelialization to take place and decrease risk of perforation. Grading of Strictures: mild - minimal narrowing moderate - narrowing of lumen to half of its diameter. severe - lumen greatly narrowed or pinhole. Technique - through existing gastrostomy, placed at time of admission to the hospital and allowing 6-8 weeks to heal. Goal - 32-38/40 Gfr. dilator for a 2- 3 yr. old child over a series of dilations for 2-3 weeks. Goal is for 46 Fr. or larger for adults. Stricture Perforation during dilatation - should be treated with esophagectomy and replacement. Esophageal Replacement - needed for strictures that are refractory to dilation after 6-12 months or strictures that cannot be adequately dilated should be replaced.

Choice of Replacement - stomach as conduit after transhiatal esophagectomy is best but often scarring or contracture from original injury may necessitate replacement with colon interposition. If this is performed the original esophagus should be removed because of the small risk of esophageal CA with history of alkali ingestion. Esophageal Stricture I II Mild or Moderate Severe II Antegrade Dilation <------------------------------------------------ Retrograde response I I No response Esoph. Replac

Scenario #2 - Achalasia
55 y.o. female presents with dysphagia and occasional regurgitation of food. Issues: a.) Appropriate workup for dysphagia. If scope for biopsy, decompress pouch first with NGT b.) What is diagnosis if birds beak found on UGI? c.) How should UGI be performed? d.) What are the manometric findings consistent with achalasia? Esoph. body, LES, contraction? What is normal LES? What are manometric findings of DES Diffuse Esophageal Spasm)? e.) Appropriate nonsurgical therapy? Appropriate surgical therapy and how is this performed? ANSWER: Background: failure of the distal esophageal sphincter to relax on deglutition. Additional features include partial or complete loss of progressive peristalsis in the body of the esophagus, elevation of the intraluminal esophageal pressure, and hypertension of the LES at rest. Nonrelaxation of the sphincter and loss of peristalsis lead to dilated esophagus. Presentation: Initially complains of dysphagia, primarily of solid foods. As the organ dilates into a reservoir organ, the sensation of dysphagia may pass. In this situation, regurgitation and aspiration of esophageal contents is common.

Diagnosis: suggested by clinical presentation of dysphagia and regurgitation. WORKUP: UGI: Birds Beak - deformity of the esophagus at the level of the esophagogastric junction. Varying degrees of esophageal dilatation may be present. Useful first test to confirm diagnosis, identify anatomy and rule in achalasia so that esophagus can be emptied prior of endoscopy and/or manometry Manometry - Achalasia 1.) absent or incomplete relaxation of the LES. (Normal LES = 15 cm.) 2.) complete absence of peristalsis in the smooth muscle portion of the esophagus - all contractions are simultaneous - no stripping wave to empty the esophagus. (The nonrelaxing sphincter produces a functional distal obstruction, which is worsened by the lack of peristalsis of the esophagus). 3.) other abnormalities - high resting pressure in the LES. - low amplitude of esophageal contraction. (< 30 mmHg) Differentiating from DES - early in the course of the disease, when esophageal contractions may be of normal or even increased amplitude making differentiating the two disorders difficult. ** - DES will have relaxation of the LES, but achalasia will not. Manometry of DES - frequent simultaneous esophageal contractions. For diagnosis of DES, must have minimal frequency of simultaneous contractions of more than 10-20% of all esophageal contractions in response to wet swallows challenges. (Simultaneous contractions will occur in 25% of volunteers) Emphasis is on manometric criteria of DES is the pattern rather than the force of esophageal contractions. TREATMENT: Pneumatic Dilatation - goal is to rapidly distend and actually disrupt the muscle of the LES while the esophageal mucosa remains intact. Balloon is placed fluoroscopically at the GE junction and rapidly inflated to diameter of 3-4 cm. Complication - esophageal perforation rate of 3-4%. Poor candidates for dilation - dilated esophagus - vigorous alchalasia making passing balloon difficult. Results - 75% achieve good result with no more than 2 sessions and never require further therapy. SURGICAL TREATMENT - recommended for those without success after 2 dilations or those in whom dilation cannot be performed because the esophagus is tortuous or dilated. - 90% improvement with surgery vs. 70% with forceful dilation. TECHNIQUE - Preop Prep - esophageal dilation is associated with retention esophagitis and submucosal inflammation which will make the dissection difficult. It is necessary to decompress the esophagus with NG sump preoperatively to allow the retention esophagitis to resolve. Esophagoscopy is important to rule out a tumor at the GE junction which is causing a pseudoachalasia or a tumor in the middle third of the esophagus (hidden by distention).

Left Thoracotomy - through the left 8th interspace. Esophagus is dissected from the hiatus to the aortic arch, ligated the esophageal arteries and protecting the vagus nerves. The phrenoesophageal ligament is incised circumferentially and the cardia is mobilized. The residual phrenoesophageal ligament and fat pad of the cardia (from vagus nerve to vagus nerve) is resected to provide exposure to the cardia for the myotomy. Myotomy - proximally from the aortic arch distally on to the stomach for a distance of no more than 1 cm. The incision is carried through the muscle to the level of the mucosa. The fibers are then separated to expose about 1/2 the mucosal tube to prevent rehealing. Associated antireflux Procedure - a total fundoplication is not performed as aperistaltic esophagus cannot overcome the pressure at the LES with a Nissan. Modified Belsey Mark IV sutures placed posteriorly in the diaphragmatic crus to tighten it so that it admits an index finger. 4 horizontal mattress sutures are used (at 12 0clock, 3 oclock, 6 oclock and 9 oclock) placed circumferentially in the hypertrophied, thickened esophagus on either side of the pouting mucosa and secured to the diaphragm adjacent to the crus. 2 heavy sutures placed between the edges of the right crus posteriorly and tied down with enough room to admit a finger. Antireflux Procedure or Not? - if the myotomy is not carried onto the cardia, an antireflux procedure is not necessary. Problem is that if the myotomy is not carried down far enough onto the GE junction, the patients dysphagia may persist. Can assure that the myotomy is adequate by carrying it down onto the cardia at least 1 cm, but this necessitates an antireflux procedure. Should consider an antireflux procedure (Belsey Mark IV) with every myotomy.

Scenario #3 - Esophageal CA
55 y.o man with dysphagia and 20 lb. weight loss. UGI shows midesophageal stricture. Issues: a.) Appropriate workup of esophageal stricture. What if scope cant pass stricture? Bronchoscopy? Criteria for resectability. Preop TPN? b.) What are operative options? How are each performed. How would you mobilize the stomach for a pull up? What if it cant be made to reach? c.) Complications of blunt esophagectomy? c.) Difference in prognosis for adeno or squamous? d.) Adjuvant therapy based on nodal status? e.) Intraop treatment of liver nodule? What if unable to get tissue? Celiac node? ANSWER: 90% present with dysphagia and weight loss.

Workup: all patients with dysphagia should have barium swallow and CXR. Radiology - Barium swallow with upper GI - unusual to show proximal dilatation with CA. Length of tumor correlates with the extent and resectability of the tumor. - CT scan of chest and abdomen - if CA present on biopsy. -Bone scan only for bone pain or symptoms of bone pain. Assist in identifying the location and extension of the primary tumor. Note the mets to the lung, celiac axis lymph nodes, liver, adrenal glands or other thoracic or abdominal abnormalities. Can also see extension into the trachea, left main stem bronchus, or mediastinum. -Brain CT for symptoms only. Esophagoscopy - should be performed for histologic diagnosis of the tumor and for evaluation of intramural metastasis. Biopsy and brushings should be performed. Biopsy 70% diagnostic, brushings 90% diagnostic, together 95% diagnostic. Should note location of cricopharyngeus, length and location of tumor and GE junction. Should use pediatric endoscope to visualize GE junction and stomach if tumor is constricting. Bronchoscopy - should be performed for upper and middle third tumors. Bulging of the distal membranous trachea and left main stem bronchus may be tumor and it is essential that these be biopsied/brushed. **THESE STEPS WILL COMPLETE THE CLINICAL STAGING. Nutritional Status - studies show that only those with severe malnourishment benefit from preop TPN. Those with less than 10% weight loss did better with chemo than those with weight loss over 10% (presumably from less advanced disease.) Enteral nutrition is chosen over TPN for preop nutritional support. This can be done with soft silastic catheters placed blindly or endoscopically if severe obstruction is present. Answer - need nutritional support for weight loss>10%, enteral is preferred. SURGICAL OPTIONS AND CONSIDERATIONS: Palliation - surgery provides excellent palliation from dysphagia and is not exceeded by any other modality. Surgical resection must be considered the preferred option if the patient is fit enough to withstand esophagectomy and the tumor is considered to be relatively localized and resectable. If patient felt to be resectable and localized Operative Staging for Resectability Celiac Node: positive with distal third tumor, may still be able to resect. positive with cervical or mid-third tumor, incurable disease, should not resect unless resection is necessary for palliation of significant dysphagia. OPTIONS FOR SURGICAL RESECTION TRANSHIATAL ESOPHAGECTOMY Description:

1.) Abdominal exploration - the tumor is palpated through the hiatus to determine resectability and involvement of tumor with aorta, spine and tracheobronchial tree. If no mets are found (or if there are celiac nodes and dysphagia but no liver or distant mets), the stomach is mobilized preserving the right gastric and right gastroepiploic arcades. Further mobilization of stomach is provided by a wide Kocher maneuver. The esophagus is dissected anteriorly and posteriorly into the mediastinum. If the tumor is adherent to the prevertebral fascia, a right thoracotomy may be necessary for resection. Mediastinum is inspected for bleeding. 2.) Cervical incision obliquely to the level of the left main stem bronchus. Care is taken to stay lateral and posterior to the TE groove and the recurrent laryngeal nerve. 3.) The esophagus is pulled through the posterior mediastinum. 5 cm margin is taken from the GE junction on the lesser curvature. Long gastric tube is made with 5-6 fires of a stapler. The stomach is pulled up through the posterior mediastinum with a penrose drain. A pyloromyotomy/pyloroplasty is performed. Cervical anastomosis is performed and drain is left. NG tube left above the pyloromyotomy. Contraindication to Transhiatal Esophagectomy without Thoractomy Absolute - tracheobronchial invasion. Most Important - surgeons assessment after palpation of the esophagus through the diaphragmatic hiatus that the tumor is fixed to contiguous structures making this unsafe.

ESOPHAGECTOMY WITH THORACOTOMY Description: Lewis/Transthoracic Esophagectomy 1.) Laparotomy performed and patient is operatively staged for resection. If resectable, a conduit is prepared.(usually stomach). If this is an extensive middle third tumor, a transthoracic approach may be performed before celiotomy to assess resectability and to mobilize the esophagus. Should the tumor be unresectable, a laparotomy is avoided. 2.) The thoracic anastomosis is place under the azygous vein. NG tube is placed under direct guidance. 2 chest tubes are placed. Total Thoracic Esophagectomy 1.) Lapartomy performed and patient is operatively staged for resection. If resectable, a conduit is prepared (usually stomach). 2.) If resectable, the neck incision is made and the esophagus is dissected. 3.) Resection of the head of the left clavicle to widen the thoracic inlet. Cervical anastomosis between the cervical esophagus and the stomach/conduit is performed. 4.) Right thoracotomy is performed for esophagectomy. Esophagus is stapled at proximal and distal ends and resected with specimen.

COMPLICATIONS OF PROCEDURES:

1.) Anastomotic leak - 0-25%. Thoracic leak is slightly less common than a cervical leak but less morbid because of the avoidance of mediastinitis. 2.) Anastomotic strictures. - more common with stapled than hand sewn. 3.) Respiratory insufficiency. - pneumonia, aspiration 4.) CHF. 5.) PE 6.) chylothorax - 5%. PATHOLOGY: Squamous Cell CA - 95% of all cases of esophageal CA. Primary tumor may extend into aorta or left main stem bronchus. Distant mets most common to lung and liver. Lymph node mets in 75% of cases. - very aggressive, 5%-12% of patients survive 5 years. - with + LN, five year survival is 3%. - without LN mets, five year survival is 42%. - 70% have LN mets at diagnosis. Adeno CA - 5% of all cases of esophageal CA. - very aggressive, 0-7% of patients survive 5 years. - with + LN, average survival is 9 months. - without LN mets, survival is possible. ** adjuvant therapy based on LN status is possible with clinical trial. Adjuvant therapy is best with combination chemotherapy and shows enhanced survival when combined with locoregional control. Survival is poor.

Scenario #4 - Boerhaaves Esophagus/Spon. Perforation.


67 y.o. male in E.R. 3 hrs after episode of vomiting with development of chest pain and epigastric pain. Issues: a.) Differential Dx (Brief). Workup of esoph. perforation.(include description of tests to rule out differential dx.) Where is leak located? Pathophysiology. GG or Ba. Resuscitation and antibiotics/preop preparation. b.) Appropriate surgical treatment at this time pt. 12 hrs? 24 hrs? 36 hrs? Plans for nutrition? How are each done? c.) Post -op leak - what do you do? Answer: Differential Diagnosis: Perforated esophagus, MI, perforated DU, pancreatitis, pneumonia. Tests: Phys. Exam (crepitus for cervical perforation or intrathoracic perforation, chest exam for possible pleural effusion or mediastinal crunch of mediastinal emphysema), vitals (tachycardia, hypotension, fever), CBC with diff, amylase, lipase, EKG, CXR (upright film to rule out free air). Gastrograffin swallow. Thin Barium swallow if GG swallow negative and still with a high index of suspicion.

Pathophysiology - increased pressure in the esophagus with retching or vomiting causes a linear tear, usually on the left side, almost always in the distal third of the esophagus. Leakage of digestive juices and bacteria into the chest cause a severe mediastinitis and may communicate with the pleural cavity. MANAGEMENT initial - IV fluids, antibiotics - cephalosporin (cefazolin, cefamandole) and aminoglycoside (gent or tobra) to control oral flora. SURGICAL THERAPY: EARLY ESOPHAGEAL PERFORATION (WITHIN 24 HRS OF INJURY) - in those not associated with intrinsic disease, treat with primary repair combined with wide mediastinal drainage. This is performed by opening the mediastinal pleura from the diaphragm to the thoracic inlet over the perforation. Approach - L thoracotomy through 6th or 7th interspace for distal third perforations. - R thoracotomy for more proximal thoracic perforations. Closure - prolene. Repair - should be buttressed. Choice of material is not as important. Fundoplication wrap can be performed for perforations of the distal third of the esophagus or GE junction. The wrap must not be left in the chest or allowed to herniate. Other options include omentum, parietal pleura and intercostal muscle. Grillo flap is a pleural flap wrapped circumferentially around the perforation site. - NGT is placed until the postop ileus is resolved and then oral feedings can be instituted. - Barium esophagram is performed at day 10. Chest tube is left in place until this exam is completed. If this exam shows a leak, the chest tube is left in place and esophagocutaneous fistula is allowed to heal slowly. Most of these will heal if they are small and there is no distal obstruction. Wide mediastinal drainage is key. As long as fistulas are small and well drained, oral feedings can be instituted. LATE ESOPHAGEAL PERFORATIONS (AFTER 24 HRS.) - primary repair of the perforation is not feasible due to friability. Though this is the rule each perforation must be inspected to decide on the ability to perform a primary repair. Critical decision - which patients will benefit from treating an esophagopleural-cutaneous fistula and which will benefit from diversion/exclusion or esophagectomy. 24-48 hrs - T - tube or wide drainage with chest tubes. 48 hrs. - diversion and total exclusion - effectively prevents further mediastinal contamination. Favor end cervical esophagostomy/spit fistula with stapling or ligation of the distal cevical esophagus, ligation of the GE junction and placement of gastrostomy

(to prevent GE reflux into esophagus) and jejunostomy tube for feeding. - should be used in the case of the desperately ill patient. Requires a second operation to restore esophageal continuity, usually with a left colon replacement. ESOPHAGEL PERFORATION ASSOCIATED WITH INTRINSIC DISEASE - while the perforation is quickly diagnosed, primary repair is often unsuccessful because of distal obstruction. Examples: Achalasia - should be treated with suture repair, esophagomyotomy, and fundoplication over the repair (Belsey mark IV) Esophageal CA, hard stricture, caustic injury, trauma - should be treated with resection. If the perforation is diagnosed quickly and the mediastinal contamination is not severe, and the stomach is adequate, a conduit may be fashioned at this time.

Scenario #5 - Zenkers Diverticulum


40 y.o. progressive dysphagia, regurgitation of not too digested food, and bad-breath. Issues: a.) Appropriate workup? b.) Surgical correction? Detailed description with anatomy. Which would you excise or pexy? What size bougie would you use for sac excision? ANSWER: ANATOMY - arises within the inferior pharyngeal constrictor muscle between the oblique fibers of the thyropharyngeus and the horizontal fibers of the cricopharyngeus muscles. The potential weak point is Killians triangle. - the hernia sac gradually enlarges above the cricopharyngeus muscle, eventually extending over the cricopharyngeus and can extend into the superior mediastinum. - it is not only the size of the diverticula that dictates the severity of symptoms, but also the degree of upper esophageal muscle dysfunction. DIAGNOSIS - contrast radiography SURGICAL THERAPY - exposure is through the left neck through an incision which is parallel to the SCM. Dissection is continued in the tracheoesophageal groove beneath the left inferior thyroid artery. - Cricopharyngeal myotomy - relieves the relative obstruction at the level of the UES. Myotomy should extend from the pouch 3-5 cm inferiorly onto the esophagus through the cricopharyngeus muscle. Sac < 2 cm - myotomy is sufficient Sac > 2 cm - smaller - diverticulopexy - pex the sac to the prevertebral fascia. - larger - the sac is stapled over a 50-60 Fr. bougie to prevent narrowing of the pharyngoesophagus and predispose suture line breakdown and leakage.

Scenario #6 - Paraesophageal/ Type II Hiatal Hernia 62 y.o. woman with early satiety and substernal and epigastric fullness after meals. Issues: a.) Appropriate workup of type II hernia? b.) Appropriate surgical repair for type II hernia? ANSWER: TYPE I - SLIDING HIATAL HERNIA TYPE II - ROLLING PARAESOPHAGEAL HERNIA TYPE III - COMBINED PARAESOPHAGEAL HERNIA. - TYPE II ANDIII arise from a protrusion of the peritoneum through the hiatus in to the thoracic cavity. - differentiating type I and type II is based on the position of the GE junction. With a type II hernia, the GE junction is located in the normal position. With a type III hernia the GE junction migrates into the thorax. - Type II and III hernias arise from a weakening of the phrenoesophageal ligament. WorkUp: Barium Swallow - reveals abnormality but cannot always differentiate a type II and III. Fiberoptic bronchoscopy - should be performed to help identify the location of the GE junction as well as detecting any areas of blood loss. Linear ersoions are often seen at the site of the stomach wall adjacent to the ring of the diaphragm which makes up the hernia ring. Acute complications: excessive bleeding or volvulus with gastric obstruction or infarction. Catastrophic complications occur in 20-30% of those with this diagnosis with diagnosed paraesophageal hernias. SURGICAL APPROACH - can be performed through the abdomen or thorax with roughly equivalent results. Basic goals - reduction of hernia sac contents into the abdomen, resection of the hernia sac, closure of the diaphragmatic hiatus to an appropriate caliber, anchoring the stomach within the abdomen Closure of the diaphragmatic hiatus - lateral fibers of the right diaphragmatic crus form the septum between the esophageal hiatus and the aortic hiatus. These septal muscle fibers are attenuated in the case of a type II hernia. Closure of this defect, crucial for the prevention of a recurrent hernia, can be assisted by dissecting the diaphragm from the pericardium for several centimeters. Prosthetic material may be used if necessary to close the defect.

Anchoring stomach - can be performed by : 1.) gastrostomy tube. 2.) suturing the proximal lesser curve to the median arcuate ligament. ?Antireflux procedure - controversial.

Stomach/Duodenum
Scenario #7 - Melena/Bleeding Peptic Ulcer 60 y.o. male with melena. Issues: a.) Initial treatment and diagnostic plan/fluid resuscitation and NGT/stabilize. b.) Most likely diagnosis/ Could this be ZE?/Differential dx of UGI bleed. c.) If NGT returns blood, what do you do? d.) On endo, posterior penetrating duodenal ulcer which is not actively bleeding, but has clot over it, what do you do? Remove clot or leave, inject or not, cauterize or not? e.) Assuming patient bleeds again 12 hrs. later, what do you do? What if the duodenum is scarred from chronic disease? f.) What would be your surgical procedure of choice and how would you perform it? Oversew =/- GDA+/- V&P. What are the potential complications of that procedure? g.) UGI bleed after bypass for unresectable CA. After resuscitation and monitoring, what are surgical options. Answer: Initial treatment - 2 large bore IVs Fluid - LR is chosen over over 0.9 NS because sodium and chloride more closely approximate that of whole blood lost. Because of the inability of renal failure patients to clear this sodium and chloride, colloid/blood and invasive monitoring should be instituted. The massively hemorrhaging patient who is hemodynamically unstable should be transfused with blood to maintain the O2 carrying capacity. Endoscopy can be essential in identifying those lesions which are likely to rebleed (esophageal varices) from those which are not (Mallory Weiss) and can assist in decision to transfuse early. Type and Cross cbc + platelet count - Hct will not reflect blood loss, need clinical eval. LFTs Coags NGT - blood - obvious upper GI bleed - no blood + bilious return - unlikely upper GI bleed (80% accurate). - no blood + no bilious return - need upper endo to rule out DU bleed Monitoring - art. line, telemetry, Foley, ICU. Differential Diagnosis:

3 most common: - Posterior penetrating duodenal ulcer - large gastric ulcers - gastric or esophageal varices. Peptic Ulcer Disease - Duodenal Ulcer - Gastric Ulcer - Marginal Ulcer Gastritis - stress ulceration - alcohol gastritis - drug induced gastritis GE varices Mallory Weiss Esophageal, gastric or duodenal tumor Dieulafoys lesion Aortoduodenal fistula. Esophagitis Angiodysplasia Hemobilia Pancreatitis induced pseudoaneurysm. DIAGNOSIS: - if bilious, nonbloody fluid is returned, lower GI bleed will be diagnosis 80% of time. - if nonbilious/nonbloody or bloody fluid is returned, upper GI bleed is suspected and upper endoscopy is performed. - if upper endoscopy is diagnostic, appropriate therapy is taken. - if upper endo is not diagnostic, decision for next test is made based on the severity of blood loss. In cases of massive hemorrhage (limit of detection is 0.5 -1 mL/min.) a visceral angiogram is necessary. Nonoperative therapy/Therapeutic Endoscopy - Injection Therapy - should be used to inject first around the clot and then in to the center of the fibrin clot. - Heat/Laser Therapy - has been shown in metanalysis to shorten hospital stay and number of units transfused. - Visible nonbleeding vessel with clot at ulcer abase - 80% chance of rebleeding in one study. THOSE WITH BLEEDING VESSEL AND THOSE AT HIGH RISK OF RE-BLEEDING (AS ABOVE) SHOULD UNDERGO EMERGENT ENDOSCOPIC HEMOSTATIC THERAPY - BOTH THERMAL AND INJECTION THERAPY WILL DECREASE THE

REBLEED RATE, NEED FOR EMERGENT SURGERY AND HOSPITAL MORTALITY. HIGH REBLEED RATE ULCERS: - LARGE GASTRIC ULCERS - ULCERS HIGH ON THE LESSER CURVE NEAR THE LEFT GASTRIC ARTERY. - POSTERIOR INFERIOR ULCERS OF THE DUODENUM CLOSE TO THE GASTRODUODENAL ARTERY. SURGICAL THERAPY GASTRIC ULCERS - partial gastric resection to include the ulcer. Biopsy to rule out malignancy necessary. If location of ulcer in stomach is in doubt, use intraop endoscopy to lacate the ulcer. DUODENAL ULCERS 1.) Oversew of Ulcer with Vagotomy and Drainage - Rate of recurrent bleeding after GDA ligation is lower. 2.) Gastric Resection with Excision with Ulcer Bleeding Duodenal Ulcer with scarred Duodenum - rather than risking leak from scarred duodenum or performing suboptimal operation in the face of chronic disease, one should perform a truncal vagotomy and antrectomy (resection + acid reducing procedure) with a gastrojejunostomy vs a gastroduodenal anastomosis. Rebleed - most often attributable to lack of control of the initial bleeding site. If patient rebleeds after oversew and vagotomy/drainage, the vessel should be directly visualized and ligated. If rebleed occurs after oversew of gastric ulcer, a gastric resection plus vagotomy should be performed. Complications of Therapy 1.) Postop GI bleeding 2.) Duodenal pyloroplasty suture line leak. Bleeding after surgical bypass for unresectable CA - punt.

Scenario #8 - Gastric Ulcer 50 y.o. male failing H2 blockers with Type I Gastric Ulcer Issues: a.) Workup of nonhealing gastric ulcer? b.) Appropriate treatment of nonhealing gastric ulcer? c.) How does plan change if biopsy shows malignancy? d.) Detailed description of gastric resection for CA. e.) What is the significance of achlorhydria? f.) What are complications of each surgical options for nonhealing ulcer? (dumping, marginal ulcers, gastric outlet obstruction, efferent loop syndrome)

Gastric Ulcer with Gastric Outlet Obstruction - what is the electrolyte abnormality and how would you go about correcting it. When would you operate and what would you do? Answer: Types of Benign Gastric Ulcer : 1.) Type I - incisura or most inferior portion of lesser curve - not associated with other gastroduodenal disease. Often associated with achlorhydria. 2.) Type II - gastric and duodenal - associated with acid hypersecretion, aka P.U.D. 3.) Type III - pyloric and prepyloric - associated with acid hypersecretion, aka P.U.D. 4.) Type IV - juxtacardiac - like Type I, not associated with other gastroduodenal disease. Workup - most patients present with complaints of vague epigastric pain, which can be caused by a variety of causes, from dyspepsia to gastric CA. - endoscopy - to identify ulcer and biopsy. All gastric ulcers to be observed should be biopsied to rule out malignancy. When taking into account endoscopic appearance, biopsy and brush cytology, 95% of benign gastric ulcers can be differentiated. Medical Treatment - because most patient s with type I gastric ulcers have normal or reduced acid levels, the use of antisecretory drugs may not make sense. However, the further decrease of acid levels may accelerate the healing process. Unlike duodenal ulcers, gastric ulcers healing is due to duration of therapy not the amount of acid depression. H2 blockers - healing rates are similar to duodenal ulcers, however - the duration of therapy needed take 2 weeks longer Omeprazole - similar or better healing rates to H2 blockers. - medical therapy requires reevaluation in 6 weeks to confirm healing and rule out malignancy. SURGICAL THERAPY INTRACTABILITY - when ulcer fails to heal with standard medical therapy, the possibility must be reassessed. Other causes of intractability are Crohn's eosinophilic gastritis, gastric lymphoma, TB, CMV. If ulcer has failed to heal in 12 weeks of maximal medical therapy. 95% of benign gastric ulcers will heal on 40 mg/d of Omeprazole for 12 weeks. - because the role of acid is less prominent in the pathogenesis of gastric ulceration (esp with type I and IV) when compared to mucosal defects, acid reducing procedures are less reliable than those that remove the defective mucosa. Type I Ulcer - antrectomy performed in a manner to include the ulcer. BI reconstruction is preferred when compared to BII because it is more physiologic. Because hypersecretion of acid is not a factor, a vagotomy has no role in addition to the antrectomy. Leak rate is low as the

duodenum is usually normal. Truncal vagotomy and pyloroplasty has little role because of ulcer recurrence rates are higher than antrectomy. COMPLICATIONS OF GASTRECTOMY: 1.) DUMPING - result from extirpation or loss of the sphincter function and loss of the storage function. The addition of a selective and truncal vagotomy may enhance this problem as both enhance the evacuation of liquids from the remnant of the stomach. 2.) DIARRHEA - post vagotomy diarrhea has not been fully elucidated. 3.) MARGINAL ULCER 4.) GASTRIC OUTLET OBSTRUCTION - SEE 3.) 5.) AFFERENT LOOP SYNDROME - occurs after BII, in which there is stasis in the remnant of duodenum and jejunum if there is partial or complete obstruction. Obstruction can occur from stenosis at anastomosis, kinking, volvulus, intusseception. Incidence is greater if the loop is longer (10-15 cm), located on lesser curve, or antecolic position. Can lead to duodenal blowout or reflux symptoms (emesis of bile without food). Causes accumulation of food in afferent limb during eating, has increased pressure resulting in increased pressure and emesis. Diagnosis is difficult. Surgery for resuspension or shortening of the limb may help. 6.) EFFERENT LOOP SYNDROME - symptoms are similar to small bowel obstruction with distention, colicky abdominal pain, emesis of bile. Jejunogastric intusseception is a rare cause and usually occurs with after antecolic BII gastrojejunostomy. Efferent loop intussecepts into the stomach. 7.) ALKALINE GASTRITIS SUBTOTAL GASTRECTOMY FOR CA RESECTABILITY - Peritoneal Seeds - Rectum and pouch of Douglas - Uterus and ovaries in women. - Palpate both lobes of liver. - dissect the greater omentum from the left half of the transverse colon. - mobilize the left colon from the spleen. - lift the stomach cepahlad to determine if the tumor is fixed to the pancreas - determine if the tumor involves the transverse mesocolon. - identify the left gastric artery and follow it to its origin. Biopsy celiac nodes. (If negative, can operate for cure.) - porta hepatis nodes are biopsied. (perihepatic and right gastric nodes are taken with specimen) - determination is made if an adequate gastric/esophageal margin can be obtained with resection. Resection - spleen is mobilized from retroperitoneal attachments and the avascular plane behind the pancreas anterior to the kidney. The spleen is mobilized. - Left gastric artery and vein are found and ligated. The left gastric stump is palpated along with the hepatic artery. The common hepatic artery is traced to the right gastric and gastroduodenal

artery. These vessels are taken and the lesser omentum is taken and dissected toward the gastric specimen. - Duodenum is mobilized via Kocher maneuver. - Line of resection is chosen and Kocher clamps placed (5cm prox and 8-9 cm distal) and the stomach is divided. Spec. is taken for frozen. Once frozen is returned negative, the small medial vessels just past the pylorus between the duodenum and the pylorus are taken - duodenum is transected 1 cm past the pylorus and the frozen is sent. The duodenal stump is oversewn. The proximal gastric stump is oversewn. Gastrojejunostomy is performed. Scenario #9 - Recurrent PUD. Issues: a.) Appropriate workup for recurrent PUD. b.) Workup for ZE. c.) Appropriate surgical treatment for ZE. Answer: Causes of Postoperative Recurrent PUD: Inadequate Surgery: - Incomplete vagotomy - Retained Antrum - Inadequate gastric resection - Stenotic gastric outlet. - Long Afferent Loop. Other causes: - Gastrinoma - Primary hyperparathyroidism - Antral G cell hyperplasia - Duodenogastric or jejunogastric reflux. - H.pylori DIAGNOSIS EGD - allows biopsy for evaluation of retained antrum, eval duodenal mass (gastrinoma), or gastric outlet obstruction. BASAL GASTRIN - 100-200 rules out gastrinoma 200-1000 suggestive of gastrinoma >1000 diagnostic of gastrinoma **increase of at least 200 following injection of secretin with gastrinoma

Retained antrum - basal gastrin 2-3 times normal - minimal effect on secretin stimulation - confirm by technitium scan Antral G cell hyperfunction basal gastrin levels 2-3 X normal ** 200-300% increase in gastrin levels with a high protein meal minimal effect of secretin infusion. Sham Feeding - 1 hr basal acid output (BAO) + 1 hr stimulated acid output (SAO) + 1 hr peak acid output following pentagastrin infusion (PAO) nonvagotomy - SAO is 40% PAO 100% vagotomy - SAO is 10% PAO (SAO is abolished and PAO is reduced) ** if BAO is elevated the sham feeding test may yield a false positive for incomplete vagotomy. Medical Therapy - indicated as it will heal most ulcers. 4 weeks minimal treatment is necessary and 12 weeks may be needed if the ulcer is healing. Ulcer after 12 weeks requires surgery. Surgical Therapy - indications include failure of medical therapy or complication on presentation. Incomplete vagotomy - as determined by BAO >2mmol/hr. postoperatively or + sham feeding should be treated with revagotomy - transthoracic vagotomy should be used in elective setting. This should not be used in face of a complication. Transabdominal approach allows evaluation of anastomisis stenosis, retained antrum, present of gastrinoma. * risk of leaving vagus intact after second procedure is high, so operation of greater magnitude should be chosen (antrectomy, subtotal gastrectomy. WORKUP OF ZOLLINGER ELLISON: TEST #1 - peripheral venous gastrin. Gastrin Interpretation 0-100 Normal 100 - 200 Indeterminate 200-500 Presumed Z.E. > 500 Diagnostic TEST #1.5 - Secretin stimulation for indeterminate gastrin - 2 U secretin per kg of body weight - obtain serum gastrin 10 and 5 minutes before and 1, 2, 5, 10, 20, 30 minutes after injection. Absolute increase of 200 is diagnostic. TEST #2 - CT SCAN - given with IV contrast - gastrinomas are seen as bright spots of hypervascularity on scan. Better for detecting metastasis than for detecting primary tumors (detects only 30-40% of primary tumors). Sensitivity of MRI is identical to that of CT scan. TEST #3 - SELECTIVE ANGIOGRAPHY - single best study as it reveals 50% of primary tumors and accurately detects liver mets. Selective venous sampling after secretin injections can be combined with selective angiography to enhance localization.Catheters are placed in the gastroduodenal, hepatic and splenic arteries and the hepatic vein. Secretin in injected into the arteries and blood is collected from the hepatic veins as well as a peripheral vein and gastrin

levels are measured. The artery that results in highest gastrin level after injection of secretin provides the likely area of gastrinoma. -secretin angiography results in diagnosis of 75% of gastrinomas. TRANSHEPATIC PORTAL VEIN SAMPLING - catheter is placed through the liver into the branches of the portal vein and blood is collected for gastrin levels. The branch closest to the tumor will yield the highest gastrin level. 80% diagnostic rate, but associated with hemobilia and bleeding in 10% of cases. SURGICAL THERAPY OF GASTRINOMA 80% of gastrinomas located in the gastrinoma triangle. Important - the pancreas and duodenum should be completely mobilized as tumors can be found within the wall of the duodenum. Intraop US has not been shown to be beneficial in localization of tumors. Localize the CBD (with probe) and the major and minor papilla with secretin (1U/kg). - rather than proceeding with a Whipple procedure for tumor s of the duodenal wall or the head of the pancreas, it is recommended that these tumors be enucleated and that lesion of the tail or body of the pancreas should be resected. Tumors of the body and tail seem to be more malignant. Liver mets - intraop US can be used to detect liver mets that cant be felt. These should be removed by wedge resection if possible at the same operation. Removal by formal lobectomy is advocated by some. Gastrinoma in lymph nodes should be resected. Metastatic gastrinoma - 40-50% will have localized disease amenable to resection. 80% of those operated on will have resections. Those with nonresectable tumors should be treated initially with H2 blockers or omeprazole. No Tumor Found 1.) Treat with H2 blockers if effective preoperatively. 2.) Treat with total gastrectomy if H2 blockers have not been effective preoperatively. Scenario # 10 Scenario #10 - Perforated Duodenal Ulcer with Prior History of PUD 18 y.o. foreign exchange student with history of epigastric pain treated with antacids presents with abdominal pain. Issues: a.) Initial treatment of the patient including IVs, antibiotics, and x-rays.

b.) What is the operative treatment assuming that you find a 4mm perforation just at the distal lip of the pylorus. c.) What are the expected results and complications of the various surgical modalities that you have to choose from. d.) What would you do if there is a 5 year history of epigastric pain but no definite diagnosis of PUD? e.) describe the operative technique of Graham patch and highly selective vagotomy. ANSWER: Situations: I.) No antecedent history, no risk factors II.) No antecedent history, + risk factors - omental patch III.) Antecedent history, no risk factors IV.) Antecedent history, risk factors - omental patch Initial Treatment - IVs, Foley, fluid resuscitation, NG tube aspiration. Antibiotics - Broad spectrum directed at mixed enteric flora. X-rays - upright abdominal radiographs should show free air, but in 20% of those with perforation, will not show free air. - Upper GI series performed with water soluble contrast may be helpful if perforation is suspected but not demonstrated with pneumoperitoneum. ?ULCER HISTORY - either by history or anatomic evidence of duodenal scarring should be sought. Many (2/3) of those with perforation will have chronic disease and will not be treated adequately with simple omental patch. 3+ Goals of Treatment 1.) patient safety 2.) peritoneal cleansing 3.) closure of perforation (4.) alteration of ulcer diathesis is minimized if applicable. OPERATIVE RISK FACTORS: 1.) Concurrent medical illness 2.) preoperative shock 3.) perforations of greater than 48 hrs. duration. Those with these risk factors: peritoneal irrigation with omental patch closure. Can be treated postop with antisecretory meds and delayed surgery if necessary. Those without risk factors (history or anatomic evidence of chronic ulceration, no shock preoperatively, no life-threatening comorbid disease, perforation present for less than 48 hrs.) Definitive ulcer procedure did not produce longer hospital stay or mortality. Should have closure of perforation and definitive ulcer procedure if applicable. Results of Treatment -

Omental Patch - Recurrence = Acute disease - 36% recurrence Chronic disease - 60-80% recurrence - Complications - leak or recurrent ulcer Arguement for definitive surgery - by 5-6 years out the recurrence rate for acute and chronic ulcers meet. Treatment with H2 blockers or antisecretory therapy means a lifetime of treatment. However, the anti ulcer procedure must be one associated with few side effects- i.e. highly selective vagotomy Highly Selective Vagotomy a.k.a. Proximal Gastric Vagotomy - Recurrence = Acute disease - 10% Chronic disease - 3-16% Study Comparing PGV, Truncal vagotomy, Omental closure Recurrence 4% 12% 63% Graham patch - 3-4 000 sutures are placed in the duodenum 1-1.5 cm from the edge of the site of perforation passed through the site of perforation with an equivalent mirror bite on the opposite side. (prevents getting backwall with closure). These sutures are placed but not tied. A tongue of omentum is placed over the site of perforation and the sutures are tied down. Abdomen is irrigated with attention to the retrohepatic, subhepatic, perisplenic and retrogastric spaces. Proximal Gastric Vagotomy - Incision - Midline or right paramedian with extension to the xiphisternum. Round ligament of the liver is divided between ligatures. Place Omni retractor with 4 blades and retraction cephalad. The left triangular lobe of the liver is incised to the IVC and the left lobe of the liver is retracted. Anterior Vagus Nerve first - first identify the nerve of Latarjet which is located 5-7 cm from the pylorus. Incise the peritoneum overlying the lesser curve beginning 7 cm from the pylorus, the branches are identified as they pass into the anterior surface stomach. Nerve branches are dissected individually between the anterior nerve of Latarjet and the lesser curve of the stomach. Nerve branches are divided between ligatures. Meticulous dissection is crucial and hematoma formation will obscure anatomy. - dissection of the posterior leaf can be simplified by rotating the stomach anteriorly. The posterior peritoneum is incised 7 cm from the pylorus and continued proximally - peritoneum over the esophagus is incised and esophagus is encircled with a penrose drain. To ensure that all branches of the GE junction are taken, the intraabdominal esophagus is dissected circumferentially. The vagal trunks are dissected away from the esophagus.

Scenario #11 - GERD 38 y.o. female with retrosternal pain after eating, worse at night, with emesis of undigested food. a.) Appropriate workup of GERD? b.) How should each type/severity of GERD be treated? c.) What are surgical indications? What procedure should be performed? How is this done? What should be done in case of stricture? Answer: WORKUP: HISTORY: Dysphagia - troublesome bit of history, that. Causes are multiple, ranging from slow passage of food through the distal esophagus because of severe Barretts or severe esophagitis to benign stricture (Schatzkis ring) to CA. Barium Swallow - will show spontaneous reflux only 40% of time. Therefore, lack of reflux on esophagram does not rule out reflux. Benefit is ruling out a foreshortening of the esophagus and lack of suitable intrabdominal esophagus for a Nissen wrap. Endoscopy allows evaluation of: - distal esophagus - metaplastic changes c/w Barretts or changes - evaluation of stricture as chronic fibrosis vs. spasm. GRADING OF ESOPHAGITIS I - reddening of mucosa without ulceration. II - erosive and exudative mucosal lesions, which are found in a linear fashion. III - cobblestone esophagus - extensive mucosal involvement with islands of edematous squamous mucosa. IV - defined by the presence of complications - large ulcers, stricture formation, shortening or the presence of columnar metaplasia. 24 HR. pH MONITORING pH studies - most sensitive test for the diagnosis of GERD -Before a diagnosis of GERD can be made, it must be proven that the esophagus is indeed exposed to excessive amounts of acid or alkaline material. - test is performed by placing a pH probe 5 cm above the manometrically defined GE sphincter. normal - rarely rises above pH of 7 and rarely falls below 4. Need to take into account the frequency and duration of episodes where pH is under 4. Daytime reflux in excess of 90 min. is abnormal. Any nighttime reflux is abnormal. Esophageal Manometry resting pressure best measured above the GE junction, where respiratory wave inversion (negative) is noted lower limit of normal = 6mmHg normal resting pressure = 15 mmHg

Manometry - controversial -dysphagia as a presenting symptom indicates need for manometry especially if stricture or intra-abdominal obstruction is absent. Generally hypotensive LES. Indicated whenever esophageal motor abnormality is suggested by dysphagia or odynophagia and the endoscopic exam does not show clear structural abnormality. Lack of abnormal cardia manometry contraindicates performance of an antireflux procedure. SELECTION OF PATIENTS: Precise diagnosis of cause of abnormal acid exposure to the esophagus is necessary. (antireflux procedure should not be performed in the absence of abnormal cardia manometry symptoms may be from duodenogastric reflux from distal obstruction or transient relaxations of LES, neither of which will benefit from antireflux procedure) Indications to perform antireflux procedure in the presence of abnormal cardia manometry: - COMPLICATIONS OF REFLUX - persistent endoscopic esophagitis, esophageal stricture, Barrets esophagus an d the documentation of recurrent aspiration pneumonia. - UNWILLINGNESS TO ACCEPT MEDICAL THERAPY - Endoscopic Esophagitis - because the development of esophagitis is related to the composition of the refluxed material - not all of those with incompetent sphincters will have esophagitis. These patients rarely respond to medical therapy and eventually require a surgical procedure. - Benign Esophageal Stricture - presence of stricture from esophagitis secondary to reflux in the presence of an incompetent LES is a failure of medical therapy and an indication for surgical intervention. Malignant stricture is ruled out before surgery via endoscopy. Prior to surgery the stricture is dilated with a 60 Fr. bougie. When dilated, manometry and 24 hrs pH monitoring is performed. Manometry - to assess distal esophageal peristalsis and rule out achalasia. pH - if normal may indicate alkaline reflux or stricture from other causes. - Barretts Esophagus - complication of persistent reflux esophagitis, always associated with mechanically deficient LES. At risk for development of stricture, large ulcers and development of adenoCA. Multiple biopsies are indicated, and grade III dysplasia or higher indicates esophageal resection.

Small Bowel
Scenario # 12 - Crohns with Enterovesical Fistula 30 y.o. with history of ileal Crohns well controlled presents with pneumaturia.

Issues: a.) Appropriate workup of a patient with Crohns disease and complication of Crohns? b.) With enterovesicle or colovesical fistula, is cystoscopy or cystogram necessary? c.) What operation would you perform? How much ileum would you resect? Would you perform a R hemicolectomy? Enterovesical Fistula - 2-5% of patients with Crohns, usually causing pneumaturia and symptoms associated with recurrent UTIs (urgency, frequency, dysuria). Recurrent UTIs danger to permanent renal damage along with persisitent intestinal disease make this an urgent indication for surgery. Radiologic evaluation of Crohns disease - contrast studies necessary to delineate the extent of disease and the severity of disease, especially in those being readied for surgery. SURGERY - resection of diseased segment of small bowel with extirpation of the fistulous tract. The opening to the bladder is usually at the dome and can be debrided and closed without affecting the trigone. Foley catheter is left in place for several days and is only removed after radiographic confirmation of the healing of the bladder. Closed suction is placed near the repair. Repair only the diseased small bowel, need for R colon is dictated by the extent of the diseased small bowel. Colovesical Fistula - BaE rarely fill the fistula. Cystoscopy will show hyperemia and inflammation consistent with chronic cystitis, but usually do not localize the fistula. The current best test is a CT scan with intraluminal contrast.

Scenario # 13 - Progressive/refractory Crohns with

stricture

40y.o. male with Crohns distal ileal stricture refractory to medical therapy. Issues; a.) Appropriate workup of Crohns disease and stricture? Nutritional status. b.) Appropriate preparation for surgery including steroids and TPN/is enteral diet preferable. c.) What is in TPN, how would you write for it? How long would give TPN? d.) What are some electrolyte abnormalities expected with diarrhea and dehydration from Crohns, and how would TPN help this? Indications for surgery in Crohns: 1.) Septic complications 2.) intestinal obstruction 3.) failure of medical therapy WorkUp - contrast radiography - SBFT - to evaluate the extent of disease the location of the stricture and extent of disease. - BaE - to evaluate for colonic disease or distal strictures which may complicate a simple small bowel resection. Nutritional support - nutritional support of malnourished patients has been shown to improve such nutritional parameters as body weight, visceral protein status, and nitrogen balance. Indications:

1. Preop nutritional support for the malnourished patient. 2. short gut syndrome 3. fluid and electrolyte replenishment for those with jejunostomy 4. *chronic small bowel obstruction TPN vs. Enteral formulas - no clear benefit of one over the other - studies have shown that there is not a clear benefit of TPN in the preoperative patient unless malnourished. Preop TPN is saved for those with the most severe disease and the severely malnourished NUTRITIONAL ASSESSMENT WEIGHT LOSS - weight loss of more than 10% of normal body weight may compromise the host by altering the ability to heal wounds and ability to develop an immune response the injury. Operative mortality is increased with loss of 20-25% of normal body weight. 40% weight loss is life threatening. - those without weight loss and do not have hypercatabolic state do not require nutritional support for 5-7 days. ALBUMIN/SERUM PROTEIN - has been shown to correlate with protein calorie malnutrition, increased hospital stay and poor outcome. Problem - tend to be low in septic patients because of acute phase reactants. TPN: Basal Energy Expenditure - BEE Male BEE = 664+ 13.7W+5H- 6.7A Female BEE = 655+ 9.6W+ 1.8H-4.6A Additional Calories - added if the patient is desired to gain weight. 500 calories - to gain 1 lb. per week 1000 calories - to gain 2 lb per week Protein Needs: - 0.8 grams per kilogram per day in the normal nonstressed patient - range is from 0.8-1.5 gram per kilogram per day based on the amount of stress. 1.5 gr per kg per day is only for those with substantial losses from wounds or fistulas. Carbohydrate and Fat Mixture - providing the necessary glucose for function of CNS, nervous system and blood will prevent the breakdown of protein to amino acids which will be converted to glucose for these tissues. Optimal Glucose Administration- 4.5 gr per kg per minute or 450 gram per day in 70 kg male.

- excessive carbohydrate will result in fatty liver and increased C02 production. - based on 4.5 gram per kg per minute of carbohydrate will result in 30% of calories from fat. ENERGY PER GRAM OF NUTRIENT: CARBOHYDRATE - 3.4 KCAL PER GRAM PROTEIN - 4 KCAL PER GRAM FAT - 9 KCAL PER GRAM Scenario #14 - Enterocutaneous Fistula 45 y.o. female who undergoes an LOA for SBO S/P hysterectomy. On POD#5, pt. develops drainage from wound growing enteric organisms, persistent drainage of 2 liters per day. Issues: a.) How should one evaluate and treat this wound? b.) 2 days after wound is cared for bilious drainage is noted on dressing, what does this suggest? c.) What is appropriate workup and therapy for enterocutaneous fistula? d.) A SBFT shows no distal obstruction, how would you treat this fistula? e.) Under what circumstances will this fistula not heal? What operative timing is necessary? f.) How specifically would you handle the TPN? g.) How would you change your management if this patient was septic? ANSWER: WOUND CARE: Initially, skin edges opened and treated with wet to dry dressings. Ostomy appliance if drainage is too brisk. Once fistula has been diagnosed: Fistula drainage is usually best controlled by use of a sump, usually a soft brown latex catheter with a vent is used (like a Robinson nephrostomy catheter) Skin care - ileostomy appliance with use of Karaya seal/powder placed and left for 7-10days to allow the skin to heal underneath. Classification of enterocutaneous fistulas: LOW OUTPUT FISTULAS/COLONIC FISTULAS = < 200 cc/24 hrs. MODERATE OUTPUT FISTULAS = 200-500 cc/24 hrs. HIGH OUTPUT FISTULAS = > 500 cc/hr. *though spontaneous closure is not strictly related to amount of output, the rate of complications is. Workup: Fistula gram - usually performed at 7-10 days at which time the patient is nutritionally more sound and the fistulous tract has had more time to mature so that water soluble dye can be used. Small feeding tube is used to infuse contrast through the feeding tube. Important data from fistulagram: (1.) site of entry of the bowel (2.) nature of adjacent bowel (inflamed, strictured) (3.) presence or absence of intestinal continuity (side fistula or

anastomotic blowout. (4.) presence or absence of distal obstruction. (5.) size of adjacent abscess if present. FACTORS MITIGATING AGAINST SPONTANEOUS CLOSURE OF FISTULA: 1.) ANATOMIC SITE: More Likely to Close Spontaneously: - lateral esophageal fistulas - lateral duodenal fistulas - biliary and pancreatic fistulas - jejunal fistulas Less Likely to Close Spontaneously: - gastric fistulas - ileal fistulas - fistulas of the Ligament of Treitz 2.) NATURE OF ADJACENT BOWEL: Disease processes which are less likely to heal: - inflammatory bowel disease - radiation enteritis - CA 3.) ADJACENT ABSCESS: LARGE -will not close spontaneously SMALL - may close spontaneously 4.) DISTAL OBSTRUCTION - PRESENT - will not close - ABSENT - may close spontaneously TREATMENT: NON- OPERATIVE - followed for 4-5 weeks to allow spontaneous healing. OPERATIVE - if fistula fails to heal in 4-5 weeks with maximal nutritional support. - early operative intervention for sepsis or progressive organ dysfunction NUTRITIONAL SUPPORT Electrolyte disturbances - disturbances in potassium, sodium, magnesium and if TPN has been used phosphate. Malnutrition - more common with high output fistulas and in the presence of sepsis. If the malnutrition is related to sepsis, the source of the sepsis i.e. abscess must be dealt with first. CT

scan can be used to localize abscess and then be percutaneously drained. If it cannot be localized the patient must be explored and the abscess drained and the fistula closed and bowel resected. TPN - because of the likelihood that the patient has lost 300-500g of lean body mass per day, the daily TPN requirements will be greater than normal. Protein = 1.7 gr protein per kg per day (corresponds to 250 mg nitrogen per day) Carbohydrate + Lipid - should be given as 75% glucose and 25% lipid emulsion Trace elements should be given and attention given specifically to Mg, Ca and PO4

Scenario #15 - Acute Mesenteric Ischemia 55 y.o. male 5 mo. S/P MI with sudden onset mid epigastric abdominal pain and nausea. Normal vitals except for rapid, irregular pulse Issues: a.) Differential diagnosis (including MI) b.) ABCs and initial resuscitation. c.) Initial evaluation including labs, flat and upright, ultrasound and possible arteriogram. What are the indications for a mesenteric arteriogram prior to OR? d.) Assuming the angio shows cutoff of the proximal SMA near the takeoff of the middle colic artery, what do you do? e.) Assuming that this patient has had an MI 5 months ago, how would reduce the cardiac morbidity during operative procedure? f.) Scenario #15a. 5 feet of gangrenous ileum? Ileum plus embolectomy? g.) Scenario #15b. Majority of small bowel is dark and dusky, how do you deal with this? h.) Describe the operative technique of an SMA embolectomy and a SMA bypass. i.) Describe how you would assess the small bowel? What would you do if you could not tell the viability? j.) What is the appropriate post-op workup for embolus - cardiac source, hypercoaguable state? Appropriate treatment long term? Answer: Background - Acute mesenteric ischemia describes a spectrum of bowel injuries from thrombosis of mesenteric vessels ranging from reversible alterations in bowel function to transmural necrosis of the bowel wall. Causes: - SMA embolus - nonocclusive mesenteric ischemia - SMA thrombosis - SMV venous thrombosis SMA EMBOLUS - usually originate from LV or left atrial mural thrombi. May be dislodged after dysrhythmia or cardiac cath. Emboli usually found at point of narrowing (branching point). Pathophys. - collateral circulation is sufficient initially, but vasoconstriction proximal and distal to the point of occlusion reduces blood flow through collaterals and worsens ischemia.

NONOCCLUSIVE MESENTERIC ISCHEMIA - thought to result from mesenteric vasoconstriction initiated by vasoactive medications or by a period of hypotension from dysrhythmia, MI with myocardial depression or hypovolemia. Vasoconstriction can persist after the initiating cause has been corrected. Precipitating factors - MI, CHF, aortic insufficiency, hepatic insufficiency and renal failure. Frequently mesenteric ischemia can show up hours to days after initiating event. SMA THROMBOSIS - occurs at the area of severe atherosclerotic narrowing, usually at the origin of the SMA. Acute mesenteric ischemia is frequently superimposed on chronic mesenteric ischemia and patient will often have had abdominal pain, weight loss and malabsorption in the weeks and months before the acute episode. Clinical Response to Acute Mesenteric Ischemia - within hours of ischemia the bowel becomes hemorrhagic and edematous, increasing intramural pressure. This can further compromise already marginal blood flow through the capillary system. the flow of fluid into the bowel wall can result in hemoconcentration and hypovolemic shock. These effects can lead to death of a patient before full scale bowel wall necrosis appears. Presentation - pain out of proportion to physical findings. Passage of blood y maroon stools can precede abdominal pain. Nausea, vomiting back pain are late sign soften indicating compromise of bowel. Lab Tests - cbc - leukocytosis > 15,000 occurs in 75% of patients with ischemia. - Hct elevated may indicate hemoconcentration, but this is nonspecific. - ABG - 50% will have metabolic acidosis, but this is a late sign usually indicating bowel - necrosis. *early in course, labs may be unremarkable. RADIOGRAPHIC EVALUATION Flat and Upright Early/Before Infarction - Normal Progression - gasless abdomen, adynamic ileus and pseudo-obstruction Late - small intestinal thumbprinting Pinkyprinting - pneumatosis or air in mesenteric or portal veins is a late sign. DECISION - SURGERY OR ARTERIOGRAM SURGERY - evidence of peritonitis. Angiogram may assist OR planning, but should only be done if the pt is not stable. ARTERIOGRAM - equivocal physical findings, hemodynamically stable. - used to diagnose major arterial occlusion by embolus and thrombosis as well as diagnosing nonocclusive mesenteric ischemia by seeing - narrowing of branches of SMA

- alternating narrowing and dilation of mesenteric branches - spasm of mesenteric arcades - impaired filling of mesenteric vessels. -> these are found in absence of pancreatitis and use of vasoactive drugs for proper diagnosis. General Principles of Treatment - initial treatment must be aimed at correcting the cause of ischemia in case of nonocclusive disease. Swan Ganz catheter should be placed to evaluate filling pressure and cardiac output as well as the presence of sepsis. - plain films should be performed to rule out other causes of abdominal pain. - if no other diagnosis is made based on abdominal films, the patient should be taken for an SMA arteriogram. -even if decision has been made to take the patient to the operating room, arteriogram should be done to manage the patient properly at celiotomy. MANAGEMENT AT LAPAROTOMY -> RESTORE ARTERIAL FLOW+RESECT BOWEL RESTORE ARTERIAL FLOW - first SMA EMBOLUS Minor Emboli - emboli branches of SMA or distal to the takeoff of the ileocolic. Can be treated expectantly if pain is relieved by vasodilator therapy. Major Emboli - at or proximal to the takeoff of the ileocolic. Can attempt vasodilator therapy if the patient has significant contraindications to surgery (recent MI), no peritoneal signs and improvement of flow distal after instituting vasodilator therapy. Embolectomy - SMA is approached through the transverse mesocolon. The SMA is dissected proximally between the pancreas and the fourth portion of the duodenum. The SMA is dissected proximally 2-3 cm and distally to the origin of the middle colic. Embolus is located by palpation and Doppler exam. Proximal and distal control obtained. Longitudinal arteriotomy made and embolus and debris removed. Balloon catheter embolectomy is performed. Arteriotomy is closed with vein patch or not. ACUTE SMA THROMBOSIS- diagnosed on arteriogram by SMA that is out and distal filling through collaterals. I f confused between thrombosis and embolus - treat as embolus. Important to differentiate acute thrombosis from old thrombosis - old thrombosis is seen with large celiac and SMA collaterals. Lack of collaterals dictates intervention. Options: - Reimplantation - thrombectomy and endarterectomy - bypass NONOCCLUSIVE MESENTERIC ISCHEMIA ANGIOGRAPHIC SIGNS: - narrowing of branches of SMA - alternating narrowing and dilation of mesenteric branches - spasm of mesenteric arcades

- impaired filling of mesenteric vessels. - PAPVERINE INFUSION BEGUN AND CONTINUED INTO OR - at celiotomy, manipulation of the SMA is minimized. Overtly necrotic bowel is resected and primary anastomosis is performed if no second look operation is planned. Papaverine infusion continued after OR and angiograms repeated daily until symptoms resolved. TESTS OF BOWEL VIABILITY 1.) EYEBALL - bowel placed in warm lap pads for 10-20 minutes and observed for peristalsis, pulsation, color. 2.) Doppler of antimesenteric side of bowel 3.) Surface fluorescence with flouroscein dye using ultraviolet light/Woods lamp Patterns: - fine, granular pattern of viable bowel - patchy, perivascular and nonfluorescent pattern of nonviable bowel. DECISION TO RESECT: - for short segments clearly necrotic and questionably viable after revascularization are resected. - for larger amounts of bowel, only clearly necrotic bowel is resected and plan is made for a second look laparotomy. This also allows further supportive therapy for improvement of bowel. Primary anastomosis should not be done. All or most of small bowel dusky - question whether to do any thing.

Colon
Scenario #16 - Ulcerative Colitis 40 y.o. male with history of U.C. presents to ER with history of bloody diarrhea and approx. 8 bowel movements per day. Issues: a.) Initial resuscitation and treatment. b.) If pt. does not improve, are steroids indicated? If so, what dose and for how long? c.) On the 5th day of hospitalization, patient develops abdominal distention and x-rays show a 10 cm. transverse colon and pt. becomes toxic, what do you do? d.) Describe the operative procedure and options of choice including possible pouch procedure, and colostomy. Answer: Part 1 - fulminant Ulcerative Colitis Part 2 - toxic Megacolon Initial resuscitation:

IV Fluids Electrolyte resuscitation NG suction Broad spectrum antibiotics to cover anaerobic and aerobic gram neg. TPN - improve nutritional status Steroids- most are on steroids and will need stress doses. Should be given trial of bolus steroids at a dose of Hydrocortisone 100 mg IV q 8 hrs.. Duration of nonoperative therapy - Fulminant UC - > 6 bm per day, should continue for 5 days and operate if there is no improvement. Surgery is indicated if patients condition worsens. Toxic megacolon - indication for surgery. Morbidity and Mortality Associated with Operation of Toxic Megacolon Overall mortality - 8.7% After total abdominal colectomy with Hartmanns = 6.1% After total proctocolectomy = 14.7% - conservative therapy better in this setting, total abdominal colectomy and colostomy. Allows for later mucosal proctectomy and ileoanal/pouch procedure. Scenario #17 - UGI/Small Bowel Bleed 32 y.o. male with Fe deficiency anemia and maroon stools intermittently. Negative UGI, Ba enema, small bowel enteroclysis and Meckels scan. Issues: a.) when should you operate? b.) if arteriogram shows small bowel source, should you operate? What do you do if no gross lesion is seen intraop? Possible sources of Occult (Upper) GI hemorrhage 1.) Colonic diverticula 2.) Vascular ectasia 3.) AV malformation 4.) Rupture pseudoaneurysm 5.) Bleeding into pancreatic pseudocyst 6.) Hemobilia 7.) Hemosuccus pancreaticum 8.) Small bowel diverticula 9.) Small bowel neoplasm ANGIOGRAPHY - not only important for diagnosis but also for treatment of upper and lower GI hemorrhage. In some cases angiographic control is more effective and expeditious than operative. Ruptured mesenteric aneurysm, hemobilia, hemosuccus pancreaticum - situations in which operation has poor chance of controlling the site of bleeding. Diagnostic and therapeutic angiography is procedure of first choice. Bleeding - after nondiagnostic endoscopy, should be taken to angio suite if bleeding is at a rate of > 1mL/min. If bleeding localized, start vasopressin at 0.2-.4 U/min for 20 minutes,

followed by a second arteriogram. If still bleeding, increase to 1U/min for 10 min. If unsuccessful, can try embolization with autologous clot, gelfoam. - temporary control is often successful. Emergent surgery is indicated for continued bleeding or recurrent bleeding. - bleeding of less than 0.5mL/min. has low diagnostic yield. - if no bleeding site is found, Priscoline 25 mg or papaverine 10 mg IV bolus can be used to assist in delineating site prior to repeat angiogram. Either embolization or infusion of vasoconstrictors can have complications: mucosal necrosis, perforation and late stenosis. Radionuclide scanning - no place in patient with hemodynamically significant t bleeding. Role is in patient who demonstrates episodic, recurrent, occult upper GI bleeding or in patients suspected of having hemobilia, bleeding into pancreatic duct, or a pseudocyst. Scenario #18 - Appendicitis vs. PID 20 y.o. female presents with RLQ pain and nausea. RLQ tenderness and guarding on exam. Normal rectal and pelvic exams. Issues: a.) differential diagnosis and initial workup including cbc, UA, urine pregnancy and possible abdominal X-rays and ultrasound. b.) What operative approach would you use? c.) What do you do if the appendix is normal? d.) what do you do assuming there is some serosanguinous fluid in the pelvis and a red purulent mass in the right fallopian tube - do you do a right salpingo-oophorectomy? e.) What would you do if there is a perforated DU? a cecal mass showing a cecal adenoCA? Differential Diagnosis: -appendicitis -PID - ruptured/non ruptured ovarian cyst - ectopic pregnancy - ovarian tumor - diverticulitis - UTI/cystitis - Crohns disease -endometriosis - torsion of adnexae Labs: - cbc - UA

- urine pregnancy - pelvic US to rule out ovarian or tubal pathology. - plain films of abdomen. Possible operative approaches: 1.) laparoscopy 2.) RLQ incision - normal appendix = if no other pathology - take out. - red purulent mass in the fallopian tube and serosanguinous fluid in the pelvis. PID ACUTE SALPINGITIS - can be treated as outpatient if nontoxic with cefoxitin (2g IM) or ceftriaxone (250 mg IM) plus doxycycline (100 mg po bid for 10-14 days). TUBO-OVARIAN ABSCESS THAT IS NEITHER RUPTURED OR LEAKING - admit to hospital, keep NPO, treat aggressively with combination antibiotic therapy. Frequent examination, vitals, urine output. Any evidence of small leakage or perforation, then-> life threatening situation LEAKING OR RUPTURED TUBO-OVARIAN ABSCESS OR THOSE NOT RESPONDING TO MAXIMAL MEDICAL THERAPY surgery usually consists of total abdominal hysterectomy and bilateral salpingo-oophorectomy, antibiotics and pelvic drainage as the infection involves both tubes and ovaries. exception - young woman of low parity who has apparent unilateral disease (normal appearing tube and ovary on the other side) may perform a unilateral salpingo-oophorectomy in attempt to preserve reproductive function.

Scenario # 19 - Bowel Obstruction - Large vs. Small

bowel

78 y.o. female from Nursing Home with nausea, vomiting and abdominal distention and old midline incision. Issues: a.) ABCs, fluid resuscitation, tubes, labs, X-rays, physical examination. b.) Assuming that no hernia is found, how would you evaluate this patient radiographically? How would you treat this patient? c.) If studies show a colon obstruction, what is the differential Dx. How should each of these forms of colon obstruction be treated? d.) If studies show a small bowel source, how do you decide if it is complete or incomplete? Would a small bowel study help? How do you manage this based on the results? e.) If hernia (internal or groin) is noted and loop of small bowel is strangulated and necrotic, how should this be dealt with? f.) What if on exploration, the pt. has tumor in the pelvis causing the obstruction? Bypass or ostomy? Tubes: NGT, Foley Labs: cbc, lytes, chem panel. CEA if concerned about a colon CA.

X- rays: flat and upright abdominal films. Physical Exam: Abdominal Exam for presence of ventral hernias, peritoneal signs, presence of groin hernias. Radiographic Exam #1 - rule out colon obstruction with Gastrograffin enema #2 - once colon obstruction is ruled out and plain films suggest SBO, can proceed with SBFT Complete obstruction - OR Incomplete obstruction - as long as there are no peritoneal signs and no evidence of a complete obstruction, may follow nonoperative therapy for 48 hrs, then opt for surgery.

Cause of SBO Percentage Adhesion 75% Malignant tumor 8% Hernia 8% Internal 20% External 80% Volvulus 3% Cause of Large Bowel Obstruction Percentage Cancer 65% Volvulus 15% Diverticulitis 10% Hernia Carinomatosis Pelvic Recurrence of Rectal CA. SURGICAL MANAGEMENT OF SMALL BOWEL OBSTRUCTION: Bowel Viability: Flouroscein - hyperemic, normal and fine granular patterns of viable intestines. - patchy, perivascular or nonfluorescent patterns of nonviable bowel. Doppler : apply to antimesenteric border. Doppler signal within 1 cm of proposed anastomosis predicts success. Adhesionolysis.

Special Considerations: Benign Stricture - can be treated with stricturoplasty. Make enterotomy, pass balloon tipped catheter into stricture to insure luminal diameter of 1.5 to 2 cm. Short strictures- 3-4 cm. Open stricture lengthwise 1 cm beyond its ends and close transversely in a Heineke - Mikulicz fashion in double or single layer closure. Long stricture - up to 12 cm. - can be used in a side to side enteroenterostomy fashioning a Finney technique with sutures or staples. > 15 cm - resection. Incarcerated Hernia Sites - femoral, internal inguinal, stomal, obturator or umbilical ring. Close reduction - only performed if strangulation is not suspected. Approach 1.) Groin incision - risks of premature reduction and loss of control of potentially strangulated bowel. 2.) Midline 3.) Preperitoneal Intusseception Adults - 30% of enteric intusseception and 65% of colocolonic intussesuption in adults is secondary to a malignant process, so resection is performed over reduction in adults with intusseception. The exception is intusseception that occurs in the early postop period. Malignant Obstruction - tempting to pursue conservative approach, but aggressive approach better as the patient will eventually require an operation. As many as 33% of obstructions will be due to adhesions, and 60-90% will be palliated. Definitive procedure should be performed(bypass, resection or ostomy) whenever possible. Postoperative Obstruction (within first 2 weeks of surgery) - observation is performed for first 48-72 hrs. As long as the patient is improving after this initial period, conservative treatment is continued. 10-14 days of tube decompression for partial obstruction is indicated before considering surgery. SBFT can help differentiate ileus from mechanical obstruction. SURGICAL MANAGEMENT OF LARGE BOWEL OBSTRUCTION COLONOSCOPY - ESSENTIAL IN INITAL EVALUATION BECAUSE IT CAN DIAGNOSE PARTIAL OBSTRUCTION AND TREAT SOME COMPLETE OBSTRUCTION. PARTIAL OBSTRUCTION - from CA or diverticulitis can be diagnosed with endoscopy and wire/ catheter can be placed through site of obstruction. DIAGNOSE VIA COLONOSCOPY OR CONTRAST ENEMA AND PREPARE FOR SURGICAL RESECTION.

COMPLETE OBSTRUCTION - from CA or volvulus. Can differentiate cecal volvulus from colonic pseudoobstruction. Can treat sigmoid volvulus prior to definitive surgery. Specific situations: Cecal Volvulus - emergent surgical treatment is indicated because 30% of time nonviable bowel is present and resection is indicated. If nonviable bowel is present, resection with or without anastomosis is indicated. If viable bowel present, options include cecostomy or cecopexy or both. Sigmoid Volvulus - initial decompression is performed with rigid sigmoidoscope. Prep and perform complete colonoscopy then resection during same admission. COMPLETE OBSTRUCTION FROM BENIGN (ISCHEMIC STRICTURE OR DIVERTICULITIS) OR MALIGNANCY - initial decompression can be performed with transverse loop colostomy or fully diverting colostomy. MALIGNANT OBSTRUCTION High risk patient, large fecal load, massive dilation --> decompressive colostomy/staged procedure. Dilation, ischemia, perforation, peritonitis, large fecal load --> resection with delayed anast. Moderate dilation and fecal load, no ischemia, stable patient --> resect with anastomosis

BILIARY

Scenario # 20 - Gallstone Ileus


60 y.o. female previously healthy who presents with crampy abdominal pain, vomiting and jaundice. Issues: a.) Initial workup and differential diagnosis. b.) How do you deal with the obstructive aspect of a gallstone ileus. c.) how do you deal with the gallbladder aspect of a gallstone ileus. d.) If the patient has an inflammatory mass making dissection of the gallbladder and duodenum difficult, how do you deal with the jaundice? ANSWER: Differential diagnosis: Acute cholecystitis Chronic cholecystitis

Pancreatitis Biliary stricture Diagnosis: accounts for 15% of simple complete small bowel obstruction for those over 70. Plain Abdominal Films: Air in biliary tree - 55-60% of patients. Show pattern of bowel obstruction Stone seen 20% of time. Removal of Stone - attempt to compress the mass proximally and then remove it through a proximal enterotomy. Since stones often cause irritation and ulceration, they may not be amenable to dislodgment, in which a limited small bowel resection of the segment containing the stone is necessary. Check the remainder of the small bowel for additional stones. If stone is removed through an enterotomy, the enterotomy is closed in two layers. Closure of Cholecystoenteric fistula - 10% risk of recurrent gallstone ileus after enterotomy alone. Removal of gallbladder and closure of enteric portion of fistula is performed. 1.) cholecystectomy 2.) Cholangiogram - to see if CBD exploration is needed. 3.) closure of small bowel component of cholecystoenteric fistula - 2 layer closure with 3.0 vicryl Connell suture running and interrupted 3.0 silk seromuscular. Avoid narrowing the lumen.

Unable to close cholecystoenteric fistula - If this is too hazardous or the patient is not doing well, clean out the gallbladder of stones and place a no. 30 mushroom catheter. In this case, leave for 3 months before tube cholangiogram and pulling catheter. Given the advanced age of patients, the risk of another gallbladder complication may be worth it. Scenario #21 - Painless jaundice S/P cholecystectomy 64 y.o. female with 1-2 week history of progressive jaundice. PMHx of cholecystectomy. Issues: a.) What is the appropriate workup for this patient with painless jaundice including initial labs and US? How would you attempt to get a detailed anatomical map of the biliary tree? b.) If distal CBD obstruction is noted, how would you proceed? Describe the exploration in detail? What should you do if you are unable to get cholangiogram dye into the duodenum c.) If on CBDE a stone easily removed with clean cholangiogram, how should one proceed?

Scenario # 22 - Ascending Cholangitis S/P cholecystectomy Part #1 - Infection/Critical Care/Stabilization - 65 y.o. male S/P cholecystectomy in E.R. complaining of dyspnea, severe epigastric and RUQ pain fever and chills. Hypotension, jaundice tender RUQ, guarding. Issues: a.) What is the initial evaluation and intervention (ABCs)

b.) What interventions are needed if the patient persists with hypotension and respiratory distress? c.) What testing is necessary (labs, US)? How can biliary duct be accessed if there is no gallbladder for cholecystostomy? How should the patient be stabilized for the OR? d.) If the biliary duct can not be accessed for drainage, and patient continues to deteriorate clinically, what should be done? e.) What should be done if a stable patient on the way to the OR becomes unstable intraop? Part #2 - Distal CBD obstruction. a.) In a stable patient, what are options for stone that cannot be retrieved from the distal CBD? b.) How should CBD drainage be performed? c.) Describe in detail how to perform a CBD exploration + choledochoduodenostomy as well as a transduodenal sphincteroplasty. ANSWER: PART #1 - EVALUATION AND TREATMENT OF CHOLANGITIS ETIOLOGY OF BILIARY TREE OBSTRUCTION: 1.) common duct stones - most common etiology =70% 2.) benign biliary strictures 3.) malignancy 4.) obstructed biliary -enteric anastomosis. A = if in respiratory distress or unresponsive/lethargic because of sepsis, intubate. B = if stable airway, start on 02, 4-10L by facemask. Titrate up with 02 sat. C= IVs - 2 peripheral IVs ASAP. - NS at 125cc/hr. - Start IV antibiotics - Foley - titrate fluid to keep UO = 30cc/hr. Persistent hypotension refractory to fluid resuscitation - admission to ICU for invasive monitoring and pressor support. Dopamine is preferable at least low dose because of the high risk of renal failure in these patients. Antibiotics - traditionally amp, gent and flagyl have been used as traditional triple therapy until identification and sensitivity is back. Piperacillin and mezlocillin can be used in place o f combination of aminoglycoside and ampicillin and is less nephrotoxic. Guiding therapy specifically at antibiotics that have high biliary concentrations has not been shown to be of use in this situation. If an aminoglycoside is chosen, serum creatinine and peaks and troughs must be followed closely because of the high risk of renal failure in these patients. Laboratory cbc - leukocytosis usually present, but leukopenia in severe sepsis.

LFTs - serum bilirubin and alk phos are elevated in patients with cholangitis and levels are higher in those with malignant obstruction. - transaminases are elevated in most patients with higher levels being seen in benign disease. Amylase - elevated in 33% of patients, in these cases usually benign cause. PT/PTT - may be elevated inpatients with liver dysfunction. CA 19-9 - elevated in cholangitis, normal when resolved. Blood Cultures - identifying causative organisms. Radiographic Evaluation - US and CT scan. Both good for showing intra and extra hepatic bile duct dilatation. CT scan is better for showing CBD stones and periampullary tumors. Cholangiography. Biliary Decompression Cholangiography - if defervesces, cholangiography can be performed after afebrile for 24 hrs. Emergent Cholangiography and Urgent Biliary Decompression - for those who do not demonstrate clinical and lab evidence of improvement within 12-24 hrs and those with septic shock. How? - studies have shown that the complication rate, rate of retained stones, length of dependency on a ventilator, and mortality are all higher in those who have septic shock and have surgery compared to those who have early endoscopic drainage. Patients with indwelling catheters - place to external drainage, treat with supportive and antibiotic therapy. Patients with contraindications to endoscopic biliary drainage - such as those with a BII gastrectomy or those in whom the ampulla of Vater cannot be cannulated or those with malignant obstruction or benign biliary stricture - PTC and external drainage are preferable. Patients in whom PTC is contraindicated - those with ascites, those with uncorrected coagulopathy or those with nondilated intrahepaic bile ducts. Patients in whom endoscopic biliary drainage cannot be performed and percutaneous biliary drainage is contraindicated or cannot be performed - stabilization and OR for CBD exploration. Unstable patient intraop who had been stable preop - T-tube and get out. PART #2 - EVALUATION AND TREATMENT OF CBD OBSTRUCTION ABCS - fluid resuscitation as indicated for hypovolemia secondary to emesis, etc. Antibiotics - enteric bacteria should be presumed to be present in the bile of patients with primary common duct stones. Instrumentation of CBD during diagnosis, elevated CBD pressures from obstruction all necessitate starting prophylactic antibiotics. Piperacillin/mezlocillin + Flagyl as above. Laboratory: Alk Phos - alk phos most sensitive indicator.

Bilirubin - chance of finding stone rises as bilirubin rises. Amylase - associated pancreatitis, usually normal PT/PTT - may be elevated in patients with liver dysfunction cbc- leukocytosis usually seen with cholangitis, usually normal with CBD stone. Transaminases - along with wbc, amylase are usually normal. RADIOLOGIC EVALUATION: US - can show CBD dilatation, but is less accurate at demonstrating CBD stones(15-30%) CT scan - also excellent for demonstrating intra and extrahepatic ductal dilatation. More sensitive than US at detecting CBD stones. (75-90%). Also better for detecting the level of obstruction and presence or absence of pancreatic mass. CHOLANGIOGRAPHY ERCPcomplications - severe bleeding and duodenal perforation occur most often when there is a sphincterotomy performed. previous BII gastrectomy, biliary stricture, malignant obstruction may make ERCP difficult. PTC - contraindications - ascites, uncorrected coagulopathy and narrow caliber biliary ducts. TREATMENT OPTIONS: ENDOSCOPIC SPHINCTEROTOMY - diagnostic ERCP performed first. One or two small stones can be removed through an intact papilla. However, for more extensive stone extraction an endoscopic sphincterotomy must be performed. This technique is difficult with large stones, large tortuous ducts and stones secondary to multiple strictures Contraindications - coagulopathy and long biliary strictures

LAPAROTOMY WITH COMMON DUCT EXPLORATION- common duct opened after a Kocher maneuver and palpation of the intrapancreatic duct. - 8F or 10 F soft rubber catheter is passed and used to irrigate the duct. A fogarty catheter is passed proximally and distally until all the stones are removed. - choledochoscopy - can reduce the incidence of retained common duct stones to 2-4 % without increasing mortality. Any residual stones can be removed with Fogarty catheters. - T tube - used in order to perform postop cholangiogram as well as to extract stones through the tract. if this is to be done, at least a 14 Fr. tube should be used. - Completion Cholangiogram - has been shown to decrease incidence of retained stones by 75%. No passage of contrast into the duodenum - due to spasm of the sphincter of Oddi or the effects of narcotics. Spasm can be relieved by glucagon, the effects of narcotics on the sphincter can be reversed by naloxone.

Laparoscopic CBDE - relative contraindication is numerous stones in the CBD. Elect either an open CBDE or postoperative endoscopic spincterotomy. DRAINAGE PROCEDURE : Open Sphincteroplasty Indications: - sphincter stenosis or dysfunction - possible ampullary tumor (represents an opportunity to do a biopsy) - recurrent pancreatitis - multiple stones in a nondilated system - Choledochocoele Perform kocherization of duodenum, CBDE and palpation of intrapancreatic portion of the duct first. Identify the ampulla by passing a Fogarty catheter through the ampulla into the duodenum. Transverse duodenotomy made over the ampulla. Two stay sutures (3.0 silk) are placed on either side of the ampulla and lifted forward. IF pancreatic duct is identified at the 4 oclock position, place a lacrimal duct probe in for identification. Make a small (1-1.5 cm) incision at 11 oclock position. Imbricate edges with interrupted 5.0 PDS. Progressively extend incision with imbrication until a Bakes Dilator the size of the CBD can be passed. Be sure to place an apex suture to prevent duodenal leak. Close transverse duodenotomy transversely to prevent narrowing. Choledochoduodenostomy Indications: **- primary common duct stones - multiple stones - long distal inflammatory stricture - Perivaterian duodenal diverticulum Can be performed in a side to side or end to end fashion: SIDE TO SIDE: Key - place choledochotomy low on the common bile duct for CBDE. Make incision 2-3 cm longitudinally. Place stay sutures of 4.0 silk. Make longitudinal duodenotomy (small because it will stretch) over the CBD over the first portion of the duodenum generally 7-8 cm from the pylorus. Close with single layer of 4.0 prolene such that all sutures are placed on the outside. Place the first stitch at the duodenal apex farthest away and then through the inferior portion of the choledochotomy. Complete the back row first, tying all of the sutures at the end, starting with apex and proceeding with the order placed. Place T-tube above the anastomosis. Potential Problem - Sump Syndrome - food from the distal duodenum is caught in the distal CBD and serves as a nidus for infection/bacterial growth. Eventually the anastomosis or pancreatic duct can become obstructed leading to pancreatitis or cholangitis. This can be prevented by tying off the distal duct and performing : END TO SIDE: transection of distal CBD and anastomosing end into the side of the duodenum using single layer 5.0 PDS and placing a T-tube above the anastomosis. ** place closed suction drains near but not at the anastomosis.

Choledochojejunostomy - can be done with limb of jejunum in continuity or Roux en Y, though a Roux is preferable. In either case the CBD should be transected and the end anstomosed to the side of the jejunum to prevent sump syndrome. Roux - brought retrocolic to provide an isoperistaltic, defunctioned drainage of the CBD with low tension anastomosis and low risk of reflux of gastric contents and cholangitis. Place a Ttube above the anastomosis. If the anastomosis is too close the hepatic bifurcation, place stents. - jejunojejunostomy is performed 60 cm downstream. TECHNICAL DESCRIPTION = COMMON BILE DUCT EXPLORATION: 1.) KOCHER MANEUVER - incise the lateral peritoneal band s of the duodenum. 2.) CHOLEDOCHOTOMY - remember = CBD lateral, hepatic art medial and portal vein posterior. 2-3 cm incision anterior lower mid CBD distal to CBD and cystic duct junction (make sure that the cystic duct isnt opened). 4.0 silk stay sutures over incision. 3.) PALPATION OF CBD - milk stones proximally and distally through the choledochotomy. 4.) RED RUBBER ROBINSON CATHETER - 8-10 FR. - irrigate duct for stones. 5.) FOGARTY - if stones are not removed, pass biliary Fogarty into duct proximally and distally and gently remove with balloon inflated. 6.) CHOLEDOCHOSCOPY - flexible choledochoscope is passed to visualize the distal CBD to the ampulla of Vater. Passed proximally into the liver to visualize the right and left biliary radicals. Biliary Fogarty can be passed into the CBD through the scope to remove stones. 7.) Stone forceps - use only if all else fails. Tend to damage duct wall. 8.) T-TUBE - use 14 FR. or large to allow postop stone retrieval. TECHNICAL DESCRIPTION = TRANSDUODENAL SPHINCTEROPLASTY INDICATED FOR STONE IMPACTED IN THE AMPULLA THAT CANNOT BE REMOVED BY ANY OTHER MEANS 1.) KOCHER MANEUVER - for better exposure and to allow tension free duodenal closure. 2.) DUODENOTOMY - longitudinal incision 3-4 cm is made over the medial aspect of the duodenum. Palpate the stone to make sure the ampulla is in the center of the incision. Place duodenal traction sutures. 3.) IDENTIFICATION OF THE AMPULLA - in case of CBDE where the CBD is now clean use a biliary Fogarty. Otherwise, feel junction of the third and fourth portions of the duodenum and milk bile from distal CBD to locate ampulla. The main pancreatic duct is located inferior border of the ampulla, slightly to the right and may be deep or superficial. 4.) INCISION - after the ampulla is located, insert a probe into the distal CBD. Incise along probe for 2-4 cm at the 11oclock position. Should see pancreatic duct inferiority within first 1 cm. 4.0 or 5.0 interrupted PDS are used to approximate the duodenal and ductal mucosa. Place apical suture to prevent duodenal leak. 5.) IDENTIFICATION OF PANCREATIC DUCT - should be seen after stone is removed by drainage of clear pancreatic secretions. If not use secretin, 1 mg/kg IV.

6.) CLOSURE - longitudinal incision is closed transversely like a Heineke - Mikulicz pyloroplasty in 2 layers. TECHNICAL DESCRIPTION = CONSTRUCTION OF ROUX EN Y

Scenario #23 - Lap Chole Pt. with symptomatic cholelithiasis. Issues: a.) Describe a lap chole in detail. b.) Describe what you would do if 6 stones were spilled? ANSWER: Yeah, right.

HEPATIC/PORTAL HYPERTENSION
Scenario #24 - Hepatoma/Liver Mass. 45 y.o. female presents with RUQ mass which moves below the ribs on inspiration. Issues: a.) What is the differential diagnosis. b.) What is the workup (CT scan, AFP, CEA, CXR, labs). How would you rule out hemangioma? How would you evaluate patients hepatic function. c.) Assuming the CT scan shows an 8 cm calcified mass on the right lobe of the liver, what would you do? d.) What incision would you use? How would you evaluate the patient for resectability? Would you biopsy this mass? e.) Describe in detail the performance of a right hepatic lobectomy. f.) Assuming bile comes from your drain on the 5th post-op day, what would you do for diagnosis and treatment? ANSWER: BACKGROUND: Study done on patients with asymptomatic liver masses showed that 51% were benign hemangiomas and 17% were malignant. Unable to identify clinical, lab or radiographic test which could differentiate a benign from a malignant lesion. Suggests malignancy - age > 55yrs., palpable mass, elevated alkaline phos. DIFFERENTIAL DIAGNOSIS OF LIVER MASS: BENIGN: CYSTS CYSTADENOMA ABSCESS ADENOMA FNH HEMANGIOMA

HEMATOMA AVM OR ANEURYSM MALIGNANCY: HEPATOMA METASTASIS CHOLANGIOCA CYSTADENOCA SARCOMA Labs: cbc - thrombocytopenia and neutropenia may reflect hypersplenism. Coags - elevated PT after administration of vit K reflects decreased liver synthetic function. lytes liver function tests - to assess for underlying liver disease or measure compromise from mass. Transaminases - elevations from ischemia or hepatitic processes. Alk. Phos. - elevations due to biliary obstruction Bilirubin - elevation in absence of biliary obstruction speaks toward impairment of hepatic function from acute or chronic disease. Albumin - decrease may be due to decreased synthesis, malnutrition, sepsis. hepatitis screen AFP - elevated in those with cirrhosis or hepatitis, but rarely above 400 in absence of HCC or germ cell tumors. Levels greater than 400 highly suggestive of HCC and mandate investigation. CEA - not usually elevated in HCC, so elevations may indicate new metastatic lesion. CXR CT scan RADIOGRAPHIC EVALUATION: US - accurate at detecting focal lesions but less accurate at detecting diffuse liver disease. Can be used to see splenomegaly, flow reversal in portal vein, ascites or other signs of chronic liver disease. Use to detect size and cystic or solid. CT scan - dynamic CT involves the injection of contrast and serial CT imaging. Incremental CT - serial imaging 20 sec after injection of contrast to allow contrast to be at its highest density as it passes through the hepatic arteries. Delayed scans done 2-3 minutes later. Arteriographic CT - images obtained as contrast is injected into the SMA/hepatic artery. ** though there are patterns that can strongly suggest a malignant or benign lesions, there are no absolute criteria. The accuracy of these techniques is often outweighed by the complication rate from either CT arteriogram or portography.

Radionuclide - 99mTC labeled rbc scan can be used to diagnose hepatic hemangioma. Arteriogram - can determine the proximity of the lesion to major vascular structures, anatomy of lesion itself, and the vascularity of the lesion. Gold standard for hemangioma though can usually be ruled out by US and tagged red cell scan. Role is less prominent than in past. Sequence of Radiographic Exams #1 - ULTRASOUND - determines whether mass is cystic or solid and whether it is single or multiple. Cystic + Single - likely benign Cystic + Multiple - benign cystic disease/abscess Solid + Multiple - likely metastasis. Solid + Single - needs careful further evaluation to determine if malignant. #2 - Tagged RBC scan - if US suggests possible hemangioma, this can be ruled in or out by tagged rbc scan. - Sulfur colloid scan - should be used in place of tagged rbc scan if central scar noted on US. Used to rule in or out FNH. If positive, no further workup is necessary. #3 - Enhanced/Dynamic CT scan - perform if hemangioma screen via tagged rbc scan is negative. CT scan is performed with CT angio or CT portography to help delineate relationship of lesions to major vascular structures. #4- Percutaneous Biopsy - performed if result will change therapy. US guided or blind - performed in those in whom nonsurgical therapy is being considered (as in the patient with cirrhosis or to confirm diagnosis of mets to liver which would not be amenable to surgical resection.) Should not be performed if patient has a lesion which is suspicious for HCC and is seen as resectable by preoperative screening. RESECTABLE HCC, ADENOMA AND METS SHOULD BE RESECTED. RESECTION OF ADENOMA - SURGICAL RESECTION IS THE PRIMARY THERAPY FOR ADENOMA. Resection eliminates risk of bleeding or hemorrhage which occurs in 2025% of patients, and rules out HCC. Resection for FNH - asymptomatic FNH can be treated conservatively if HCC is ruled out as these lesions never rupture or bleed. Resection performed in large symptomatic lesions and in those whom HCC cannot be definitively ruled out. Try to rule out this lesion with sulfur colloid scan. RESECTION OF HEPATOMA: Prognosis - dependent on tumor size, tumor growth rate, functional hepatic reserve, presence of extrahepatic spread. CRITERIA FOR RESECTABILITY: 1. LESION MUST BE LOCALIZED TO A PORTION OF THE LIVER SUCH THAT RESECTION WILL LEAVE WITH 20% OF WELL VASCULARIZED LIVER 2. NO EXTRAHEPATIC DISEASE 5 YEAR SURVIVAL IN PATIENTS RESECTED WITH CIRRHOSIS = ZERO.

RESECT FOR PALLIATION - RISK OF BLEEDING, ETC. = NO. EMBOLIZE FOR BLEEDING. TECHNICAL DESCRIPTION = RIGHT HEPATIC LOBECTOMY 1.) INCISION = bilateral subcostal with midline extension for full mobilization. 2.) EXPOSURE = use Omni Retractor for retraction of costal margins. 3.) MOBILIZATION OF RIGHT LOBE - right triangular ligament is taken down and the falciform ligament is incised, and bare area of the liver is mobilized off the diaphragm. The right branch of the hepatic artery is found and dissected out behind the CBD and isolated. The right branch of the portal vein is isolated (usually on right side of CBD). For vascular tumors, these should be clamped now. The liver is retracted to the left and anteriorly by assistant on left (help the doctor) to expose the IVC. The IVC is cleared off to the level of the right hepatic vein. The right adrenal vein is identified, ligated and divided. Small veins from the right lobe to the IVC are ligated. 4.) DISSECTION - if the hepatic artery and portal vein branches have not been clamped, do it now to demarcate the right liver. Incise the liver peritoneum with cautery, divide parenchyma by finger dissection, clamping and dividing vascular structures (vascular structures with Prolene and biliary structures with vicryl). Dividing the major vessels within the liver prevents inadvertent ligation of aberrant vascular anatomy. Finally, the right hepatic vein is divided and ligated.

Scenario #25 - UGI Bleed/ Portal HTN with GE Varices. 40 y.o. female with alcoholic cirrhosis and GE junction varices referred by gastroenterologist. Issues: a.) How would you assess this patients functional status? b.) When patient develops hematemesis, how would you evaluate this? Discuss resuscitation and ABCs. c.) How would you treat this patient? What is the appropriate dosing of Pitressin? What are contraindications to its use? describe in detail the placement of a Sengstaken Blakemore tube. How should the patient be further worked up once the bleeding is under control? f.) What are the indications for a shunt to control bleeding? g.) What is necessary anatomically to perform a Warren shunt? What other shunts are possible if the splenic vein is clotted or too small for a Warren shunt? FUNCTIONAL STATUS = CHILDS CRITERIA FOR HEPATIC RESERVE CRITERIA CLASS A CLASS B CLASS C (minimal) (moderate) (advanced) Bilirubin <2 2-3 >3 Albumin >3.5 3-3.5 <3

Ascites None Easily controlled Poor control Neurologic Disorder None Minimal Poor control Nutrition Excellent Good wasting Labs: cbc - anemia from bleeding. Leukopenia and thrombocytopenia from splenomegaly. Coags - elevated PT after administration of vit. K reflects decreased liver synthetic function. lytes - abnormalities seen with cirrhosis and ascites include hyponatremia, hypokalemia, metabolic acidosis. BUN, Creatinine - elevated Cr. may reflect prerenal azotemia, recent GI bleed or hepatorenal syndrome. liver function tests - to assess for underlying liver disease or measure compromise from mass. Transaminases - elevations from ischemia or hepatitic processes. Alk. Phos. - elevations due to biliary obstruction Bilirubin - elevation in absence of biliary obstruction speaks toward impairment of hepatic function from acute or chronic disease. Albumin - decrease may be due to decreased synthesis, malnutrition, sepsis. hepatitis screen. T&CX6U prbc. Airway - if unstable from massive hematemesis, shock, or decreased level of consciousness from worsening encephalopathy. Breathing - if airway stable, start 02 at 4-10 L by facemask. Titrate by O2 saturation. Circulation - peripheral IVs (14-16 gauge) X2. Resuscitate with.D51/2 NS and blood as needed. Transfuse with FFP. Place Foley and titrate fluid to UO of at least 30cc/hr. Place art line. UPPER ENDOSCOPY - crucial in initial evaluation of the site and cause of GI bleeding in patient with GI bleed and portal hypertension. Bleeding usually from esophageal varices, gastric varices or hypertensive portal gastropathy, but can occur from Mallory Weiss or peptic ulcer as these occur more often in this population than in the normal population. TREATMENT OF BLEEDING VARICES - mortality in bleeding episode is directly related to the hepatic functional reserve as those with extrahepatic portal vein thrombosis with portal HTN do not die of bleeding while those with decompensated cirrhosis die at a rate of 50%. MUST CONSIDER 3 SITUATIONS: ACUTE BLEEDING, DEFINITIVE THERAPY TO PREVENT RECURRENT BLEEDING AND PROPHYLACTIC THERAPY FOR THOSE THAT HAVE NOT YET BLED. ACUTE BLEEDING: ABCs and resuscitation, then upper endoscopy. Visualization of medium and large varices, even if not bleeding confirm varices as the cause of hemorrhage. Endoscopy is crucial because it localizes bleeding site to esophagus or stomach (from gastric varices or portal gastropathy) and will dictate treatment. Because of the comorbid factors associated with cirrhosis and portal HTN, the risk of surgery for acute bleeding is very high. This is why sclerotherapy and ligation of varices is the mainstay of treatment if applicable. However, sclerotherapy and ligation of varices cannot be used for gastric varices and portal gastropathy, so these patients need early surgical therapy.

ENDOSCOPIC SCLEROTHERAPY - sclerose with sodium morrhuate or sodium tetradecyl sulfate or ligate with rubber bands. Early therapy should be directed at bleeding varices and followed up at 1-2 weeks until all varices are ligated. - endoscopic therapy = 65-90%. Standard of therapy. When is pharmacological Rx or balloon tamponade used instead of endoscopic therapy? When acute bleeding is so brisk that endoscopic therapy can not be performed. Complications of Endoscopic Therapy - recurrent bleeding (bleeding episode after 2 sessions) is most common. Others: esophageal stenosis and perforation, fever. PHARMACOTHERAPY - GOAL IS TO LOWER PORTAL PRESSURE BELOW 12MMHGTHE CRUCIAL LEVEL. This is performed by splanchnic vasoconstrictors which decrease portal inflow. Vasopressin - Bolus = 20U IV - Infusion = 0.2-0.8 U/min. Nitroglycerin - 40-400 micrograms/minute depending on blood pressure. Problem with vasopressin - also causes vasoconstriction of other beds causing coronary vasoconstriction, decreased cardiac output, hypertension, bradycardia. Many of these side effects are eliminated by using nitroglycerin.

BALLOON TAMPONADE - 3 SITUATIONS: 1. THOSE WITH MASSIVE HEMORRHAGE PREVENTING ENDOSCOPIC THERAPY. 2. PERSISTENT BLEEDING AFTER ENDOSCOPIC THERAPY. 3. TEMPORIZING PATIENTS IN TRANSIT. Advantage - cessation of bleeding in most cases. Disadvantage - persistent bleeding. ** because of bleeding after deflation, need to plan for definitive therapy while tube in place. TECHNICAL DESCRIPTION: SENGSTAKEN BLAKEMORE TUBE BEFORE: Consider nasotracheal/orotracheal intubation Attach NGT to tube above esophageal balloon Evacuate blood from stomach with large tube. Insert through nose.

AFTER: Low, intermittent suction to stomach balloon Continuous suction to NGT. Inflate stomach balloon by 25cc increments to total of 100cc. Pull inflated stomach balloon to GE junction and fix to nose with slight tension using foam tape. Add additional 150 cc to gastric balloon. Clamp on tube to prevent loss of air from gastric balloon. Inflate esophageal balloon to 25-45 mmHg. X-Ray - confirm position of balloon check Hcts q 6 hrs. Deflate balloon after 24 hrs. Leave in place for 24 hrs., remove if no further hemorrhage. EMERGENCY SURGERY FOR BLEEDING: rare in the day of TIPPs, but present in 2 situations: - failure to control bleeding after 48 hrs. maximal therapy. - recurrent hemorrhage after 2 sessions of sclerotherapy. (remember that gastric varices and portal gastropathy cannot be treated by sclerotherapy or SB tube, only by medical treatment) **approach - aggressive, as mortality climbs as long as the patient continues to bleed. OPERATIONS; 1.) ESOPHAGEAL TRANSECTION - easier for those not familiar with shunt surgery. No clear benefit based on mortality. Vagus nerves dissected away from GE junction. Anterior gastrotomy, EEA stapler placed in GE junction through gastrotomy. Silk suture used to tie esophagus to stapler in open position. Stapler fired. Confirm 2 doughnuts. Close gastrotomy. 2.) PORTOCAVAL SHUNT 3.) INTERPOSITION MESOCAVAL SHUNT. 4.) DSRS - only if preop angio showing adequate anatomy and either no or medically controlled ascites. Bleeding Algorithm - Rikkers, p. 668. DEFINITIVE THERAPY FOR PREVENTION OF RECURRENT HEMORRHAGE. PHARMACOTHERAPY - beta blockers can reduce bleeding rate by 33%, but works best in those who have varices but have not yet bled. Compliance problematic. LONG TERM SCLEROTHERAPY - benefit for long term not as clear as acute bleeding, but because of noninvasive nature has become more popular than shunting. Failure rates of sclerotherapy is 20-40%, so patient must be near a center where definitive therapy can be delivered. TIPPS - long term results for prevention are not yet known, though TIPPS tend to have failure both early and late from graft thrombosis. Ideal for the transplant candidate. NONSELCTIVE SHUNTS - problem - deprive liver of nutrients and hepatotrophic hormones and shunt cerebral toxins away from liver to the brain. TYPES: -end to side portocaval shunt. -side to side portocaval shunt -**interposition mesocaval shunt - good for the transplant pt. (CENTRAL) splenorenal shunt with splenectomy - proximal splenic vein to left renal vein.

use in emergency setting when TIPS not an option and with bleeding from portal hypertension in patient with intractable ascites. SELECTIVE SHUNTS - distal splenorenal shunt Contraindications: - intractable ascites - operation can worsen ascites. - incompatible anatomy - vein smaller than 7 mm - previous splenectomy - clotted splenic vein - splenic vein and left renal vein separated by more than one vertebra on angio. ? no portal flow to liver - not a contraindication, can be reestablished with ligation of collaterals. DISTAL SPLENORENAL SHUNT - ligation of coronary vein - ligation f IMV - removal of splenic vein from portal vein and oversewing of splenic vein stump. The splenic vein is brought down to the left renal vein and anastomosis performed. PARTIAL SHUNT - portocaval shunt using a 8-10 mm interposition graft. Only recently has this form of therapy been used consistently, so studies for efficacy are not done. PREVENTION OF INITIAL EPISODE OF VARICEAL BLEEDING - therapy must be minimally invasive and have low morbidity as only 33% of those with varices will bleed. 1.) propranolol 2.) ?surgery - not effective as prophylaxis.

Pancreas
Scenario #26 - Pancreatitis/ Pancreatic Abscess Woman with idiopathic pancreatitis develops worsening abdominal pain, fever, tachycardia and tachypnea. a.) ABCs. b.) What is the appropriate workup for this pt. with worsening AIP? c.) What are the Ransons criteria? d.) What should be done if fluid/nonperfused pancreas is seen on scan? e.) What if pus is demonstrated on CT scan? f.) If debriding necrotic pancreas and end up with dark blood oozing from pancreas, what should you do? Airway = if unstable from decreased level of consciousness from sepsis, intubate

Breathing = if airway stable, start 02 at 4-8 L by facemask and titrate by 02 saturation. This resolves but should be treated with diuresis. Hypoxemia - often CXR is clear with early hypoxemia. Pulmonary edema on CXR often shows up 2-4 days after initial episode. Circulation = treat aggressively, because if patient becomes hypovolemic and the splanchnic circulation is compromised the pancreas may be ischemic. May need to place Swan Ganz catheter and monitor resuscitation closely. High rate of renal failure in this group of patients. RANSON CRITERIA: AT ADMISSION: 1.) Age > 55 yrs. 2.) Blood Glucose > 200 mg/dl (11 mm/l) 3.) SERUM LDH >300 4.) SEUM SGOT > 250 5.) WBC > 16,000 AFTER 24 HRS. 6.) HCT FALL >10 7.) BUN rise > 5 mg/dl (1.8 mm/l) 8.) Ca++ < 8 mg/dl (2 mm/l) 9.) Arterial p02 < 60 10.) Base Deficit >4 11.) Fluid Sequestration > 6L DIAGNOSIS OF ACUTE PANCREATITIS: 1.) BIOCHEMICAL MARKERS - AMYLASE - single most important lab test, but still has a significant false pos. and false negative rate. Though elevated amylase and lipase can be used to predict the presence of pancreatitis, they cannot predict the severity of the disease. 2.) RANSON CRITERIA - 4 OR MORE POSITVE SIGNS PREDICT COMPLICATED COURSE. RANSON CRITERIA: AT ADMISSION: 1.) Age > 55 yrs. 2.) Blood Glucose > 200 3.) SERUM LDH >300 4.) SEUM SGOT > 250 5.) WBC > 16,000 AFTER 24 HRS. 6.) HCT FALL >10 7.) BUN rise > 5 8.) Ca++ < 8 9.) Arterial p02 < 60 10.) Base Deficit >4 11.) Fluid Sequestration > 6L

3.) DYNAMIC CT SCAN - used to visualize areas of the pancreas which are nonperfused and likely to undergo necrosis subsequently. Early CT scan can predict the likelihood of subsequent complications. The amount of unperfused pancreas correlates with the number of positive Ranson's signs. Should be used early to identify areas of the pancreas that will necrose and guide surgery if it is necessary and to predict those who will have complications based on the degree of unperfused pancreas. Dye is given and then 5mm sections are taken at 5 sec intervals through the pancreas. Should be about 2/3 as bright as the aorta.

MANAGEMENT OF ACUTE INTERSTITAL PANCREATITIS: 1.) FLUID REPLACEMENT - ABCS. Use D51/2 NS and invasive monitoring -Swan Ganz and art line. Degree of fluid sequestration will give prediction of how severe the pancreatitis will be. 2.) HYPOXEMIA - ABCS. Check serial ABGs. 75% mortality for those who require ventilatory support. 3.) RENAL STATUS - urine output and serial BUN/CR. must be followed. 4.) MINIMIZING PANCREATIC SECRETIONS -NPO -Somatostatin - no effect on course of disease. -NGT - only with ileus for aspiration prevention, but prolonged NGT does not have beneficial effect vs pancreas for long term. - H2 blockers - no effect on disease. 5.) ANTIBIOTICS - should not be used in mild or moderate cases of pancreatitis, but Imipenim or Mezlocillin should be used when indicated for severe cases of pancreatitis. 6.) NUTRITION - no evidence that using fat in TPN will increase the pancreatic secretions and worsen pancreatitis disease course. Mainstay of therapy. - enteral nutrition can be difficult because of the ileus. If given, is best in the jejunum. 7.) HEMORRHAGE/ COAGULOPATHY - transfusion with red blood cells and FFP. 8.) METABOLIC/ELECTROLYTES - usually hypokalemia, hypocalcemia. Treat with replacement. TREATMENT - ALGORITHM, RIKKERS, p. 824. ** almost all of those with AIP will recover with conservative therapy in 2-3 weeks. Many of those with necrotizing pancreatitis will also recover without surgery, so initially all are treated with conservative management and surgery based on response. ALGORITHM - CT SCAN - if no necrosis, observe. If clinically worse, repeat CT scan. - if necrosis present but pt. stable, observe. - if necrosis present, questionable stability - percutaneous needle aspiration. If infected, operate. If sterile, observe. - if necrosis present and the patient not stable, operate.

TREATMENT EARLY (first 4 days) - lavage through a percutaneously placed catheter. 2 liters of fluid per run is used, and fluid is evacuated, removing the toxins. The effects of this treatment should be noted within several hours. MID PHASE (4 days to 2 weeks) - percutaneous aspiration/ drainage is for diagnosis only, not for treatment. The treatment of sterile necrosis is controversial, but if patient is unstable, should have operation. Procedure - midline incision to allow for placement of stomas if necessary. Obtain cultures from any free fluid. Access to the lesser sac is obtained through the transverse mesocolon in the avascular areas on either side of the middle colic artery. Bleeding vessels in the mesocolon should be ligated. Dissection through the gastrocolic omentum can be difficult and can be associated with injury to the mesocolon, stomach or colon. The necrosis is gently dissected away down to the viable pancreas. Strings of tissue are ligated as these represent vessels. BLEEDING FROM PORTAL AND SPLENIC VEIN - make initial effort to ligate bleeders. However, as tissue is friable, this may make bleeding worse. Pack with laparotomy sponges and close if necessary. - place several large drainage catheters that can be used for irrigation and drainage. Options include irrigation and drainage of the pancreas and packing of lesser sac with return to OR for repeat performance. LATE PHASE (> 2 WEEKS) - abscesses the most common cause of death, result from the infection of necrosis. Diagnose via CT scan. Percutaneous drainage - major draw back is the inability to evacuate thick, tenacious pus and break up all the loculations. Despite these, percutaneous route can be successful, though it may take several drains. Those no responding to percutaneous drainage should have surgery.

Scenario #27- Chronic Pancreatitis 45 y.o. male alcoholic presents with epigastric pain, new onset diabetes and diarrhea. Pain radiates to back and pt. presents for IV Demerol. Issues: a.) Initial workup of pain including labs, and radiographs. b.) What is the diagnosis if the plain X-rays show calcifications of the pancreas and stool that is positive for fecal fat. c.) Is ERCP indicated and why? What do you do if there is a ductal stricture in the head of the pancreas? d.) What do you do it there is s stricture in the mid body of the pancreas? e.) what do you do if there are multiple strictures in the pancreas? ANSWER: DIFFERENTIAL DIAGNOSIS: though it may seem straightforward, it is difficult to differentiate from cancer of the pancreas, which may also present with pain and weight loss. CT scan and ERCP may be needed but sometimes is not enough to differentiate.

WORKUP AND LABS/X-RAYS: Pancreatic Insufficiency - hallmarks are steathorrhea, diarrhea and azotorrhea. Do not occur until 90% of pancreatic function is gone. Can occur with gradual parenchymal loss or ductal obstruction from stricture or tumor. Diabetes - 70% of those with chronic pancreatitis will have glucose intolerance and 35% will have frank diabetes. LABS: OF MINIMAL VALUE. Most will have relatively normal LFTs and amylase. Nothing will differentiate chronic pancreatitis from CA. RADIOGRAPHIC EVALUATION: Plain X-Rays - speckled calcifications on plain x-ray seen in 30-50% of those with chronic pancreatitis. US - sensitive for detecting pseudocysts, dilated pancreatic duct and calcifications. CT scan - same as US, but more sensitive. Should be done for those being considered for surgery. ERCP - procedure of choice for chronic pancreatitis. 92% accuracy. Will delineate areas of stricture, dilatations, filling defects, stones, pseudocysts. If preop ERCP cannot be done, intraop pancreatography should be done. INDICATIONS FOR SURGERY: - PAIN - most common indication - COMMON BILE DUCT OBSTRUCTION - 10% will have persistent CBD obstruction after acute episode has subsided from fibrotic stricture of CBD. Stricture is long, tapered and incomplete as opposed to CA in which it is short, asymmetric and complete. If this is the only symptom, can be treated with bypass. - DUODENAL OBSTRUCTION - true diagnosis may not be made until surgery when biopsy can rule out CA. Can be treated with bypass./gastrojejunostomy. - COLON OBSTRUCTION - rule out CA with colonoscopy, perform colon resection. - VEIN THROMBOSIS - can cause left sided portal HTN with varices. Perform splenectomy. SURGERY FOR PAIN - GOALS: 1.) Relieve pain 2.) Preserve as much pancreas as possible. PEUSTOW PROCEDURE/DRAINAGE PROCEDURE Normal ductal anatomy = 4-5 mm in head, 3-4 mm in body and 2-3 mm in tail once ductal size reaches 7-8 mm or more, a Peustow can be performed with good chance of success. Closed suction drains are placed near the anastomosis to control leakage and fistula.

Generally has good results for pain. Recurrent pain is an indication for ERCP to visualize anastomosis. PANCREATIC RESECTION - indicated for treatment of patients with ducts of normal or decreased diameter. Resection is directed at the area of the gland that is most effected. Whipple - pylorus preserving for benign disease, for disease in the head. Distal Resection - for disease in the body or tail of pancreas Subtotal (80-95%) - diffuse disease with small ducts or for recurrent pain after smaller resections. WHIPPLE - for extensive disease of the head. Indicated in situations in which there is combined duodenal and CBD obstruction and when cancer cannot be ruled out. DISTAL RESECTION - for disease localized to the tail, for those with primary disease of the body and tail with ductal obstruction of the neck and for those with ductal stenosis after trauma. High incidence of postop diabetes.

Scenario # 28 - Subphrenic Abscess/Pancreatic Duct

Leak.

30 y.o. male with history of ITP S/P splenctomy now with fever and LUQ fluid collection, fever, abdo pain, nausea, LUQ pain, wbc=18,000. Issues: a.) without the LUQ fluid collection, what is the differential dx. o f a septic patient after a splenectomy. b.) Appropriate workup for the LUQ fluid collection?(labs, radiographic studies) c.) Is it appropriate to percutaneously drain this mass? (yes, to sample the amylase content of the fluid). Answer: Differential Diagnosis: 1.) Subphrenic Abscess 2.) Gastric Leak (necrosis at site of short gastric ligation) 3.) Pneumonia 4.) Empyema 5.) Pancreatic duct leak 6.) UTI 7.) OPSS 8.) Addisonian crisis Workup: Labs: -cbc - elevated wbc may indicate infection - platelet - need to evaluate platelet number before intervention - lytes - may be indication of adrenocortical insufficiency - amylase - pancreatic duct leak or pancreatitis of the tail of the pancreas may show elevated amylase in serum. - UA

- urine lytes - urine cultures - blood cultures - sputum cultures CXR - look for pneumonia, L pleural effusion, or haziness of the left hemidiaphragm. US - look for fluid in LUQ UGI with Gastrograffin - rule out leak of stomach cardia. CT scan - fluid in LUQ, inflammation of distal pancreas. Percutaneous drain via CT or US guidance - test for amylase content. If pancreaticocutaneous fistula, leave drain in and treat conservatively with TPN for 4- 6 weeks. If the fistula does not dry up, obtain ERCP to evaluate for stenosis of pancreatic duct. Surgery - persistent sepsis and failure of the percutaneous drain to adequate drain a (loculated) abscess located in the LUQ. Should be approached through a left lateral subcostal incision. Alternatively, abscesses in the left subphrenic space may be drained posteriorly through the bed of the twelfth rib. The abscess is generally deeper than thought. Make sure the area is cleared of infected material by palpating the aorta, esophagus at the esophageal hiatus, the caudate lobe of the liver and the anterior margin of the left lobe of the liver. Scenario #29 - Mass in Tail of Pancreas 74 y.o. male with mass in tail of pancreas. Issues: a.) Differential diagnosis and initial workup. b.) Describe resection in detail.

Colon II
Scenario #30 - Colon CA 70 y.o. make presents with complaint of occasional blood in his stool. issues: a.) Differential diagnosis and initial workup. b.) Assuming a mid-sigmoid CA is found, what type of operation must be performed (include vessels taken, type of anastomosis and margins of resection). c.) Transect L ureter 5 cm from ureteropelvic junction, what do you do? d.) Palpate a nonvisible 3 cm mass on the left lateral segment of the liver. What do you do? What do you do if the mass is deep medial segment of the left lobe of the liver? e.) What are recommendations for the patient with a CII lesion but no mets? BII lesion? etc? f.) What is the appropriate preop preparation?

Answer: Differential Diagnosis: 1.) Colorectal CA/ Polyp/Neoplasia 2.) Hemorrhoids 3.) Rectal trauma. 4.) Diverticulitis 5.) Meckels Diverticulum 6.) Angiodysplasia 7.) Inflammatory Bowel Disease 8.) Vasculitis 9.) Colonic or rectal varices from portal hypertension 10.) Ischemic colitis 11.) Amebiasis - diarrhea 12.) Clostridium Difficile - diarrhea WORKUP: AGE/COMORBID FACTORS - careful history and physical exam as well as preop EKG and PFTs if indicated. If EKG or history of chest pain/SOB or recent MI, start with stress thallium testing. LABS: cbc - present hematocrit, anemia may point toward prolonged occult hemorrhage. wbc - if infectious etiology such as diverticulitis, amebiasis or C. Diff. LFTs, PT/PTT, platelets - if history of alcoholism or liver failure are possible cause of varices. Abnormal testing is not predictive of met. disease to liver. Stool C+S - if diarrhea and infectious etiology possible C. Diff - if diarrhea and C. diff. possible. CEA - if colon or rectal CA suspected. may be elevated in diverticulitis, chronic or active hepatitis, pancreatitis or renal failure. BaE Colonoscopy - indicated for preop visualization of lesion and biopsy. Once histologic diagnosis has been made, the remainder of the colon should be surveyed for synchronous CA (5%) or synchronous adenomatous polyps. Either BaE or colonoscopy is acceptable, but colonoscopy has the benefit of allowing biopsy of other lesions to rule out synchronous CA and perform polypectomy for polyps outside the desired resection area. CT scan - colon CA - not indicated as surgery will be performed for palliation in event of Metastatic disease. - Rectal CA - indicated if there is a question of resectability or involvement of contiguous structures such as the bladder, prostate and bony pelvis. STAGING - DUKES CLASSIFICATION Stage Description A Limited to mucosa B1 Limited to muscularis propria, nodes negative

B2 Through muscularis propria, nodes negative B3 Through bowel wall, contiguous organ involvement, nodes negative C1 Limited to muscularis propria, nodes positive C2 Through muscularis propria, nodes positive C3 Adjacent organ involvement, nodes positive D Mets Preop staging - much more important for rectal CA than colon CA. SURGICAL THERAPY AND CONSIDERATIONS. ANATOMY Cecum, Ascending Colon, Proximal Transverse Colon - midgut Arterial Supply - SMA-> ileocolic, right, middle colic. Venous drainage - SMV - > drains into portal vein. Distal transverse colon, descending colon, sigmoid - hindgut Arterial Supply - IMA-> middle, left colic, sup. hemm. Venous drainage - IMV -> drains into splenic/portal vein Rectum Arterial Supply - middle rectal arteries (hypogastric br.) - lower rectal arteries (pudendal br.) Venous drainage - middle + inferior rectal veins - > IVC. Lymphatic Drainage - parallels vascular anatomy with 4 levels of drainage - > epicolic, paracolic, intermediate and principal nodes --> cysterna chyli. ONCOLOGIC PRINCIPLES OF COLON RESECTION: RIGHT HEMICOLECTOMY - for lesions of the cecum and ascending colon. Ligate the ileocolic and right colic arteries. Divide the bowel at the distal ileum and at the midtransverse colon. EXTENDED RIGHT HEMICOLECTOMY - for lesions of the hepatic flexure or proximal transverse colon. Ligate the ileocolic and right and distal left colic/marginal artery of Drummond. Divide the bowel at the distal ileum and at the proximal/ mid-descending colon. LEFT HEMICOLECTOMY - for lesions of descending colon. Ligate the marginal artery beneath the distal transverse colon and left colic artery. Divide the bowel at the mid/distal transverse colon and at the proximal sigmoid colon. EXTENDED LEFT HEMICOLECTOMY - for lesions of the distal transverse colon and the splenic flexure. Ligate the marginal artery beneath the mid/distal transverse colon and the left colic, sigmoid and upper rectal branches. Divide the bowel at the mid/distal transverse colon and at the distal sigmoid/upper rectum. TRANSVERSE COLECTOMY - for lesions of the mid transverse colon. Ligate the marginal artery at the hepatic flexure, distal middle colic and the marginal artery at the splenic flexure. Divide the bowel at the hepatic and splenic flexure.

SIGMOID COLECTOMY - Ligate the proximal branches of the left colic artery (left colic, sigmoid and upper rectal branches). Divide the bowel at the distal descending colon and at the upper rectum and mobilize the splenic flexure as needed for a tension free anastomosis. No touch - no data. Intraluminal spread - no data. SPECIAL CIRCUMSTANCES: 1.) SYNCHRONOUS CA OR POLYPS - should be considered for subtotal colectomy depending on the distance between lesions and the number of lesions. If patient has multiple polyps or CAs should have subtotal colectomy to facilitate surveillance of remaining colon. 2.) OBSTRUCTING CANCERS: OBSTRUCTIING RIGHT COLON CANCERS - can have right hemicolectomy and ileocolic anastomosis. OBSTRUCTING LEFT COLON CANCER - OPTIONS: - if tumor can be intubated endoscopically, a wire can be passed through the lumen and a tube be passed over the wire into the proximal obstructed bowel and allow the bowel to be decompressed and mechanically prepped for surgery. This should be used for a partial obstruction or near complete obstruction to ensure adequate mechanical preparation of the colon for safe resections and anastomosis. - resection with primary anastomosis and colonic cleanout with or without protective colostomy. - primary resection and colostomy with Hartmans pouch for second stage anastomosis. - subtotal colectomy with primary anastomosis. - decompressive left colon loop colostomy followed by formal resection of tumor and primary anastomosis. 3.) ADJACENT ORGAN INVOLVEMENT - appears to be a variant of colon CA which grows to large size and is locally invasive that is less aggressive from a metastatic potential point of view. Because of this, these lesions are potentially curable even when they involve adjacent structures. LIVER METS - those which can be resected with clear margins by wedge resection should be resected and those lesions that will require a formal hepatic resection should be postponed because of the magnitude of the operation. Also allows use of intraop ultrasound. 4.) OOPHORECTOMY - indicated for mets to the ovary and for direct extension of the tumor to the ovary. Can also offer prophylactic oophorectomy for postmenopausal women. ONCOLOGIC PRINCIPLES OF RECTAL RESECTION: APR - standard of therapy for rectal CA. En bloc resection of rectum, mesorectum, perineal skin and sphincters. Downside is the permanent ostomy and the bladder and sexual dysfunction. LAR - 5 cm distal margin rule has proved wrong since tumor rarely spread further than 2 cm. This has allowed for some distal mid third and proximal lower third tumors to be resected by LAR as opposed to APR.

COLOANAL ANASTOMOSIS - can be use for very low, invasive rectal cancers (a very low LAR which utilizes a colonic J pouch for improved continence). Often recommended a protective colostomy. LOCAL EXCISION - GOAL IS TO REMOVE FULL THICKNESS DISK OF RECTUM WITH 1-2 CM MARGIN OF NORMAL RECTAL TISSUE BEYOND THE TUMOR. Anterior Wall Lesions - prone or jacknife prone Posterior Wall Lesions - high lithotomy position. Advantage over laser/thermal ablation or radiation- entire specimen is available for histologic analysis. Criteria for local excision: 1.) distance of less than 8 cm from anal verge. 2.) Size of less than 3-4 cm. 3.) mobile, nonulcerated, noncircumferential 4.) well to moderately differentiated 5.) T1 or T2 lesions 6.) Diploid by flow cytometry. ADJUVANT THERAPY RADIOTHERAPY COLON - indicated for tumors that extend through wall posteriorly or laterally. RECTUM - NEOADJUVANT - should be sued for tumors that are fixed or unresectable at the time of evaluation for surgery. Can convert marginally resectable to resectable lesions and can permit sphincter sparing operations for tumors that would have required APR. - ADJUVANT - since pelvic recurrence of CA is a catastrophic event, therapy which can decrease local recurrence is desirable - and radiotherapy has been shown to decrease chances of local recurrence. Used for B2 and B3 (through muscularis propria or through wall) and for C2 and C3 (through muscularis propria or through muscle wall). CHEMOTHERAPY COLORECTAL - should be offered 5-FU + levamisole based chemotherapy in patients with Dukes stage C colorectal cancers based on improved survival in these patients vs. controls. FOLLOWUP - not only for recurrences but also for metachronous polyps and cancers. COLONOSCOPYPreop perforation or obstruction- soon after surgery to evaluate the rest of the colon. REST - CXR and colonoscopy at 1 year. If negative, every 2-3 years. If polyps present, remove and repeat annually until negative. CEA - at 6 weeks and then every 3 months for 2 years.

Scenario #32 - Rectal Villous Adenoma with High Grade

dysplasia.

45 y.o. male who is found on screening sigmoidoscopy to have a villous adenoma with high grade dysplasia 5 cm in size 6 cm from the dentate line. Issues: a.) Appropriate workup of patient with villous adenoma? b.) How would you remove this lesion if the remainder of colon is evaluated as normal. c.) How often should patient be followed for this lesion? How often should you follow up a patient who has had colon CA? ANSWER - LAR OR PIECEMEAL COLONOSCOPIC POLYPECTOMY WITH FOLLOW UP COLONOSCOPY AND BIOPSY IN 2-3 MO.

Scenario #33 - Cecal Adenoma S/P Sigmoid Colectomy 50 y.o. S/P sigmoid colectomy for Dukes BII CA. On F/U colonoscopy found to have cecal tubulovillous adenoma without evidence of dysplasia. Issues: a.) What sort of resection should be entertained in this patient? b.) Follow Up? ANSWER - ENDOSCOPIC POLYPECTOMY IF POSSIBLE. IF NOT, CELIOTOMY AND LIMITED RESECTION. NEOPLASTIC COLORECTAL POLYPS CARCINOMA IN SITU = SEVERE DYSPLASIA= ADENOMA WITH ATYPIA = FOCAL CARCINOMA = INTRAMUCOSAL CARCINOMA. IN SITU - DO NOT METASTASIZE INVASIVE - DO METASTASIZE POLYPS **the larger the polyp and the wider the base (sessile>>pedunculated), the greater the chance of malignant potential. - small polyps (< 1 cm) dont present with malignancy often, but can present with bleeding. -CANCER CAN OCCUR IN ANY POLYP, BUT MOST OFTEN IN VILLOUS. TUBULAR ADENOMA - more frequent than villous and increase in age. Tend to be multiple in contrast to malignant polyps which are more often singular. May occasionally present with bleeding, but generally are asymptomatic and are found on contrast studies of the colon or colonoscopy. VILLOUS ADENOMA - like cancerous polyps, tend to be single as opposed to multiple. Have a greater propensity to harbor an in situ or invasive carcinoma. In one series, over half of villous adenomas over 2 cm were histologically malignant. Represent almost 10 fold increase in frequency of CA over tubular adenomas less than 1 cm in diameter.

- because of large surface area may initiate bleeding, cause obstruction, cramping and obstipation. May occasionally produce a diffuse watery diarrhea that can lead to hypokalemia. - TEND TO OCCUR IN THE CECUM, SIGMOID AND RECTUM. - invasive malignancy occurs in 20-30% for polyps > 2cm diameter. TREATMENT OF COLORECTAL POLYPS - simple polypectomy is considered appropriate therapy for tubular or villous adenomas that contain foci of in situ carcinomas (absence of penetration of the muscularis mucosa and invasion of the submucosa). SHOULD BE COMPLETELY REMOVED AT COLONOSCOPY OR SIGMOIDOSCOPY. DO NOT METASTASIZE. - if invasion of CA/malignant cells is deep to the muscularis mucosa, these patients should be treated as invasive carcinoma of the colon. The risk of invasion deep to the muscularis mucosa is greater with sessile than pedunculated polyps. Because of chance of incompletely excising tumor endoscopically and not treating potential lymph node mets, the failure rate of treating sessile polyps with invasive CA by endoscopic polypectomy is 21%. With pedunculated polyps, the risk of lymph node mets is lower and the risk of incomplete excision is lower, but the rate of failure is still 10%. For pedunculated polyps in which the malignancy has reached the neck of the stalk, a standard resection should be performed. If CA is limited to the head of a pedunculated polyp, risk of mets is only 3%. VILLOUS ADENOMAS - SHOULD BE COMPLETELY EXCISED BY COLONOSCOPY OR TRANSANAL EXCISION IF POSSIBLE. - often these tumors are large and pedunculated and cannot safely be excised via a colonoscopic polypectomy. If the polyp has been incompletely excised or biopsied, and lie above the peritoneal reflection, a celiotomy and excision of the entire lesion (segmental resection) should be performed. - tubulovillous adenomas lie between tubular and villous in malignant potential. TREATMENT OF POLYPS: TUBULAR DYSPLASIA/CARCINOMA IN SITU - COLONOSCOPIC/OPEN COMPLETE SIMPLE POLYPECTOMY TUBULAR PEDUNCULATED WITH INVASIVE CA AT HEAD COLONOSCOPIC/OPEN COMPLETE SIMPLE POLYPECTOMY IF: 1.) CA is not highly undifferentiated. 2.) blood vessels or lymphatics do not contain CA. 3.) stalk is long enough to allow histologic evaluation and malignancy does not extend to neck of adenoma.

4.) CA does not extend to the base of the stalk 5.) entire polyp is not CA. TUBULAR SESSILE WITH INVASIVE CA - RESECTION OF COLON+ MESENTERY VILLOUS - all villous adenomas should be removed. DYSPLASIA/CARCINOMA IN SITU - COLONOSCOPIC/OPEN COMPLETE SIMPLE POLYPECTOMY VILLOUS PEDUNCULATED WITH INVASIVE CA AT HEAD COLONOSCOPIC/OPEN COMPLETE SIMPLE POLYPECTOMY IF: 1.) CA is not highly undifferentiated. 2.) blood vessels or lymphatics do not contain CA. 3.) stalk is long enough to allow histologic evaluation and malignancy does not extend to neck of adenoma. 4.) CA does not extend to the base of the stalk 5.) entire polyp is not CA. VILLOUS SESSILE WITH INVASIVE CA - RESECTION OF COLON AND MESENTERY

KEY - biopsy or partial polypectomy does not reliably assess the presence or absence of malignancy, as well as the extent of the spread. Need the whole polyp to rule in or out CA, and to tell the level of invasion if CA. Pathologic diagnosis : invasion to muscularis mucosa, predominant adenoma type, presence of lymphatic or vascular invasion, adequacy of margin of resection. FOLLOW UP - EVERY 2 YEARS IF MORE THAN ONE ADENOMA REMOVED - EVERY 3 YEARS IF THE PATIENT HAS HAD ONE INDEX LESION. SUBTOTAL COLECTOMY - aggressive approach may be indicated in high risk patients with previous complete endoscopic polypectomy for CA and found to have villous or tubulovillous adenoma.

Scenario # 34 - Low Rectal CA 80 y.o. male with rectal mass at 3 cm from anal verge, about 3 cm in diameter. Bx is positive for adenoCA. Issues: a.) How else would you evaluate the colon? b.) Describe the technique of a transanal excision of a mass? What are options for the anesthetic?

c.) What if further workup revealed CAD, angioplasty performed - would this change your mind concerning options for treatment? What if the patient had had an MI 2 months ago? What if the pt. had had a recent MI and now has a near obstructing rectal CA? Role for preop radiation to shrink tumor? d.) APR/LAR performed and positive nodes found, what chemo should be offered? e.) LAR is performed for 2x4cm mass at 5 cm, upon stapling the anastomosis, an incomplete ring is found and posterior defect is confirmed by insuffaltion - what do you do? Do you place a colostomy? LOCAL TREATMENT FOR DISTAL/LOW RECTAL CA. RATIONALE FOR LOCAL TREATMENT OF RECTAL CA 1.) Local treatment is confined to that group of patients with CA limited to rectal and perirectal tissues - it is this group of patients that will not benefit from a more radical resection. Since APR only benefits some of those with early Dukes C disease, if all patients are treated with this therapy, only 10-15% will benefit from resection over local treatment. 2.) Local treatment is also beneficial for those who are symptomatic from a rectal tumor but are known to have distant disease and are candidates for palliation but not cure. When symptoms can be eradicated by local treatment an d the potential morbidity, mortality and inconvenience of a more extensive procedure is avoided. However, local treatment for disease that has spread to the nodes is inadequate therapy. 3.) Local treatment also avoids unnecessary surgery in patients with significant comorbid disease, who might die from general anesthesia or celiotomy. PROBLEM - in patient without distant disease, how do you select patients with Dukes A or B disease who would benefit from local treatment prior to surgery. ANSWER - accurate preop staging. Problem is staging for presence of nodal disease. Methods - digital rectal exam. - CT scan is no better than digital rectal exam - Endorectal US - negative predictive value (ability to predict tumor that are not through wall or into nodes). 70-95% for penetration of wall (vs. 40-60% for CT). 80-90% for nodes (50-70% for CT). LOCAL TREATMENT OPTIONS: LOCAL EXCISION: should be limited to tumors that are 3 cm or less in diameter. - should not be used for poorly differentiated histology. - should not be used for tumors which are more than 1/3 circumference.

Technique - full thickness excision of rectal wall including 1-2 cm beyond the tumor. to minimize bleeding, should use cautery for excision. Proctoscopic examination should be performed at the end of the procedure to assume the lumen has not been compromised. Results - poor prognostic signs include positive margins, poorly differentiated histology and depth of invasion. INTRACAVITARY RADIATION: Limited tolerance of tissues for external beam radiation does not make this viable for curative treatment of rectal CA. Use a short probe to deliver high intensity dose of radiation with limited tissue penetration. Criteria for treatment - well differentiated or moderately well differentiated. - limited to bowel wall on preclinical assessment. - maximal diameter of 4.5 cm. - ulcerated lesions within 9 cm of verge, polypoid lesions within 12 cm of verge. Results - local control of the tumor with 92.5% of patients. ELECTROCAUTERY -Criteria - similar for histology, circumference and size for transmural excision. - should be limited to tumors at or below 10 cm. to avoid peritoneal soiling for transmural cauterization. Tumors below 10 cm can be cauterized through he rectal wall and attempts should not be made to close a transmural defect or protect it with a colostomy. Results - 5 yr. survival rates of 50-65% when treated with cautery alone. Bleeding is a common complication (7%). LOCAL RECURRENCE can occur in 6-44% of patients receiving local treatment for low rectal CA. Though a second trial of local treatment can be tried for recurrence, LAR or APR are the option most chose. Salvage rate is 21-87% for these procedures. ADJUVANT THERAPY - promising for those who receive adjuvant radiotherapy after either local excision or electrocautery. ROLE OF PREOP AND POST OP RADIOTHERAPY PREOPERATIVE THERAPY - studies show that pelvic recurrence rates are lower than those treated with surgery alone. There is still questions as to whether there is improved 5 year survival with this treatment. Should be focused on T3 and T3N1 lesions as well as T4(circumferential, near obstructing) lesions which will have the highest rate of locoregional failure and the best chance of shrinkage. SELECTION - rectal ultrasound is best for determining depth of tumor invasion but not quite as good for detecting local nodes. T3 lesions (full thickness of wall) and T4 lesions (involving adjacent structures), near obstructing lesions and circumferential lesions should be selected for preoperative radiotherapy. POSTOPERATIVE RADIOTHERAPY- incidence of locoregional failure is reduced when radiotherapy is used with surgery as compared to surgery alone. Recommended for T3N0 lesions (through submucosa into subserosa) or for any TN1lesion. When accompanied with 5-FU and levamisole lower locoregional recurrence to 10-14%.

FOLLOW - UP AND POSTOPERATIVE SURVEILLANCE RATIONALE - goal is to find tumor when it is still resectable as most tumor recurrences are found when unresectable leading to patients death. - almost all recurrences occur in the first 5 years after resection

Scenario -#31 - H/O Colon CA with Rising CEA. 50 y.o. male is S/P sigmoid resection for BII CA. Found on routine F/U screening examination to have rising CEA. Issues: a.) What is your initial workup? b.) What do you do if metastatic workup is negative and patient is found to have a cecal CA on colonoscopy? c.) What do you do if the metastatic workup is positive for liver and lung mets and there is a cecal CA on colonoscopy? d.) What do you tell the patient if the metastatic workup is negative but the path comes back positive for lymph nodes after a total colectomy and ileo-proctostomy is performed? e.) What do you tell the patient regarding the frequency of bowel movements? ANSWER: CAUSES OF ELEVATED CEA: cigarette smoking, emphysema, bronchitis, peptic ulcer disease, gastritis, hepatitis, obstructive jaundice, pancreatitis, diverticulitis, renal failure. - because of the number of causes of elevated CEA there is no role for using CEA as a screening test for large populations. Cannot distinguish between benign, noncolorectal cancer and colorectal conditions. PROGNOSTIC IMPLICATIONS - 25-75% of patients with colon cancer will have elevated CEA. There is a positive correlation of the incidence of positivity and the magnitude of the elevation of CEA with advancing Dukes staging. - CEA levels should normalize 4-6 weeks after resection. Persistent levels implicate an incomplete resection and ominous prognosis. In order for CEA level to have use as a prognostic factor, it must return to normal after surgery. WHEN TO MEASURE: 1.) PREOP 2.) POSTOP - 4-6 WEEKS 3.) EVERY 3 MONTHS FOR 2 YEARS 4.) EVERY 6 MONTHS FOR NEXT 3 YEARS. **vast majority of recurrences will recur within 5 years. Most will recur in 2 years.

STANDARD FOLLOWUP : 1.) CXR, LFTS - EVERY 6 MONTHS 2.) FOLLOW- UP VISITS - history, abdominal exam, hemoccult, CEA. 3.) COLONOSCOPY - ANNUALLY FOR AT LEAST 3 YEARS ?ABNORMALITIES - hemoccult positive - colonoscopy - CXR nodule- chest CT - LFTs - abdominal CT. - elevated CEA upper limit of normal - 2.5 in nonsmokers - consider recurrence until - 5.0 in smokers - proven otherwise. >10 - recurrent colorectal CA. CASE #1 - ASYMPOTMATIC WITH ELEVATED CEA first - CT scan of chest, abdomen and pelvis - colonocopy - bone scan second - if colonsocopy positive and others negative. - laparotomy - if bone scan positive or CT shows distant spread - ?palliation - if none of the above positive - second look laparotomy CASE #2 - SYMPTOMATIC PATIENT WITH ELEVATED CEA first - stage disease with CT scan of chest, abdomen and pelvis, bone scan and colonoscopy. second - based on symptoms and preop staging, options are palliation via surgery, radiation, chemotherapy and any combination of these. - other options are re-resection for cure with or without the addition of chemotherapy/radiotherapy. EXPLORATORY LAP FOR ELEVATED CEA - midline incision. Inspect visceral and parietal peritoneum for tumor implants. Bimanually palpate the liver. Use intra-op ultrasound to more thoroughly examine the liver. Examine the retroperitoneum in clockwise fashion from the GE junction. Run the GI tract. - can have patients undergo preop testing with radiolabeled murine monoclonal antibody after pretest sensitivity to Ab. Allows localization of recurrent tumor with exception of liver which takes up radioactivity nonspecifically. Also allows scanning intraop with gamma probe, and biopsy of areas with increased counts. COLONIC RECURRENCE - requires resection of anastomosis with primary anastomosis. Should take 5 cm margin (ideally 10 cm on either side) to prevent further recurrence. Wide resection of mesocolon should be taken to prevent further recurrence. Any adjacent structures are taken en bloc. LIVER METASTASIS - wedge resection can be performed if possible over lobectomy, bilaterally if necessary. Minimum of 1-2 cm of normal tissue as margin. Prior to proceeding with lobectomy, should make certain that there are no implants in pelvis, omentum or peritoneum.

PELVIC RECURRENCE - if pain is present as symptom the tumor is locally advanced and usually incurable. If tumor extend s into sacrum or bladder, en bloc resection is performed. If recurrence has positive margins by gross or microscopic criteria, brachytherapy needles are inserted. Scenario #35 - Simultaneous Colon and Liver Masses. 70 y.o. male with sigmoid CA undergoing resection; noted intraoperatively to have a 4 cm mass in the left lobe of the liver? Issues: a.) How would you evaluate the liver lesion? Would you perform a biopsy? b.) If the biopsy came back positive for CA, how would you proceed? c.) Explain the rationale for treating colorectal mets to the liver including indications, contraindications and technique. d.) Discuss the anatomic limits, technique and vascular anatomy for major liver resection. ANSWER: - of all patients with disseminated colorectal cancer, 15-20% will have lesions which are localized to the liver, and a smaller percentage will be surgical candidates. Extremely highly selected group of patients. Rationale for treatment = 2 yr. survival of patient with unresected mets is 45%. 5 yr. survival is zero. Those that do not undergo resection will die of disease. SITUATION #1 - LIVER METS DISCOVERED AT TIME OF COLON RESECTION. MUST EVALUATE; size and location of mets, status of portal and celiac nodes, presence or absence of extrahepatic disease. IF RESECTABLE - resect the colon/rectal primary and then decide whether to resect with this operation or wait. CRITERIA FOR SIMULTANEOUS RESECTION OF LIVER TUMOR WITH COLON RESECTION: 1.) single met which can be removed by wedge resection. 2.) colon/rectal resection has gone flawlessly. 3.) adequate incision. 4.) surgeon comfortable with resection under circumstances. SITUATION #2 - LIVER METS DISCOVERED ON FOLLOWUP; 3 GROUPS -

1.) LIVER METS + PORTAL/CELIAC INVOLVEMENT mets in celiac and portal nodes are result of spread from liver. Dismal 5 yr. survival. Do not resect. 2.) LIVER METS + EXTRAHEPATIC DISEASE - 2 situations in which extrahepatic disease does not rule out liver resection - simultaneous lung mets and anastomostic recurrence. In the case of anastomotic recurrence, re-resect colon/rectal tumor and wait 4-6 weeks for liver resection. If other extrahepatic disease is found, do not resect. 3.) LIVER ONLY DISEASE - DO NOT RESECT MORE THAN 4 METS. The survival after resection decreases with increasing number of mets. The upper number of mets is arbitrary. In resecting mass, obtain a 1-2 cm margin of normal tissue if possible. Inability to obtain a 1 cm margin does not contraindicate resection. EVALUATION OF LIVER MASS - either synchronous mass that will be resected at a later date or a metachronous mass(recurrence) found on follow-up needs to have workup for extent of metastatic disease. Sites of Recurrence - anastomotic recurrence, liver and lungs Tests for recurrence colonoscopy, BaE, CEA, abdo/pelvic CT, LFTs, chest CT. Colonoscopy/BaE- evaluate the anastomotic site as well as the remainder of colon for a metachronous primary. Abdo/pelvic CT - should be used to evaluate the number and location of liver mets as well as portal and celiac nodes and the presence of extrahepatic disease within the abdomen. LFTS - elevations of alk phos and bili may indicate more mets than seen on CT or presence f cirrhosis. Unusual for mets and cirrhosis to present at the same time- can remove 75% of normal functioning liver. Chest nodule - does not rule out liver resection. May either be a benign granuloma or met, and one or two colorectal mets to the lung can be resected either at the same time or in a staged manner. BIOPSY OF LIVER MASS - INTRAOP AT TIME OF COLON RESECTION --> YES - METACHRONOUS LESION ---> N O. (positive biopsy does not change plan and negative biopsy is seen as inadequate biopsy) Preop Prep - usual bowel prep - if anastomotic recurrence is found during exploration of for liver resection, the anastomotic recurrence is resected and the liver resection is delayed. SURGICAL RESECTION INCISION - plan for a bilateral subcostal, but start with a right subcostal incision for exploration. EXPLORATION - explore the abdomen for extrahepatic disease. If none present, proceed with taking down of the falciform ligament for intraop ultrasound. INTRAOP ULTRASOUND - will change operative plan 20% of time, either from finding additional mets or through defining the relationship of the mass to the hepatic veins. RESECTION - if resectable, extend incision to bilateral subcostal

TECHNICAL DESCRIPTION = RIGHT HEPATIC LOBECTOMY 1.) INCISION = bilateral subcostal with midline extension for full mobilization. 2.) EXPOSURE = use Omni Retractor for retraction of costal margins. 3.) MOBILIZATION OF RIGHT LOBE - right triangular ligament is taken down and the falciform ligament is incised, and bare area of the liver is mobilized off the diaphragm. The right branch of the hepatic artery is found and dissected out behind the CBD and isolated. The right branch of the portal vein is isolated (usually on right side of CBD). For vascular tumors, these should be clamped now. The liver is retracted to the left and anteriorly by assistant on left (help the doctor) to expose the IVC. The IVC is cleared off to the level of the right hepatic vein. The right adrenal vein is identified, ligated and divided. Small veins from the right lobe to the IVC are ligated. 4.) DISSECTION - if the hepatic artery and portal vein branches have not been clamped, do it now to demarcate the right liver. Incise the liver peritoneum with cautery, divide parenchyma by finger dissection, clamping and dividing vascular structures (vascular structures with Prolene and biliary structures with vicryl). Dividing the major vessels within the liver prevents inadvertent ligation of aberrant vascular anatomy. Finally, the right hepatic vein is divided and ligated. TECHNICAL DESCRIPTION = LEFT HEPATIC LOBECTOMY 1.) INCISION = bilateral subcostal with midline extension for full mobilization. 2.) EXPOSRE = use Omni retractor for retraction of costal margins. 3.) MOBILIZATION OF LEFT LOBE - take down the left triangular ligament by opening the gastrohepatic ligament and mobilizing the left triangular ligament to the IVC. Left phrenic vein is identified and divided. (Careful for the left French nerve which lies across the vein to be divided.) This mobilization will leave you at the IVC/left hepatic vein junction. The falciform ligament is divided to the IVC(done already for intraop US). Retract the left lobe to the right (help the doctor). The IVC is seen by retracting the caudate lobe anteriorly and to the right. The adventitia is opened up over the IVC, opening up the posterior suprahepatic cava for full mobilization of the IVC. 4.) HILAR VASCULAR DISSECTION - hilar dissection is done by identifying the bifurcation of the hepatic artery, portal vein and bile duct. Left portal vein is divided by clamping and overseeing using a running proline. The left bile duct and left hepatic artery are divided. The liver substance is divided from the gallbladder fossa toward the junction of the middle hepatic vein/IVC.

Scenario #36 - Lower GI Bleed 70 y.o male in ER who 2 hrs earlier had passed 100 cc of bright red blood per rectum. a.) ABCs including type and cross. b.) Initial evaluation and exam (including flex sigmoidoscopy). NGT?

c.) Appropriate workup for localization of a lower GI bleed (know rates of bleeding for detection by arteriogram and bleeding scan). When is colonoscopy indicated? Are there any blood tests that may be helpful? d.) What are the surgical options if the bleeding source cannot be localized? ANSWER: A= if airway unstable because of hypotension, intubate. B = in absence of unstable airway, place on 02 4-6L by facemask to increase 02 carrying capacity. C= peripheral IVs - 16 gauge or larger. Resuscitate with LR. Place Foley catheter and titrate fluid for a UO of 30cc/hr. Place art line. Admit to ICU. LABS: cbc - Hct and platelet level. Cirrhotic may be thrombocytopenic. Coags - PT may be elevated in patient with cirrhosis. LFTs - may indicate chronic liver disease and cirrhosis. T&C X 4U prbcs. Differential Diagnosis of Massive Hematochezia Cause FREQUENCY Angiodysplasia 30% Diverticulosis 17% Neoplasm 11% Colitis, ulcers 9% Small bowel lesions 9% Mixed Upper GI source 11% EVALUATION OF LOWER GI BLEEDING NGT - to rule out 11% chance f UGI bleed. If positive, proceed to upper endoscopy. If negative, leave in case needed for colonscopy. ANOSCOPY/SIGMOIDOSCOPY - should be performed immediately after digital rectal exam to rule out hemorrhoids, anal fissure, rectal ulcers, trauma, or low rectal CA. Anal lesions can be controlled with local lidocaine/epi. Hemorrhoids can be banded. Decision made to go to OR for bleeding low rectal CA. If site not located and patient hemodynamically stable: DIAGNOSTIC/THERAPEUTIC COLONOSCOPY - problem - unprepared colon will not yield a good exam.. Go-Lytely through NGT at a rate of 1L/hr until bowel effluent clear. Sedate. If bleeding is located and cannot be cauterized, use flouro to localize the lesion. Examine the terminal ileum for evidence of blood or Crohns disease. If hemodynamically unstable: after anoscopy, proceed to upper endoscopy quickly to rule out upper GI source. If the upper endoscopy is negative, proceed to mesenteric angio. MESENTERIC ANGIO: for bleeding at 1cc/min. locates lesion, can be therapeutic and directs surgical intervention. 75% of all lower GI bleeding can permanently or temporary controlled angiographically. Particularly useful in patient with massive bleeding, no known site and cant prep the colon for colonoscopy.

? Embolize - NO. There is not the rich collateral circulation in the lower tract that is present in the upper tract, so embolization likely to cause ischemia. IF bleeding slower and more intermittent : SCINTIGRAPHY - for bleeding of 0.1 cc/min. Accuracy depends on rate of bleeding and the transit of blood through the small bowel/colon. As the sensitivity of the bleeding increases, the localization decreases. Performing surgery solely based on a positive scintigraphy has undesired results in 40%. SURGERY -- for 6%, no lesion can be localized. Position in the Sims position to assist with intraop colonoscopy. Midline incision. Palpate the stomach, duodenum, small bowel and large bowel to determine the proximal extent of the bleeding. Bleeding in the stomach or duodenum should prompt intraop upper endoscopy. - if massive colonoscopic bleeding, place colotomy in cecum via the appendix and lavage colon. Administer at 0.3L/min. Place large rectal tube in to catch effluent. Once colon is clean, advance scope into rectum and allow surgeon to guide the scope through the colon. If no site is found, blind resection or ostomy for trans-stomal endoscopy. Blind resection should be based on the localization of bleeding to the right or left side. This has better results and less morbidity than subtotal colectomy. Subtotal colectomy is done as last option.

Scenario #37 - Acute Diverticulitis with Phlegmon/Abscess 60 y.o. female presents to the emergency room with the complaints of LLQ tenderness and fever. Issues: a.) What is the differential diagnosis and the initial workup? b.) Assuming the CT scan shows thickening of the sigmoid with phlegmon, what is you initial treatment. If there is no air fluid level or fluid collections; what if there is a fluid collection? What would you do if a CT scanner was not available? c.) Assuming the patient is not better in 24 hrs., what do you do? What about the possibility of Crohns disease in this patient? d.) Assuming that the patient defervesces and improves, what would be the length of treatment and discharge advice to her? What would you tell the patient regarding the possibility of a colostomy? e.) Assuming she does not get better, what would your operative treatment be -i.e. - how would you decide how much colon to remove? Describe in detail how you would remove the colon including identification of important anatomic structures? f.) Upon removal of colon, there is a rush of urine - how would you evaluate this? (ureter and bladder) g.) What would you do if the sigmoid colon is densely adherent to the uterus? Are sections necessary for CA?

ANSWER: DDX OF LEFT LOWER QUADRANT PAIN AND FEVER: 1.) Diverticulitis 2.) Crohns Disease of the colon. 3.) Perforated Colon Cancer. 4.) Perforated DU with LLQ abscess. 5.) Pyelonephritis. 6.) Mesenteric Infarction 7.) Ischemic Colitis 8.) Stercal Ulcer 9.) Tuboovarian Abscess (unlikely here) 10.) Foreign body perforation 11.) Pseudomembranous colitis 12.) Lymphoma. WORKUP: CBC- assess for leukocytosis and/or anemia. Bun/Creat.- evaluate renal status and guide selection and dosing of antibiotics. LFTs - evaluate transaminases, alk phos and bili. for survey for liver complications of diverticulitis. UA - look for UTI or evidence of fecaluria. Radiological Tests: Upright chest film - look for free air from perforation. Abdo flat and upright - assess mechanical obstruction or ileus. CT Scan - question is whether to pursue a CT scan right away or give a presumptive diagnosis of diverticulitis and start medical therapy and get CT scan when it appears that medical therapy is not succeeding (2-3 days). At least as sensitive as contrast enema for diagnosing diverticulitis and abscess and better at diagnosing pathology that isnt diverticulitis and can be used for placement of percutaneous catheter for abscess drainage. How? - enteric contrast is given by mouth or NGT. Whenever possible, 150-200 cc of contrast is given by enema. If colovesical fistula is suspected, do not give IV contrast to see if GI contrast fill the bladder. TREATMENT OPTIONS FOR DIVERITCULITIS: UNCOMPLICATED (does not need an operation) Outpatient - mild pain, minimal tenderness, low-grade fever, insignificant leukocytosis. Clear liquid diet and oral antibiotics Inpatient - comorbid disease, severe manifestations of inflammation, altered immune status. Hospitalization, bowel rest, IV antibiotics, IV fluids. NGT if emesis or evidence of obstruction. Hold oral intake until signs of inflammation has resolved. Lack of improvement of diverticulitis in 72 hrs. means this is likely complicated diverticulitis and should be investigated with a CT scan. ABSCESS - CT guided percutaneous drainage - successful in 75% of patients. Failure is associated with feculent drainage. Treat concurrently with antibiotics aimed at aerobes and B. Fragilis. Obtain sinogram at 4-7 day intervals for development of fistula to colon and

monitor collapse of abscess. Leave drain in place. After leukocytosis and pain resolve, patient can be started on low residue diet and discharged with drain in place. Leave drain in place until surgery which can be performed 10-14 days after discharge. If septic picture redevelops, CT should be redone to see if there are other collections to drain. NO ABSCESS - no immediate diagnostic tests are necessary. Wait 4-6 weeks for resolution of inflammation. Should get colonoscopy to rule out CA or Crohns disease and be to see diveritculi. Hospitalization rarely requires more than 7-10days. ? Operation - those who have been treated as above (successful medical therapy), 1/3 will have a second attack and 25% of those will need hospitalization, usually within the first year. Mortality climbs with each successive episode of diverticulitis. WARN ABOUT THE POSSIBILITY OF A COLOSTOMY (ENCOUNTERING UNDRAINED ABSCESS, DILATED BOWEL PROXIMAL TO SIGMOID, ETC) COMPLICATED DIVERTICULITIS (REQUIRING OPERATION). INDICATIONS: ABSOLUTE: *Perforation./Peritonitis/Acute Abdomen *Obstruction *Abscess (not drainable percutaneously) Abscess - those with successful percutaneous drainage should have resection. Fistula Inability to exclude CA. Stricture. Immunocompromised patient.(should have operation with initial admission) *indications for emergency surgery. RELATIVE: Two or more attacks Persistent mass Persistent urinary tract symptoms EQUIVOCAL One attack under 40 yrs. (data for age of 50 as cutoff is controversial) Abscess - undrained abscesses should be drained operatively. Patients who undergo percutaneous drainage should have resection as these often lead to fistulas, strictures, persistent attacks and other indications. 70% of percutaneously drained mesocolic abscesses will not require surgery. SURGERY FOR DIVERTICULITIS 1.) 3 STAGE - historical interest.

2.) 2 STAGE - resection with Hartmans+colostomy takedown. - resection with anastomosis and proximal colostomy+colostomy takedown 3.) 1 STAGE - resection with anastomosis. Only in those who have received colon prep, so cant be used in those with perforation or pelvic abscess. SITUATIONS: 1.) FECULENT OR PURULENT PERITONITIS - 2 stage procedure with resection of involved colon and colostomy. 2.) ABSCESS THAT HAS BEEN PERCUTANEOUSLY DRAINED - single stage with preop bowel prep 3.) UNDRAINED PELVIC ABSCESS - usually a 2 staged procedure. If resection and anastomosis with protective proximal colostomy are to be performed, an intraop bowel cleansing will be necessary. No difference in the morbidity/mortality for the two 2 stage procedures. 4.) UNDRAINED PERICOLIC/MESOCOLIC ABSCESS - can be dealt with in a 1 stage procedure. Because of the limited inflammatory process with a mesenteric abscess, a prep bowel prep can be performed allowing a resection and anastomosis. 5.) COLOVESICAL FISTULA - single stage procedure. Resect colon. If there is an identifiable defect in the bladder, close in two layers. If no defect is seen, distend the bladder with 0.5% iodine through Foley to look for leak. If no leak, do not look further - will need to leave Foley for 7 days anyway. Cystogram prior to d/cing the foley. 6.) COLOVAGINAL FISTULA - single stage procedure. No need to repair the vaginal defect after the sigmoid has been resected. 7.) COLOUTERINE FISTULA - standard has been one stage procedure with resection of the uterus, though the study group is not large enough for adequate analysis. 8.) CHRONIC STRICTURE - single stage. TECHNICAL ASPECTS PREP - place in lithotomy position in case rectal irrigation or stapler may be needed. Place Foley for bladder drainage and irrigation if necessary. Ureteral stents may help. INCISION - MIDLINE. MOBILIZATION - look for perforation or purulence. Note structures that the sigmoid is stuck to. GET FROZENS TO RULE OUT CANCER. If the sigmoid is stuck to the bladder, it should be free inferiorly. Gently peel the colon off the posterior bladder, the trigone is unlikely involved. If adhesions are dense, probably a fistula. If defect is seen, close in 2 layers. - if hysterectomy has been preformed previously, colon may be stuck to vaginal cuff. This is unlikely a fistula. Easier to get at once upper rectum has been taken down. MOBILIZATION OF COLON - start with the descending colon and take down the lateral peritoneal attachments up the level of the inferior pole of the left kidney. Dissect the colon from retroperitoneal attachments to the level of the gonadal vessels. This will allow identification of the ureter, which can be traced inferiorly. Distal descending colon is divided at the level where there is no inflammatory reaction. Do not worry about diverticuli in proximal specimen, diverticuli in descending(or transverse colon) do not cause disease. - continue dissection inferiorly with both ureters identified. Dissection often becomes tougher near the iliac bifurcation. (at this time if the colon is stuck to the vaginal cuff, it can be mobilized)

- distal bowel is divided and the specimen removed with ureter in sight. Inspect the ureter for leak or devascularization. ON TABLE PREP - clamp terminal ileum with bowel clamp. Insert catheter via appendicostomy and irrigate with saline followed by 0.5% iodine. Irrigate rectum with sigmoidoscope. ANASTOMOSIS- performed to upper rectum

BREAST

Scenario #38 - Inflammatory Breast CA 40 y.o. female presents with a 6 cm erythematous left breast which she has ignored for 1 year. She has palpable axillary nodes. Issues: a.) What is the appropriate workup of this patient? b.) Assuming that the bone scan lights up in several places and the FNA shows CA, what should you do? Mammograms necessary? c.) What is the stage of this cancer and what are your recommendations to her? d.) Is there a role for surgery in this patient? ANSWER: DIAGNOSIS - BIOPSY + FOR CA AND 3 CLINICAL FINDINGS 1.) peau dorange/edema 2.) erythema (heat) 3.) ridging/wheals of skin BIOPSY = cytologic aspiration, needle core, or incisional - need not demonstrate dermal invasion of lymphatics if al l of the clinical criteria are present. Send for ER and PR. ** > 1/3 HAVE METASTATIC DISEASE AT PRESENTATION. SYSTEMIC DISEASE. WORKUP: 1.) CEA, CA 15-3 - can use for serial monitoring if found elevated. 2.) CXR 3.) Bone Scan - follow up with bone X-rays if positive. 4.) CT of abdomen - rule out live mets. 5.) BILATERAL MAMMOGRAMS - rule out contralateral disease and serve as a baseline for treatment. Many will develop contralateral disease during the course of treatment. 6.) ER and PR - small subset of tumors may respond to hormonal manipulation.

TREATMENT: CHEMOTHERAPY - INITIAL FORM OF TREATMENT FOR IBC, A SYSTEMIC DISEASE. F= 5-FU A= Doxorubicin C = Cyclophosphamide V= Vincristine P = Prednisone. - 3 cycles of chemotherapy are given and the response to therapy is gauged, the next step of therapy is based on the response to chemo. Those with clinical response or partial response should have mastectomy to measure the residual disease in the breast. Prognosis and 5 year survival correlate with the amount of residual disease in the breast. Those who have surgery are followed up with more chemotherapy and XRT. SURGERY - does not affect survival, as the difference in disease free survival is no different for those who have surgery from those who dont. However, surgery can provide important information about the response to chemo. Those that respond to chemo can have long term survival and can be spared the highest dose of XRT and its late complications. For those that dont respond to chemo, surgery is only a debulking procedure. Scenario # 39 - DCIS/LCIS 55 y.o. female referred for mammography finding and stereotactic biopsy giving diagnosis of DCIS. Issues: a.) What do you do? What questions would you ask the previous surgeon? b.) With path of DCIS, what would you recommend? c.) If the path is LCIS, what would you recommend? d.) how would you follow this patient? e.) what would you tell the person who asks about the risk of CA with fibrocystic breast disease? f.) what if the biopsy reveals sclerosing adenosis and a small focus of LCIS - is any further workup needed? Any further therapy needed? ANSWER: a.) Path slides for own pathologist, presence of positive margins, mammograms to determine other sites of microcalcifications. b.) If no contraindications, local therapy and radiation. DCIS - DEFINITION = transformation of normal ductal epithelial cells into malignant cells that remain in anatomic position and have not yet broken through the basement membrane where lymphatic and vascular structures are. DCIS by definition cannot metastasize. - it is thought that DCIS may be the precursor to invasive cancer if the malignant cells break through the basement membrane and are capable of metastasis. Therefore, surgery should aim to use minimal therapy to remove cancerous cells and cure chance of cancer while preserving the breast.

AKA = intraductal carcinoma which is confined to the ducts (as opposed to the terminal lobules in LCIS). Pagets disease of the Nipple - represents extension of intraductal carcinoma to involve the nipple and histologically consists of permeation of the nipple epidermis by carcinoma cells. Almost always is associated with DCIS of the immediate subaureoloar ducts, and sometimes with coexisting invasive carcinoma. MICROSCOPIC PATTERNS OF DCIS: COMEDO DCIS - marked by cellular atypia and central necrotic tissue, like a zit, hence its name. Believed to be a (single) unique subtype and likely a precursor of invasive carcinoma. BAD. NONCOMEDO DCIS - several subtypes.(cribiform, micropapillary, solid, papillary) displaying various amounts of atypia and microscopic patterns.

GRADING OF DCIS: HIGH - cytologic atypia and necrosis(zit) - COMEDO DCIS INTERMEDIATE - show either central necrosis or atypia but not both (as in HIGH) CRIBIFORM, MICROPAPILLARY, SOLID. LOW - proliferating cells without atypia or necrosis - CRIBIFORM OR MICROPAPILLARY WITHOUT ATYPIA OR NECROSIS. PRESENTATION - can present as mass, nipple discharge, Pagets disease, or asymptomatic (like invasive CA). Seen most often now asymptomatically as part of a screening mammogram. MAMMOGRAPHIC FINDINGS: calcifications (75%), soft tissue abnormality (10%), both (12%). TREATMENT OF DCIS: CONTROVERSY - while total mastectomy is curative in 100% of cases, mastectomy for a benign lesion is more radical than accepted local treatment of a breast malignancy. Problem with local therapy - high incidence of multicentricity (lesions outside index quadrant) and patients who develop recurrence may not be detected by mammography and may develop systemic breast CA disease and die. Radiation - those that receive local resection without radiation have 18% recurrence rate while those treated with resection and radiation only recurred 9% of time. Therefore, benefit of radiation is to lower recurrence rate. Most effective way of lowering recurrence and preserving breast.

INDICATIONS FOR LOCAL THERAPY - DCIS in patients whom the entire tumor can be surgically removed with negative histologic margins and in whom the remaining breast tissue can be reliably assessed clinically and radiographically. CONTRAINDICATIONS TO LOCAL THERAPY 1.) Extensive multifocality (another lesions within the index quadrant) may be present and make it impossible to excise tissue to obtain a negative histologic margin and is a contraindication to local therapy.(CANNOT EXCISE TO A NEGATIVE MARGIN). 2.) Presence of extensive microcalcifications throughout the breast. These may represent other areas of DCIS but regardless (CANNOT RELIABLY ASSESS THE BREAST RADIOGRAPHICALLY) FOLLOW-UP - CLOSE LCIS DEFINITION = transformation of normal ductal epithelial cells into malignant cells that remain in anatomic position and have not yet broken through the basement membrane where lymphatic and vascular structures are. = LCIS is confined to the terminal lobules. PRESENTATION: always an incidental finding, unlike DCIS which can present with mass, Pagets or nipple discharge. INCIDENTAL FINDING ON BIOPSY. Clustered microcalcifications of DCIS are not seen with LCIS, few mammographic clues of LCIS. Often, a benign lesion will appear suspicious and biopsy will show a benign lesion and incidental LCIS. 30-50% chance of bilateral LCIS if contralateral biopsy done. 1/3 of patients with LCIS will develop invasive CA, 50% in index breast, 50% in contralateral breast. Hence, LCIS should have 100% multicentricity (arising outside the index quadrant) if looked for diligently enough. -subsequent cancer can be lobular or ductal. TREATMENT OF LCIS - since the risk is the same for both breasts, the treatment must be the same for both breasts. OPTIONS: BILATERAL TOTAL MASTECTOMY OR OBSERVATION. BILATERAL TOTAL MASTECTOMY - for those who cannot or will not follow up, for those who have a strong family history of breast CA, those who are difficult to follow clinically or radiographically (extensive fibrocystic disease, previous radiation therapy). e.) what would you tell the person who asks about the risk of CA with fibrocystic breast disease? f.) what if the biopsy reveals sclerosing adenosis and a small focus of LCIS - is any further workup needed? Any further therapy needed? FIBROCYSTIC DISEASE - not so much a disease as changes in breast tissue related to hormonal cycling typified by fibrosis, cysts and adenosis. may be asymptomatic (as in women with tender breasts) or symptomatic (as in women with nodular, painful breasts). symptomatic women need to be treated (as with Danazol).

- changes do not confer an increased risk of CA. However, breasts are more difficult to follow clinically and mammographically be cause of the nodularity of the breasts. Patients need excellent monthly self BSE, biannual breast exams by MD and annual mammograms. New masses need to be pursued aggressively even if mammogram is negative.

Scenario # 40 - Nipple Discharge 35 y.o. female who is otherwise healthy who is otherwise healthy comes in with the complaint of a 2 month history of intermittent left nipple discharge. Issues: a.) What is the initial differential diagnosis? b.) What do you do if you cannot find the offending duct on the first examination? c.) What do you do if you cannot find the offending duct on the second examination? d.) Is a mammogram indicated? e.) Is cytology useful as a positive predictor of the presence of cancer? f.) Is the diagnosis of carcinoma likely in this setting and is it reasonable for this patient to be followed if she elects this option? Differential Diagnosis: 1.) CA - bloody or serous/bloody discharge. 20-30% of those with bloody nipple discharge will have CA. 2.) Intraductal Papilloma - bloody or serous bloody. 3.) Fibrocystic disease/change - green, yellow or brown. Brown can look like old blood and can be differentiated from blood as in CA or papilloma using a guaiac card. 4.) Subaureoloar infection - discharge is purulent and nipple is erythematous. Treat with antibiotics. 5.) Galactorrhea - bilateral - may be due to elevated prolactin from a pituitary prolactinoma or from medications such as phenothiazines, Tricyclic antidepressants, oral contraceptives. should check prolactin levels and thyroid function tests. WORKUP; 1.) GUAIAC DISCHARGE 2.) MAMMOGRAMS 3.) CONSTRAST DUCTOGRAM - perform only in high likelihood, i.e. those with guaiac positive discharge. If a subareolar defect is found, a fine lacrimal duct probe is inserted into the offending duct.Incision made around the edge of the areola one half the circumference of the nipple. Nipple/ areola complex is elevated and dissected from the surrounding fat. duct containing probe is isolated and subareolar tissue is taken. Same approach to discharge from fibrocystic disease that is unrelenting. Discharge cytology - high false positive and negative rate.

Scenario #41 - Breast Mass 52 y.o. postmenopausal female presents to your clinic with the complaint that she has discovered a 1.5 cm. new mass in the upper outer quadrant of her left breast. Issues: a.) What is your workup of this lesions? b.) Assuming that she comes in with a nonpalpable lesion with microcalcififcations on mammography in the same location that is not palpable, what is your workup? c.) What would you tell this patient regarding your recommended course of action to establish a diagnosis and treatment? Is a contralateral biopsy needed? d.) What would you tell this patient if an FNA showed malignant cells? e.) What advice would you give about breast reconstruction? f.) Will treatment (surgical and/or chemo) vary depending on whether or not the patient is preor post menopausal? g.) What would you recommend about adjuvant chemotherapy if this were a premenopausal woman? h.) In this post menopausal woman, what chemotherapy would you recommend? Tamoxifen? 45 y.o. female sent in by LMD because of a stellate lesion seen on the mammogram. No calcifications. Her mother and sister both have had breast CA, and she has had lumps removed from her breast before - all of which were benign. Issues: a.) What would you recommend of her mammogram? b.) Biopsy comes back as invasive lobular carcinoma, what would you recommend given her strong family history? d.) What advice would give for this premenopausal woman if the path shows negative nodes? positive nodes? e.) Would you recommend Tamoxifen? ANSWER: WORKUP OF DOMINANT MASS: KEY - FOR WOMAN OVER 30, START WITH BILATERAL MAMMOGRAM. FOR WOMAN UNDER 30, SELECTIVE USE OF MAMMOGRAPHY WHEN INDICATED. WOMAN UNDER 30 : 1.) BREAST ULTRASOUND: used to rule out cyst/adenoma and direct mammography. 2.) US= cyst or adenoma - Observe. If changes in cyst or adenoma, BIOPSY. 3.) US= not cyst or adenoma - BIOPSY. WOMAN OVER 30 (PREMENOPAUSAL) : 1.) BILATERAL MAMMOGRAM - sensitivity and specificity increases with age. Risk of exposure to radiation decreases with increasing age. Utility of mammograms minimal for women under 30 because of density of breast tissue. 2.) NEEDLE ASPIRATION CYTOLOGY 3 options:

- cyst which resolves upon aspiration. Nonbloody aspiration. Can be observed - if it does not recur, it is benign. If it recurs or enlarges it should be biopsied. - yields diagnostic cytology benign - in order for observation, the aspirate must be diagnostic and benign. If the aspirate does not contain appropriate cells, a formal biopsy should be done. If this is to be observed, it should be corroborated by other evidence, such as a breast ultrasound in women under 30 or either a normal mammogram or second benign aspirate in women over 30. - nondiagnostic cytology - such as no cells. formal biopsy. WOMAN OVER 30 (POST MENOPAUSAL) 1.) BILATERAL MAMMMOGRAM - all post menopausal woman should go immediately to mammogram. 2.) BIOPSY - because of the high risk of CA in this age group, biopsy should be done for any mammographic abnormality. ASPIRATION CYTOLOGY - 10cc syringe and 21 gauge needle. No local. Insert needle into mass. Withdraw/suction on the needle making several passes through the mass without withdrawing the needle from the skin. If cystic and disappears on aspiration, return in 4-6 weeks for re-eval. If bloody, send for cytology. - if solid, material in barrel of needle should be placed on slide and fixed. Cytology - 10-15% FALSE NEGATIVE. 5% FALSE POSTIVE. - if benign, should have normal mammogram to corroborate for observation. return in 6-8 weeks. BIOPSY INCISIONAL - removes a portion of the mass. Should be reserved for those with lesions 3 cm or larger/too large to allow complete excision without unacceptable cosmetic sequelae. EXCISIONAL - removal of the entire mass, allows optimal diagnostic. - incision should be over the mass and oriented circumareolarly such that it could be encompassed in mastectomy incision. SURGICAL OPTIONS: PREOP ASSESSMENT BILATERAL MAMMOGRAMS - already done. Used to detect the size of the lesion in the ipsilateral breast and the presence of multicentric cancers or microcalcifications. Screen contralateral breast for malignancy. CXR - rule out met disease to the lungs. Alkaline Phosphatase - with normal alk phos and normal physical exam, chance of bone mets or intrabdo mets is minimal. BREAST CONSERVING THERAPY VS. MRM -

- BREAST CONSERVATION = size of breast vs size of lesion. FACTORS FAVORING BREAST CONSERVATION 1.) Patient preference. 2.) Tumor size vs. breast size favorable and in good position. 3.) Unifocal tumor 4.) Anticipate ability to follow with physical exam and mammography. FACTORS CONTRAINDICATING BREAST CONSERVATION/IRRADIATION; 1.) Total excision of mass causes significant breast deformity. 2.) Multiple microcalcifications - inability to exclude multicentric tumors will make radiographic followup difficult and should have MRM. 3.) extensive intraductal carcinoma in infiltrating ductal CA. 4.) lobular CA 5.) Patient preference. 6.) contraindication to radiation - like pregnancy. 7.) Inability to achieve negative margins on excision (either invasive or in situ). 8.) Inability to follow the patient closely by physical exam or radiographically. Importance of Radiotherapy to Breast conservation - lowers recurrence rates from 40% to 10%. TECHNIQUE OF BREAST CONSERVATION - incise over mass (ellipse skin if old incision en bloc). If previous incision and biopsy, do not enter cavity - simply reexcise around old site. Orientation sutures are beneficial. Perform ALND. BREAST RECONSTRUCTION: TIMING - while reconstruction can be at the same time or delayed, it is generally better delayed if consideration is being given to radiation therapy. (unusual in case of MRM). No problem with timing of chemotherapy when considering reconstruction. Trend now is toward immediate reconstruction. OPTIONS; 1.) TRAM - autogenous reconstruction of choice. Rectus abdominus muscle and island of skin are mobilized and rotated. Can also use latissimus dorsi flap or omental flap. 2.) Prosthesis - usually an implant into subpectoral region, need to preserve pectoralis fascia and median pectoral nerve during mastectomy. May need tissue expander placed at the time of mastectomy to expand the skin. Complications - infection, tissue loss and poor cosmetic results. ADJUVANT CHEMOTHERAPY RATIONALE FOR THERAPY - large number of prospective trials have confirmed improvement in disease free and overall survival with adjuvant chemotherapy in premenopausal women and a lack of benefit in those who have gone through menopause or who are over 50. - have also shown a major survival advantage after adjuvant chemotherapy in breast cancer patients under 50 years with positive nodes.

CONSENSUS - TREAT PREMENOPAUSAL WOMEN WITH NODE POSITIVE BREAST CA WITH ADJUVANT CHEMOTHERAPY AND TREAT POSTMENOPAUSAL WOMEN WITH NODE POSITIVE BREAST CA WITH TAMOXIFEN. MILAN TRIAL - median survival of premenopausal women with node positive disease who were not treated is 8 years. Median survival of premenopausal women treated with CMF (cyclophosphamide, methotrexate and 5-FU). Why? - uncertain. May be that chemo causes a chemical oophorectomy by killing the ovaries early in premenopausal patients giving antiestrogen result. - may be that premenopausal women can tolerate higher doses of chemo and subsequently get more therapy which is effective in killing tumor cells. - may be that premenopausal malignant cells are biologically different and respond differently to therapy. Chemotherapy + Tamoxifen - would think a natural synergistic effect of the two but studies are not that clear. Some have actually shown decreased survival with the combination. POSTMENOPAUSAL WOMEN WITH NODE POSITIVE DISEASE - not the benefit of adjuvant chemotherapy seen in those premenopausal/under 50 years. What to do? - those with ER positive tumors and positive nodes should be treated with tamoxifen.. TREATMENT OF NODE NEGATIVE PATIENTS - several studies have shown a slight but not statistically significant improvement in overall survival in patients with node negative disease. I t seems that some node negative patients would benefit from postop chemo but it is not yet known which ones. Patients with larger tumors may benefit from chemo even with negative nodes. TUMOR SIZE PREMENOPAUSAL POSTMENOPAUSAL ER (+) ER(-) ER(+) ER(-) < 1 CM, NEG.NODE NT NT NT NT > 1 CM, NEG.NODE TAM. CHEMO TAM CHEMO POS. NODES CHEMO CHEMO TAM* CHEMO * - same results as chemo, but tamoxifen less toxic - those post menopausal, node positive woman have improved disease free survival but not overall survival. Premenopausal, node positive women have improved disease free and overall survival. - appears to be increased disease free survival, but probably not overall with adjuvant chemotherapy. Results of trials are needed.

HEAD AND NECK


Scenario #42 - Parotid Mass 40 y.o with 2 cm mass at the angle of the mandible that has been present for at least one month and may be slowly increasing in size. Issue: a.) Initial work-up and differential diagnosis of this mass. b.) Operative approach to this lesion, including what is it most likely to be? c.) What are the types of parotid tumors and how do they dictate your initial management? d.) Operative approach if the facial nerve is going through the center of the mass? After excision of the facial nerve, what are the possible sequelae and possible remedies to these? e.) What are the indications for sacrificing the facial nerve? f.) What are the indications for an accompanying neck dissection? ANSWER: DIFFERENTIAL DIAGNOSIS: 1.) Suppurative parotitis 2.) Chronic parotitis 3.) Recurrent chronic sialadenitis 4.) Chronic sialolithiasis 5.) Trauma 6.) Mucocoele 7.) Cyst 8.) Fistula 9.) Tumor - benign or malignant. DIAGNOSIS AND WORKUP: CLINICAL FACTORS - need to know whether the facial nerve is paralyzed preop as it is unlikely that its function will return after a parotid resection. RADIOLOGY - should be used for mass lesions and for localization in either a deep or superficial lobe. BIOPSY - is not appropriate for differentiating malignant from benign (test is not that sensitive) All masses of the parotid with normal facial nerve function will be treated with superficial parotidectomy. Open biopsy is discouraged for fear of injuring the facial nerve. Since surgery will be utilized for removal of benign lesions and frozen section can be obtained at that time, distinguishing benign from malignant preoperatively will not save the patient an operation. STAGING: 1.) Tumor Size 2.) Local extension of the tumor. 3.) Spread to regional nodes. 4.) Distant Metastasis.

TYPES OF BENIGN PAROTID TUMORS PLEOMORPHIC ADENOMAS - 90% of all benign lesions, are slow growing and may have a thin capsule. Unless they have undergone malignant degeneration, these are benign encapsulated lesions. do not invade facial nerve. 100% curable if the lesion is removed with entire capsule, but if small pseudopods are left behind it will recur. Re-resection is difficult because of multifocality of recurrence and high likelihood of injuring facial nerve. ADENOLYMPHOMA (WARTHINS TUMOR) - SECOND MOST COMMON - 10% bilateral. Superficial, located in the tail of the gland. do not become malignant, do not recur. Do not enucleate and require partial parotidectomy. ONCOCYTOMA - rare, tend to be bilateral. do not recur. MONOMORPHIC ADENOMA - thought to be a precursor lesion to either pleiomorphic adenoma or adenoid cystic carcinoma. Importance is avoiding confusion of this lesion with malignant adenoid cystic carcinoma. Treatment of Benign Lesions - Superficial Parotidectomy - implies sparing of the facial nerve. The exception to this is a tumor that lies in the deep lobe. Re-resection for recurrence of pleiomorphic adenoma is also done superficially to spare the facial nerve. - serves as a diagnostic procedure or a grand biopsy to remove the entire superficial lobe intact without capsular disruption. Frozen section is obtained at this point to determine whether the lesion is malignant or benign. Should have frozen section diagnosis before sacrificing any vital structures. TYPES OF MALIGNANT PAROTID TUMORS: MUCOEPIDERMOID - MOST COMMON PAROTID MALIGNANCY. Can be low or high grade. Low grade have excellent prognosis. High grade tend to recur and metastasize. ADENOID CYSTIC CARCINOMA - A.K.A. CYLINDROMA - marked by perineural invasion and mets to lymph nodes and lungs. Tend to be slow growing and tend to recur, so usually a long course that results in death. Good short term prognosis is deceptive, because long term prognosis is really shitty - dying. MALIGNANT MIXED TUMOR - tend to occur in the presence of pleiomorphic adenoma, leading some to believe it is malignant degeneration of pleiomorphic adenoma. Tend to present as a sudden enlargement or facial nerve paralysis/pain in a longstanding parotid tumor. worse prognosis than the other low grade malignancies (adenoid cystic and mucoepidermoid).

ACINOUS CELL CARCINOMA - another low grade malignancy that tends to grow slowly and recur. Lymph node mets about 15%. ADENOCARCINOMA - these are all glandular tumors that do not fit into the above categories. Fall into 3 grades which range from low grade to high grade with prognosis that range from good to shitty. SQUAMOUS CELL CA - sometimes difficult to distinguish primary involving skin from a skin primary involving/metastatic to the parotid. Aggressive tumor with shitty prognosis. Treatment of Malignant Parotid Tumors - complete conservative parotidectomy - implies preservation of main trunk of facial nerve and its branches. SPECIFIC SITUATIONS IN PAR0TID RESECTION 1.) Tumor adherent to nerve - can be dissected off and treated with tumoricidal doses of radiation (5000 -6000 Gy). 2.) Low grade malignancy involving portion of facial nerve - should be treated with subtotal parotidectomy with preservation of uninvolved branches of the facial nerve. 3.) Massive tumors involving facial nerve with facial nerve paralysis, high grade tumors and malignant recurrences should be treated with sacrifice of the facial nerve. Nerve grafting is attempted if the facial nerve worked preop. 4.) Malignancy involving facial nerve - must take along with involved structures. Should attempt grafting with sural or greater auricular nerve graft. Indications for accompanying MRND 1.) High grade lesion that tend to metastasize. 2.) Clinically suspicious nodes. Scenario # 43 - Oral Mucosal Cancer 70 y.o. male, pipe smoker, presents with a 0.5 cm ulcer on his lower lip, slightly to the right side and midline. First noted approximately 8 months ago and has never entirely healed. It is moderately painful with some induration at the margins. There are palpable, mildly tender nodes on the upper neck on the right submandibular triangle. Issue: a.) Differential diagnosis and initial treatment (initial treatment should include the use of oral antibiotics for 2 weeks to see if the lymph nodes decrease in size and tenderness.) Is incisional or excisional biopsy appropriate? b.) Operative approach for this lesion, including initial closure or rotational flap closure should the defect be too large. What is the appropriate margin? c.) Is MOHS surgery appropriate for this type lesion d.) is radical neck dissection indicated if the nodes do not disappear after treatment with oral antibiotics? e.) What is the stage of this lesion? f.) What are 5 yr. survivals with and without positive nodes? g.) How is treatment and survival changed by a tumor located in - anterior 2/3 tongue - posterior 1/3 tongue

- floor of mouth ANSWER: PATHOLOGY - head and neck cancers are either sqaumous cell CA of the mucosal lining of the upper aerodigestive tract. Remainder are salivary gland tumors (see question # 42). Premalignant lesions - several types (leukoplakia, acanthosis, hyperkeratosis, parakeratosis) are all benign in contrast with premalignant leukoplakia which shows atypical cells and is difficult to differentiate from carcinoma in situ. Rarely have malignancy and very small progress to cancer. Erythroplasia - much higher potential for malignant generation (30%). Level of Lymph Node Metastasis Primary Site I - above the digastric(submandibular) Lip, oral cavity, skin II - superior SCM (post to gr. auricular n) Oral cavity, oropharynx, nasopharynx hypopharynx, larynx III - mid SCM (ant to fr. auricular n) Oral cavity, orophar. hypopharynx, larynx, thyroid IV - inf SCM (below omohyoid) oropharynx, hypophar. larynx, cervical esoph. thyroid V - posterior triangle nasopharynx, scalp, breast, lung, GI tract, scalp. - importance is not to memorize but to know that there is an orderly progression of tumor cells through lymphatic drainage. Involvement at multiple layers, it is usually represents a progression from superior to inferior echelon nodes and is considered BAD. Presence of confirmed lymph node met cuts survival rate in half compared to disease localized to primary site. Those with mets at multiple levels in the lymph node chain will not be cured regardless of therapy. PREOP EVALUATION: CT SCAN - useful for deciding the amount of bony involvement of the mandible or maxilla in oral cavity tumors. - assessment of neck lymph node mets is also enhanced by CT scan. PRINCIPLES OF TREATMENT

- for small tumors such as T1 (2 cm or less in largest diameter) or T2 (tumor larger than 2 or less than or equal to 4 cm) surgery and radiotherapy give equivalent results. Factors in selecting one over the other depend on morbidity of treatment, complexity of treatment and other shit, you get the picture. Early laryngeal tumors - best treated by radiotherapy first, reserving surgery for salvage in the case of failure. Early oral lesions - those that can be easily excised perorally in a single setting with primary reconstruction are best treated by single modality surgical therapy and avoiding prolonged radiation treatments and their radiation related complications. - for larger tumors such as T3 (larger than 4 cm) or T4(invading adjacent structures) multimodality therapy - surgery + postoperative radiotherapy is the standard of care. Selective regional neck dissection is used in cases where early lesions have a question of lymph node mets. Radiation - external beam - used for bulky primary tumors and extensive nodal metastasis. Other situations in which radiation is used is in cases where there are positive margins and where there has been extensive soft tissue involvement by the primary or mets. PRIMARY TUMOR + CLINICALLY POSITIVE NODES - open biopsy of node is of little value in diagnosis and detrimental to treatment. If a primary tumor is identified and proven by biopsy, any clinically suspicious nodes have traditionally been treated as metastasis for the purposes of staging and treatment planning. Can use FNA if it will change treatment (as in a questionable node in the presence of a small =T1 or T2 tumor which would add radiotherapy to the treatment plan of an oral lesion.) STAGING 5 YR. SURVIVAL I T1NOM0 70-90% II T2NOMO 60-80% III T1 N1MO 30-50% T2 N1MO T3 N1MO T3 NOMO IV T4 NO-1MO 25% OR LESS T1-3N2-3 MO T1-3 N0-3M1

SURGICAL TREATMENT - BLOCK resection of tumor with grossly and microscopically clear margins in continuity with lymph node shed. MRND- indicated for large tumor (T2 or greater) or in association with clinically positive nodes. LIP - early lesions are easily treated with surgery or radiotherapy, ease of surgery makes a wedge excision of the lip lesion with reconstruction in a single setting. - lip shave procedure - good for areas adjacent to tumor with severe actinic changes and dysplasia. MOHS surgery is an effective alternative. - palpable nodal mets indicate MRND.

Reconstruction - most lip cancers are 2 cm or smaller at diagnosis. For these patients, resection with negative margins will be less than 1/3 of the lip. Shield excision - shield shaped incision is made around the tumor and vermillion border marked with scalpel. Full thickness lip is incised and closed in three layers. W- shaped incision- W shaped incision is made around tumor with point of W near tumor. If more tissue needs advancing the incision can be extended laterally along labiomental folds. - Excision of 1/3-2/3 of lip - can use stair step excision which is a form of a complex Z-plasty. TONGUE ANTERIOR 2/3 - chronic non-healing ulcer. At presentation, 1/3 will have cerivical lymph node mets at presentation. During the course of disease, 2/3 will develop cervical nodal mets. T1 cancers have 5 yr. survival of 80%, nodal mets drop this in half. - early lesions should be excised through oral approach with transverse excision. Margin should be 1.5 cm for small tumors. - MRND for T2 or greater lesions, also for locally invasive tumors. POSTERIOR 1/3 OF TONGUE - 2/3 will present with lymph node mets. FLOOR OR MOUTH - small, early tumors can be treated by surgery or radiation, but radiation can be complicated by osteonecrosis of the mandible. Enlarged salivary glands may represent occlusion of the Wharton glands and may be benign. Smaller lesions should be resected transorally. Larger lesions approximating mandible must be treated with partial mandibulectomy. Extension into the mandible requires a commando type of procedure where they rip off half of your face. As always, stage I and II lesions have a 5 year survival of 70-90% and nodal disease (stage III) reduces this by one half. Scenario #44 - Solitary Neck Mass 30 y.o. female with solitary neck mass/lymph node. Issue: a.) What is the appropriate workup of this neck lesion? What test would you obtain to image areas outside the head and neck? b.) what is the best initial means of obtaining tissue? c.) What operation would you perform if this came back SCCA? AdenoCA? MCT? Hodgkins Disease? d.) What are sites of origin of SCCA above the clavicle? Appropriate workup? ADENOCA Appropriate work- up? ANSWER:

-excluding benign primary thyroid tumors, 80% of neck masses in adults are malignant, of which 80 % will be metastatic. 80% of metastatic neck masses arise from above the clavicle. INITIAL EVALUATION AND EXAMINATION: EXAM - if the mass moves on swallowing, it is likely under the strap muscles and is likely a thyroid primary. - use otoscope, tongue depressor, nasal speculum, glove and mirrors to examine the oral cavity (glove and tongue depressor and otoscope), oropharynx, hypopharynx, larynx and nasopharynx (nasal speculum and mirrors for indirect laryngoscopy). - also examine groin and axilla (adenopathy associated with Hodgkins disease), breasts (breast CA), abdomen and rectal exam (GI tract CA), respiratory tract (lung CA). BIOPSY - FNA Positive Results: Metastatic thyroid - definitive surgery. Lymphoma - remove entire node to confirm and establish cell type. Branchial Cleft cyst - definitive surgery. SCCa with unknown primary - search for primary before providing definitive treatment. - CT scan - CXR - squamous cell lung CA can go to neck. - Upper GI - squamous cell of esophagus. - Head and Neck Exam under anesthesia. AdenoCA with unknown primary - search for primary before providing definitive treatment. - CT scan of chest and abdomen - rule out adenoCA from lung or GI primary. Also rules out renal cell/upper Gusystem tumor as primary. - UGI/Upper endoscopy + BaE/Colonoscopy - rule out adenoCA of GI primary. - Mammogram - rule out breast CA primary. - Cystoscopy - rule out lower GU primary Indeterminate Results - proceed to open biopsy. What kind? - parotid - None. Leads to too high recurrence rate and could injure facial nerve. Do superficial parotidectomy. - small - excisional biopsy. - large - incisional biopsy. Head and Neck Exam under anesthesia - indicated for diagnosed SCCA by FNA without known primary site by initial examination. Examine the oral cavity, nasopharynx, tonsil, base of tongue, hypopharynx and larynx. Biopsy any suspicious lesions. These should be done as frozens. If nothing is seen, proceed to Triple endoscopy. Triple endoscopy = esophagoscopy, laryngoscopy, bronchoscopy. Biopsies - nasopharynx, base of tongue, tonsil, pyriform sinus. Do as frozen to allow repeat biopsy for questionable lesions.

TREATMENT : 1.) NECK - if it can be surgically removed, it should be removed along with a MRND. Need to determine the presence of tumor, the number of affected lymph nodes and the presence of extracapsular spread. If multiple nodes are involved or there is extracapsular spread, postop radiotherapy is necessary. 2.) PRIMARY Nasopharynx, tonsil, base of tongue, hypopharynx and larynx - treated with radiotherapy. Oral Mucosal - lip, anterior 2/3 of tongue, floor of mouth - surgical excision. Metastatic SCCa without known primary = 3% - results are the same after MRND if you irradiate the head and neck or whether you wait for the primary to show up. When it does show up, you can always treat with surgery or radiation. If you decide to observe, check every month for first year, every 2 months for next year and every three months for third year.

TRAUMA
#45 - Polytrauma with Pelvic Trauma 25 y.o. female who fell three stories to the pavement and brought to ER. Initial vitals BP=100/60. RR = 22. Issues: a.) How would you manage this patient? ABCS and initial evaluation (primary survey) and intervention. b.) Secondary survey shows blood at the meatus and pelvic film showing inf/sup ramus disruption and widened SI joint? What intervention is necessary before placing a foley? What if an extraperitoneal bladder injury is found? c.) How would you treat this pelvic fracture if the patient becomes unstable? How would you rule out other injury? how is this performed? If the other injury is ruled out, and the patient becomes more unstable (from pelvic injury) what is appropriate intervention? Discuss which types of pelvic fractures do and do not benefit from an external fixator? d.) CXR shows widening of the mediastinum. What does this suggest? What is the mechanism of this injury and how is it surgically repaired? e.) CXR shows haziness on right side. CT is placed and puts out 500cc? What is the initial output and hourly output that would prompt you to operate on this patient? ANSWER: PELVIC FRACTURES:

-many patients with severe pelvic trauma have multiple injuries, such that the source of hemorrhage may be the pelvic fracture itself, or some combination of associated intraperitoneal, thoracic or long bone injuries. Only after the major sources of hemorrhage have been found and treated and the patient is stabilized should the associated structures (rectum, bladder and nervous system) be evaluated. CLASSIFICATION - multiple classification systems, revolve around the stability of the fracture which is dictated by the number and position of the fracture sites. UNSTABLE - generally involves more than one fracture site and disruption of the pelvic ring. Open Book - pelvic ring disruption - with AP force, the pelvis separates anteriorly and hinges open on the posterior sacroiliac joints. Pubic diathesis. Vertical Shear - pelvic ring disruption in front and back with force from below resulting in displacement of one hemipelvis superior to another. Lateral Force - force comes from lateral side disrupting the pelvic ring in front and in back. Crush Injury - involves several breaks in the pelvis ** direct correlation between the number of fractures and the number of blood transfusions necessary. STABLE - involve single breaks in ring or iliac crest or symphysis. MANAGEMENT - hemorrhage is the leading cause of death in severe pelvic fractures and survival depends on rapid identification and control. 2 situations in which there is a delay in treating bleeding (with increased morbidity and mortality) is a delay in diagnosis and putting the patient through a negative celiotomy. UNSTABLE PATIENT - ongoing bleeding from pelvic hemorrhage - need to control with external fixator or MAST trousers. Approach #1- place MAST (legs to 50 mmHg and abdo to 40 mmHg) trousers in ER and then do DPL. If positive, celiotomy. If negative, arteriography and embolization. Approach #2 - take to OR for DPL. If positive, celiotomy. If negative, place external fixator in OR. If bleeding continues, go for arteriogram. DPL - supraumbilical tap yields a false positive rate of 10-18%. CELIOTOMY - midline incision is used. Pelvic hematomas are not to be opened unless rapidly expanding or pulsatile. That is, never. Bilateral internal iliac artery ligation is not successful be cause of the rich collateral network. HEMODYNAMIC INSTABILITY AFTER CELIOTOMY - patient should be taken immediately to arteriography for emobilization. Embolization will stop arterial bleeding but not venous ooze, so if arteriogram shows no arterial bleeding, an external fixator (or MAST trousers) should be placed (if not already placed) to decrease the venous oozing. If arteriogram shows leakage, embolize. Then to ICU for observation.

UROLOGIC INJURIES - URETHRAL INJURY SHOULD BE SUSPECTED IN ANY MALE WITH A PELVIC FRACTURE. Anyone with blood at meatus, high prostate, scrotal hematoma should have retrograde urethrogram prior to placing Foley. Treatment of Urethral Injuries - place suprapubic tube. Most partial tears will heal spontaneously. BLADDER INJURY - If urethrogram is negative, place Foley catheter. Indications for cystogram are gross or microscopic hematuria with pelvic fracture. TREATMENT EXTRAPERITONEAL - treat with drainage (foley or suprapubic). INTRAPERITONEAL - operative drainage of bladder with 3 layer closure. BLUNT THORACIC VASCULAR INJURY falls from great heights with deceleration when hitting the ground, deceleration injuries from MVA and direct blows to the precordium can cause blunt injury to the aorta or the innominate artery. = 15% will survive the initial insult. Of those that make it to the ER, 50% will rupture in the first 24-48 hrs. = strong history of mechanism and widening of the superior mediastinum more than 8 cm, loss of aortic knob, multiple left rib fractures, pleural cap or left pleural effusion indicates retrograde femoral arteriography to detect site of aortic tear. Site of injury at the ligamentum arteriosum distal to the origin of the left subclavian artery. REPAIR - left posterolateral thoracotomy in fourth interspace after placing double lumen tube. Isolate the aortic arch, descending aorta and left subcalvian artery with umbilical tapes.. Clamp above and below and sew in a Dacron interposition graft. Rate of paraplegia is the same regardless of type of repair used.

#46 - Blunt Trauma with Hypotension 18 y.o. male S/P MCA comes into E.R. with normal GCS and BP =60. C-spine, pelvis, CXR are clear. DPL is grossly positive. Urine is pink. Issues: a.) ABCs. b.) How would evaluate the urine/kidneys? c.) How would you deal with patient with hypotension/+ DPL. d.) Intraoperatively, you encounter a large midline retroperitoneal hematoma. How will you get appropriate control? e.) Once you have appropriate vascular control, how would you expose and explore this hematoma?

f.) You find that the pancreas is transected over the spine. How do you deal with this? How do you deal with the spleen? g.) You find the left renal avulsed at the origin, what do you do? What if there is a blow-out and reimplantation cannot be performed? ANSWER; a.) b.) One shot IVP c.) SRA or IRA clamp. Iliac clamp distally. d.) SRA/ Right - Kocher and right colon mobilization. e.) SRA/Left - mobilize the spleen and left colon to midline. IRA - clamp below renals, clamp at iliacs. Open midline. f.) see below. g.) Reimplantation/repair difficult in blunt trauma -> opt for nephrectomy.

RETROPERITONEAL INJURY: Hemodynamically unstable patient with pelvic fracture - to OR for supraumbilical DPL. If positive, celiotomy. If negative, external fixator or arteriogram. Hemodynamically unstable patient without pelvic fracture - to OR for celiotomy. Hemodynamically stable with or without pelvic fracture - CT scan. ZONE I INJURY - superior from aortoesophageal hiatus to sacral promontory (aortic bifurcation). Lateral to the proximal renal vessels. Contains the aorta, cava, duodenum, pancreas, proximal renal vessels. ALL MUST BE EXPLORED. EXPOSURE AORTA CLAMPING SUPRARENAL - in lesser sac, divide the right crural fibers over the aorta. Clamp here. - alternatively, thoracotomy may be performed in OR. INFRARENAL - below renals as in AAA. - DONT FORGET TO GET DISTAL CONTROL AT THE ILIACS BEFORE MOBILIZATION. -EXPOSURE OF THE SUPRARENAL AORTA = SMA, IMA, LEFT ILIAC AND LEFT RENAL ORIGIN - reflect the spleen and left colon toward the midline. CAVA, RIGHT RENAL HILUM, RIGHT ILIAC ARTERY - perform a Kocher maneuver combined with mobilization of the right colon. Arterial injuries - suprarenal aorta, celiac axis, SMA and renal artery. Venous Injuries - renal vein, peripancreatic veins. -EXPOSURE OF THE INFRARENAL AORTA - can expose the infrarenal aorta as one would for an infrarenal AAA in midline

Arterial injuries - infrarenal aorta or IMA. Venous injuries - IVC --EXPOSURE OF THE IVC - whole infrarenal IVC can be exposed by mobilizing the right colon and performing a Kocher maneuver. Cava can be controlled by clamping with a Satinsky and compressing with sponge sticks. Posterior holes may need to be closed through the anterior defect (transluminal repair). Ligation of IVC - only for desperate circumstances. Ligation of renal veins is fatal. ZONE II - from diaphragm to iliac crests lateral to psoas muscle. Implies injury to kidney or colon. One shot IVP = should be used when patient is taken directly to OR from ER because of hypotension or + DPL. Shows whether there are two functioning kidneys (rules out renovascular injury), but does not show injury to collecting system. Indications for exploration Intraoperative : (1) Expanding or pulsatile perinephric hematoma. (2) Suspected renal vascular injury. Control Intraop - proximal control of renal artery and vein prior to opening hematoma by mobilizing the right or left colon. This may be difficult in patient who is hemodynamically unstable, in this case, use digital control of the hilum and rapid exposure of the kidney. DECISION - whether to perform nephrectomy or not. Must have idea of contralateral function via IVP. Assuming contralateral kidney functions, nephrectomy if: - operated on after 6-8 hrs. - unstable patient - associated life threatening injuries. - severely injured kidney ?Revascularization of Renal vascular Injury - because of occlusion or intimal disruption. Blunt Trauma - no. Rarely successful. Penetrating Trauma - possibly. ZONE III - bladder anteriorly, sacral promontory posteriorly and iliac crests laterally. Do not explore unless it has ruptured into the peritoneal cavity.

#47 - Trauma with Liver Laceration. 25 y.o. trauma patient is brought to the emergency room after an MVA. Examination is negative except for a tender abdomen with guarding. Heart rate 110, BP=80/palp Issues: a.) What do you do- CT vs DPL vs immediately to OR and what are the advantages of each? b.) Describe preoperative need for lines, positioning, prepping and draping. c.) Describe the initial exploratory lap in a trauma patient.

d.) The only injury you find is a large stellate laceration high on the posterior dome of the liver, which bleeds massively when the packing is removed, what do you do? Should the liver be mobilized? How? How should you obtain vascular control of the livers blood supply? How do you differentiate injury from the arterial supply from the venous supply? How can you directly expose the hepatic veins? If controlling the arterial supply controls bleeding, what do you do next? What should be done if suture ligation does not succeed? How would you perform a hepatic resection from this liver fracture? Would you perform an omentopexy? What should you do if the control/occlusion of the arterial supply does not control the bleeding? ANSWER Airway = for those comatose or obtunded. Apneic = intubate orally. Apneic with facial trauma = emergency cricothyroidotomy. Breathing but need airway control = nasotracheal intubation. Breathing but combative = rapid sequence induction with cricoid pressure and in-line traction. Breathing = pulmonary ventilation is necessary. Confirm by physical exam - no ventilation can occur because of poor EET placement, hemothorax, pneumothorax, aspiration of teeth and shit, pulmonary contusion. Circulation a.k.a. shock - 14 gauge IV X 2 - draw blood for HCT and T&C (and other labs) - Adults = 2L of LR. - Peds = 20cc/kg Response - those that improve and stabilize are unlikely to have ongoing bleeding and can be approached systematically. Those that do not improve or improve transiently and then deteriorate are likely to have ongoing hemorrhage and should be taken to OR. Secondary Survey - detailed physical exam. Rectal exam, Foley, NGT. CXR, pelvic X-ray. Laparotomy - for those who remain hemodynamically unstable after 2L of fluid or those that improve and then deteriorate. DPL - 95% sensitivity. Positive if : 10ml of gross blood, or after instillation of 1L of fluid yields an effluent of 100,000 rbcs. Also positive if effluent seen in NGT, Foley, or chest tubes or enteric material or stool present. Advantage is 95% specificity for major organ injury and speed. disadvantages are the lack of ability to diagnose retroperitoneal injuries. CT scan - advantages are nonoperative/nonintervention means of diagnosing solid organ injury (liver, spleen and kidneys) as well as diagnosing retroperitoneal injuries. Disadvantages are need to prep pt. and time necessary for study, as well as poor identification of small bowel injuries. - indicated for the hemodynamically stable patient in whom nonoperative therapy is an option. also indicated for those in which other CT scan will be necessary (head trauma). Needs supplementation with contrast studies of the upper and lower GI tract. NONPERATIVE THERAPY - better in children than adults - for those who are hemodynamically stable blunt trauma patients. Generally seen in patients with large hematomas and lacerations who are hemodynamically stable. These patients are placed on bedrest and in ICU. 50% will need transfusions but should not exceed 2 units in 24 hrs.

OPERATIVE THERAPY POSITION - SUPINE PREP AND DRAPE - NECK TO THIGHS INCISION - LONG MIDLINE (xiphoid to pubis, if no pelvic fracture.) EVACUATE - liquid blood and clots. PACK - place packs into each quadrant to assess the site of hemorrhage. EVALUATE - each quadrant, pay particularly close attention to the spleen and mesentery as sources of ongoing bleeding as a coagulopathy develops. if the liver is injured, the right costal margin is retracted superiorly and the right and left lobes are palpated for defects. MOBILIZATION OF THE LIVER - retract costal margins superiorly with self retaining retractor. Mobilize the right lobe by dividing the right coronary and triangular ligaments medially taking care not to injure the lateral wall of the right hepatic vein. Retrohepatic cava and right adrenal gland lie here. -- mobilize the left lobe of the liver by dividing the triangular ligament toward the cava taking care not to injure the left hepatic vein or the retrohepatic cava. - if exposure to the retrohepatic veins is needed, the incision should be directed superiorly as a median sternotomy. Divide the pericardium and diaphragm radially. METHODS OF TEMPORARY CONTROL OF LIVER HEMORRHAGE. MANUAL COMPRESSION AND PERIHEPATIC PACKING RIGHT LOBE - difficult to control. Take lap pads, folded, 2-3 at a time and pack around injury site. Place pads between the liver and diaphragm and the liver and anterior costal margin. Have assistant exert pressure down on chest wall. LEFT LOBE - more difficult to pack because of anatomy. However, this can usually be controlled by compressing the left lobe by hand. POTENTIAL COMPLICATIONS - decreased cardiac output from caval compression and increased airway pressure from upward pressure of packs on diaphragm. PRINGLE MANEUVER - best to place the clamp from the left side by opening the lesser omentum while guiding the posterior blade through the foramen of Winslow with the left index finger. This maneuver can differentiate hemorrhage from t he hepatic artery or portal vein from the hepatic vein or caval injury. Can be placed for 1 hr. Should be placed along with packs for life threatening hemorrhage on entering abdomen. HEPATIC VASCULAR ISOLATION - if hemorrhage is still brisk after placing the Pringle clamp, injury is likely to the hepatic veins or retrohepatic cava. 1.) HEPATIC ISOLATION WITH CLAMPING - used frequently in elective situations, but has limited role in trauma because of patients unstable condition and lack of tolerance for clamping of cava and decreased left ventricular filling pressure.

Performed by clamping aorta, suprarenal and suprahepatic cava. 2.) ATRIOCAVAL SHUNT - median sternotomy, take down pericardium and diaphragm. Place umbilical tapes around cava just above renal and around cava as it enters the right atrium just above the diaphragm. 36 Fr. chest tube with additional hole cut about 17 cm from end (intended to be in the mid-atrium). Pursestring suture placed in the right atrial appendage. Guide CT down through atrium and through cava with care taken not to further damage cava. Pursestring sutures are tied and portion of tube outside atrium is clamped. Dont try this at home. 3.) MOORE - PILCHER TUBE - balloon placed through iliac vein and guided up to defect and inflated to tamponade bleeding. Same results as atriocaval shunting (piss-poor) 4.) VENOVENO BYPASS - cut down on axillary artery and femoral vein. Avoids having to open chest. DEFINITIVE MEASURES TOPICAL MEASURES - for minor lacerations and grade I or II injury, manual compression along with topical agents can be used. Can cauterize raw surfaces. Fibrin glue is an option but if you use in front of those old fucks theyd probably laugh to death. SUTURING PARENCHYMA - should be used for lacerations that are 3 cm or less in depth or in situations in which instability does not allow hepatotomy or individual ligation of bleeders. Use 0 or 2-0 chromic on a large blunt tipped needle. Use mattress sutures for deeper lacerations. Tighten knot until hemorrhage ceases and liver blanches. This may work for oozing surfaces and hepatic veins but is unlikely to work for larger hepatic arteries. HEPATOTOMY WITH SELECTIVE LIGATION - SHOULD BE USED IF PARENCHYMAL SUTURE DOES NOT WORK. SHOULD BE USED FOR PENETRATING TRAUMA. Use finger fracture as hepatotomy down to the level of the bleeding vessel. Major vascular structures (right, left or middle hepatic veins) should be repaired instead of ligated. HEPATIC ARTERY LIGATION - used in case of deep hepatic arterial hemorrhage in which Pringle maneuver works to control hemorrhage. if selective clamping of the left or right hepatic artery slows hemorrhage, it can be ligated and tolerated. RESECTIONAL DEBRIDEMENT - used for removal of nonviable parenchyma at periphery. Will incur blood loss with this maneuver, so should be used in those that can tolerate blood loss. PACKING - used not necessarily for the laceration but around the liver between the liver and diaphragm/anterior chest wall to compress the edges of the laceration together. Take to SICU and remove after 24-48 hrs. Use for grade IV and V and those who develop coagulopathy. - should be used for patients who develop the triad of hypothermia, acidosis and coagulopathy. If patient is sufficiently ill, skin can be closed with towel clip and patient rewarmed. COMPLICATION - ABDOMINAL CAOMPARTMENT SYNDROME - when pressure increases to 20 mmHg, caval return decreases. Take patient back, remove blood and packs and re-pack. HEPATIC RESECTION - HAS LARGELY BEEN REPACED BY OTHER MODALITIES. Morbidity of 50%.

2 SITUATIONS: Extensive injury to the left lateral segment of liver. Packing has been successful and necrotic tissue leaves right or left lobe nonviable. HEMATOMAS - smaller grade I and II should be observed. larger can be explored, packed or left6 alone. Those that are expanding should be explored. DRAINS - USE CLOSED SUCTION DRAINS. OMENTAL PACKING - fills defects and buttresses sutures. COMPLICATIONS - INFECTION, BILOMAS (wait for fistula to mature), HEMOBILIA (treat with arteriogram/embolization).

# 47 - Trauma with RUQ Retroperitoneal Hematoma 25 y.o. male S/P MVA from referring institution found on exploratory laparotomy to have a subhepatic hematoma the local surgeon could not handle. Issues: a.)How would you approach this patient referred after trauma? b.) What studies would you like to get for initial evaluation assuming the patient is stable? c.) A duodenal perforation is noted on preop UGI, what would you do? How would you assess the common bile duct and the pancreatic duct? d.) How would you repair the duodenal perforation?

# 48 - Penetrating Trauma to RUQ with Retroperitoneal 18 y.o male with gunshot to RUQ presents in shock to the E.R.

Hematoma

ISSUES: a.) ABCs b.) What do you do? c.) Intraoperatively, you find a nonbleeding liver laceration, a hole in the second portion of the duodenum and retroperitoneal hematoma. How do you proceed with midline retroperitoneal hematoma- how do you get proximal and distal control? d.) how do you evaluate the pancreaticoduodenal complex? how would you manage injuries? How would you manage the duodenal perforation? e.) How do you manage injury to the IVC? ANSWER: Airway = for those comatose or obtunded.

Apneic = intubate orally. Apneic with facial trauma = emergency cricothyroidotomy. Breathing but need airway control = nasotracheal intubation. Breathing but combative = rapid sequence induction with cricoid pressure and in-line traction. Breathing = pulmonary ventilation is necessary. Confirm by physical exam - no ventilation can occur because of poor EET placement, hemothoroax, pneumothorax, aspiration of teeth and shit, pulmonary contusion. Circulation a.k.a. shock - 14 gauge IV X 2 - draw blood for HCT and T&C (and other labs) - Adults = 2L of LR. - Peds = 20cc/kg Response - those that improve and stabilize are unlikely to have ongoing bleeding and can be approached systematically. Those that do not improve or improve transiently and then deteriorate are likely to have ongoing hemorrhage and should be taken to OR. Secondary Survey - detailed physical exam. Rectal exam, Foley, NGT. CXR, pelvic X-ray. PENETRATING TRAUMA - EXPLORE BLUNT TRAUMA - difficult situation is identifying pancreatic injury in the absence of other indications for exploration (shock, peritonitis, + DPL). Many of these will remain asymptomatic for a long period of time. Generally associated with high energy impact to the epigastrium. AMYLASE - not a great indicator of pancreatic injury. Elevation without pain should prompt repeat measure. Development of pain and failure of the amylase to normalize should prompt investigation with CT scan or ERCP. Problem with CT is that it has only subtle signs of injury early. If first scan didnt show anything and symptoms persist, repeat. ERCP- good for hemodynamically stable patients with persistent elevated amylase and pain. Ideally this could be performed within the first 12 hrs of injury. Transected duct warrants surgery. INTRAOP ERCP - when injury to duct cannot be seen on careful examination. Can be done by duodenotomy, ampulla cannulation or by transection of the tail and distal duct cannulation. INTRAOP EVALUATION - signs warranting pancreatic evaluation: midline hematoma, edema around pancreatic gland and lesser sac, retroperitoneal bile staining. EXPOSURE - divide the gastrocolic ligament outside of the gastroepiploic vessels. Retract the stomach anteriorly and superiorly. To see the head and uncinate process, Kocherize the duodenum medially to the superior mesenteric vessels. Mobilization of the hepatic flexure of the colon helps. - to expose the tail for injury, perform a medial rotation of the colon, spleen and pancreas for bimanual palpation of the tail. PENETRATING TRAUMA - if the injury has not involved the substance of the pancreas, no ductal study is necessary. Duct study necessary if the substance of pancreas is injured. BLUNT TRAUMA - can have transected duct with the peritoneal covering intact or with the gland not completely transected. TECHNIQUE OF INTRAOP PANCREATOGRAPHY -

1.) 18 gauge angiocath in to gallbladder - give morphine to induce sphincter contraction and inject 20-30 cc of water soluble contrast. 2.) Duodenotomy and ampulla cannulation 3.) Tail of pancreas transection and duct cannulation 4.) Transected gland and duct - cannulate and get dye study of proximal duct to visualize the duct that will remain. Use 5 Fr. feeding tube and 2-5 cc of contrast under low pressure. TREATMENT OF PANCREATIC INJURY CONTUSION AND LACERATION WITHOUT DUCT INJURY these account for 80% of injuries. These should be treated with hemostasis and drainage with soft, closed suction drains. Attempt to close laceration are as likely to make the situation worse as better. If the amylase content of drainage is that of serum, remove drains at 24-48 hrs. If amylase is high, leave drains in until output is low. Nutrition - oral diet as soon as possible. If ileus persists because of pancreatic injury feed via preplaced J-tube with elemental formula. DISTAL TRANSECTION/DISTAL PARENCHYMAL INJURY WITH DUCT DISRUPTION - treat with distal pancreatectomy with or without splenectomy. Pancreaticogram via transected duct to assess remaining proximal duct. Normal duct - staple the pancreas with stapler, but individually close the duct with figure of eight Prolene, butress with omentum and place a drain. Abnormal proximal duct/stricture - distal resection of gland using either mattress sutures or stapler and internal drainage with Roux en Y limb of jejunum. ?Splenectomy - concern over OPSS must be balanced with the blood loss and time in trauma patient when considering splenic conservation. NUTRITION - PLACE J-TUBE BEFORE CLOSING. PROXIMAL TRANSECTION OR INJURY WITH PROBABLE DUCT DISRUPTION - important to perform pancreatography early in course. Head and Neck Injury without Duct injury - provide wide external drainage. Head and Neck injury, unstable patient, Duct injury unclear - wide drainage. Proximal Duct injury, ampulla+duodenum spared - pancreatectomy distal to duct injury with wide drainage of transected pancreas. - if the amount of pancreatic tissue remaining with this will leave patient with insufficiency, the tail of the pancreas can be preserved and anastomosed to a Roux en Y limb of jejunum. This is done less frequently because of complications and the generally good result with subtotal pancreatectomy. NUTRITION - PLACE J-TUBE BEFORE CLOSING.

COMBINED PANCREATICODUODENAL INJURY NEED: CHOLANGIOGRAM - can be performed through the gallbladder using 18 gauge needle and 20-30 cc water soluble contrast. EVALUATION OF AMPULLA PANCREATICOGRAM - may be done through the injury site or via tail of pancreas transection and duct cannulation. KEY -> IF CHOLANGIOGRAM SHOW FLOW OF CONTRAST INTO THE DUODENUM WITHOUT EXTRAVAZATION, THE DISTAL COMMONBILE DUCT AND AMPULLA ARE INTACT. THE DUODENAL INJURY AND PANCREATIC INJURY (BASED ON INJURY AND PANCREATICOGRAM) CAN BE TREATEDSEPARATELY PANCREATICOGRAM -if it cannot be obtained intraop and duct cannot be assessed, place multiple drains around the head rather than a total pancreatectomy. DIVERSION OF GASTRIC CONTENTS IN SEVERE COMNBINED DUODENAL AND HEAD OF PANCREAS INJURY A.) DUODENAL DIVERTICULIZATION - consists of primary closure of the duodenal wound, antrectomy, vagotomy, T-tube drainage of CBD, lateral tube duodenostomy. Idea is to divert gastric contents and biliary contents away from the duodenal repair, provide nutrition through the gastrojejeunostomy and provide a controlled lateral fistula. B.) PYLORIC EXCLUSION- easier procedure with less resection and no biliary drainage. Make a gastrotomy inferiorly on antrum along greater curve. Grasp pylorus with babcock and close pylorus with 0 suture and construct loop gastrojejunostomy. Diverts gastric flow away from duodenal injuries until they heal. The pylorus will open in 2 weeks to 2 months and the gastrojejunostomy will close. PANCREATICODUODENECTOMY - only for massive injuries of the duodenum and head of pancreas with destruction of ampulla with distal CBD or proximal pancreatic duct making reconstruction impossible. When indicated this procedure is really just debridement of devitalized tissue. Can be performed in situation where patient is stabilized from associated vascular injury. COMPLICATIONS: FISTULAS - more frequent with pancreaticoduodenal injuries. Most of these are minor (<200cc/day) and will spontaneously resolve in 2 weeks. High output failures (>700 cc/day) will take longer to close. I f the volume doesnt decrease in 10 days, get ERCP to evaluate pancreatic duct. Treat patient via preplaced J-tube or TPN. ABSCESSES - should be drained percutaneously or operatively as soon as possible. SECONDARY HEMORRHAGE - should be treated operatively. PSEUDOCYSTS - major determinant is the integrity of the pancreatic duct. If the duct is intact, percutaneous drainage will be successful. If the duct has been injured and the injury was missed with initial evaluation, percutaneous drainage will convert the pseudocyst into a

chronic fistula. ERCP is done before percutaneous drainage. If abnormal, surgery is necessary for partial gland resection. DUODENAL INJURIES DIAGNOSIS - should obtain a serum amylase when injury is suspected, though this is not specific for duodenal injury. A persistently elevated amylase or rising amylase should raise suspicion of pancreatic duodenal injury. If retroperitoneal duodenal rupture is suspected, it can be confirmed by CT scan with IV and PO contrast or an UGI with Gastrograffin (or Barium if Gastrograffin is negative). TREATMENT - 85% of injuries can be repaired with primary anastomosis. Overall morbidity and mortality is related to the severity of the injury. Mild injuries have 0% mortality and 2% fistula rate. Major injuries have 6% mortality and 10% fistula rate. DETERMINANTS OF SEVERITY MILD SEVERE Agent stab Blunt or missile Size <75% wall >75% wall Duodenal site 3, 4 1, 2 Time from injury to surgery <24 hrs. > 24 hrs. Adjacent Injury No CBD injury CBD injury Mild injuries = primary repair Severe injuries = more complex repair EXAMPLUS GRATIAS: DUODENAL LACERATION WITH NO PANCREATIC INJURY - primary repair with no diversion. DUODENAL TRANSECTION - mucosal debridement and primary repair can be used for all transection s except those involving the ampulla. - can also bring a Roux en Y limb to the proximal duodenal defect and closure of the distal duodenal defect. Of course this is more work. PPROTECTING PRIMARY DUODENAL REPAIR IN SEVERE INJURY 1.) Omental patch - data has not shown benefit. 2.) Serosal patch with loop of jejunum. - data has not shown benefit. 3.) Pyloric exclusion - see above 4.) Duodenal diverticulization - see above. - complications of gastric diversion is marginal ulceration at the site of the gastrojejunostomy. 5.) 3 Tubes - gastrostomy for decompression, jejunostomy proximally for decompression and distal feeding jejunostomy. ** regardless of type of diversion (3-5.), data shows that some type of diversion is beneficial and lowers mortality and fistula rate.

DUODENAL HEMATOMA - many will obstruct within 48 hrs. from fluid shifting into the hematoma. Diagnosis made by Gastrograffin swallow (followed by Barium for delineation of detail). Coiled spring sign or stacked coin signs. - MUST BE ABLE TO RULE OUT ASSOCIATED INJURIES, ESPECIALLY TO PANCREAS WHICH OCCURS 20%. - start NGT suction and TPN. OK if looks obstructed. If signs of obstruction persist, repeat upper GI at 5-7 days. -- explore at 2 weeks if conservative therapy has failed to yield results to look for contained perforation or injury to the head of the pancreas which would lead to obstruction. - INTRAOP - if duodenal hematoma is seen intraop, it should be explored via complete Kocher maneuver to look for perforation. This will often release a subserosal hematoma.

# 49 - Shotgun Injury to Flank 22 y.o. male is brought to E.R. after sustaining a shotgun blast at close range to his right flank. It is immediately apparent that the peritoneal cavity has been entered. His BP is 70/palp., heart rate = 150, resp rate = 42, anxious but alert and oriented. Issues: a.) Preoperative resuscitation, ABCs. b.) Preoperative workup, if any> c.) Intraoperatively the patient is found to have a complete transection of the ascending colon, multiple small bowel injuries from the shotgun pellets, a 6 cm segment of R mid ureter is missing, and there is a retroperitoneal hematoma just to the right of the midline which proves to be a caval injury. How would you go about dealing with these injuries? What are the options for repairing the damaged ureter in this setting? If the ureter is repaired primarily over a stent, and there is a leak postoperatively, what can be done nonoperatively to assist healing. # 50 - Gunshot Wound to Abdomen 22 y.o. male with gunshot wound to abdomen presents to E.R. hypotensive and unresponsive. Issues: a.) ABCs. b.) Evaluation of possible ureter injury. c.) Intraoperatively, you find a hole in the right colon with gross spillage and a large central hematoma. how would you get vascular control for exploration of this central hematoma? how does one control the vena cava proximally and distally? ANSWER: URETERAL INJURY KEY - PRIMARILY AN INJURY OF PENETRATING TRAUMA.

DIAGNOSIS - via IVP showing cutoff or extravasation. However, IVP is negative 15% of time and diagnosis should be made via retrograde ureterogram. However, many instances present with unstable trauma and do not have the time for a preoperative IVP. - with penetrating trauma, often a ureteral injury can be ruled out based on the path of the penetrating object. However, this may be difficult in the case of a massive retroperitoneal hematoma. Get an intraop IVP. Dissecting out entire ureter to look for injury is not a good idea. - use IV methylene blue to look for extravasation from the ureter. TREATMENT SHORT OR NO SEGMENT LOSS - primary ureteroureterostomy with stenting or reimplantation into the bladder. LONG SEGMENT LOSS Lower Injury - reimplantation into the bladder via a psoas hitch or bladder pedicle flap. May also try a Transureteroureterotomy Middle - may try a Transureteroureterostomy. Upper - either an autotransplantation of the kidney or bowel interposition. Missed Injuries - diagnose with IVP or retrograde urethrogram. COLON AND RECTAL TRAUMA DIAGNOSING COLON AND RECTAL TRAUMA PENETRATING TRAUMA Gunshot Injury - 85% organ injury rate. Mandatory laparatomy. Stab Injury - 33% organ injury rate. Selective laparotomy - Lateral flank injuries should be highly suspicious for colon injuries. These patients do not need to be mandatory explored. May be evaluated by serial physical exams or with contrast enhanced CT scans if physical examination can not be performed reliably. - Anterior abdo wall stab wound. Do not mandate exploration. Should be followed with physical examination if possible. Those showing signs of persistent bleeding or those with peritoneal signs should be operated on. Those who cannot be followed with physical examination should have diagnosis aggressively pursued. Problem is that RBC limit for positive DPL is not agreed upon for stab wounds. CT scan is not reliable. Favor laparoscopy or formal laparotomy if colonic injury cannot be ruled out. RECTAL INJURY - look for gross blood (100%reliable). Otherwise proctoscopy in ER. BLUNT TRAUMA - difficult because associated head injuries make reliable physical exam difficult. CT scan has several diagnostic and suggestive CT criteria for colon perforation.

Takes small amount of blood (20 cc) to make a DPL positive - DPLs can lead to high rate of negative laparotomies. PREOPERATIVE PREPARATION - MUST HAVE PROPHYLACTIC IV ANTIBIOTICS. In several studies, blah, blah, blah, blah. Use single coverage with cefotetan or cefoxitin, or triple coverage with ampicillin, aminoglycoside/third generation cephalosporin and metronidazole. TREATMENT OF COLON INJURIES RESECTION REASONS TO RESECT: 1.) Preop shock (BP< 60 mmHg) 2.) Severe hemmorhage (> 1L) 3.) Injury to more than 2 organ systems 4.) Signficant fecal contamination. 5.) Operation starting 8 hrs after injury. 6.) Severely destructive colon wounds. 7.) Loss of abdominal wall requiring g mesh. Technique - resect to normal bowel, remove bowel close to mesentery to avoid vascular injury. No need for wide resection. Suture ligatures in vascular structures. Use stapler to divide bowel. Proceed to anastomosis or stoma. PRIMARY ANASTOMOSIS Technique - use either a stapled side to side or 2 layer hand sewn anastomosis (though this takes more time.) COLOSTOMY - should be used whenever there is concern about an anastomosis failing. Many of the reasons for failure of anastomosis are the same as reasons to resect rather than close (preop shock, severe hemorrhage, fecal contamination, multiple organ system injuries, operation long after injury). Can be end or loop colostomy. - decision of end colostomy as Hartmanns procedure or mucus fistula depends on situation. In situations in which there is hemodynamic compromise, it may make more sense to do a Hartmanns procedure and delayed maturation of the colostomy. In a hemodynamically stable patient, a mucus fistula will make the takedown much easier and should be performed. LOOP COLOSTOMY - for similar indications as end colostomy (protect distal injury site) but can be performed more quickly. Areas where the loop colostomy can be created are areas where the colon can be brought out to the abdominal wall with the mesentery sufficiently loose such that there is no tension (most frequently the transverse and sigmoid colon.) to ensure a complete diversion the distal loop may be stapled. Ostomy can be matured later if the patient is unstable. PRIMARY REPAIR - in general, the situations in which resection should be performed are also reasons why a primary repair should not be performed (preop shock, severe hemorrhage, fecal contamination, multiple organ injuries, operation long after injury). - in blunt trauma, the amount of energy needed to create a colon injury will usually create a colon submucosal hematoma as well as a mesenteric hematoma. These severe injuries would obviously need resection and favor ostomy.

RECTAL INJURIES - PROXIMAL DIVERSION WITH COLOSTOMY. If rectal injury occurs above the peritoneal reflection should be repaired primarily with proximal diversion. Those below the peritoneal reflection are difficult to repair and often associated with pelvic hematoma which should not be released. Presacral drainage with closed suction drains exiting the perineum are key to treating injury. - important to lower the infection rate of related hematoma and pelvic fracture by irrigating out the rectum -> leave skin open. Complications = fecal fistula through wound or entrance site.

# 51 - Penetrating Trauma to Chest 20 y.o. male is stabbed in the left anterior chest and is brought to the ER with a BP of 40 systolic. Issues: a.) ABCs. b) Patient goes into V-Tach and V.Fib. What do you do? c.) Is an ER thoracotomy warranted in this situation? d.) Is an ER thoracotomy indicated in the presence of intractable hypotension in a patient with blunt trauma? 45 y.o. male is stabbed in the left anterior chest and is brought to the ER hypotensive and tachycardic. Issues: a.) ABCs b.) Pt. remains hypotensive and tachycardic despite fluid boluses. What intervention would you perform? c.) Chest tube output is minimal but patient remains hypotensive and tachycardic. What intervention would you perform? d.) Pericardiocentesis shows nonclotting blood, what would you do? e.) Intraoperatively, pt has transection of LAD. What would you do? Airway = for those comatose or obtunded. Apneic = intubate orally. Apneic with facial trauma = emergency cricothyroidotomy. Breathing but need airway control = nasotracheal intubation. Breathing but combative = rapid sequence induction with circled pressure and in-line traction.

Breathing = pulmonary ventilation is necessary. Confirm by physical exam - no ventilation can occur because of poor EET placement, hemothoroax, pneumothorax, aspiration of teeth and shit, pulmonary contusion. Circulation a.k.a. shock - 14 gauge IV X 2 - draw blood for HCT and T&C (and other labs) - Adults = 2L of LR. - Peds = 20cc/kg - check CVP. - EKG. Response - those that improve and stabilize are unlikely to have ongoing bleeding and can be approached systematically. Those that do not improve or improve transiently and then deteriorate are likely to have ongoing hemorrhage and should be taken to OR. Secondary Survey - detailed physical exam. Rectal exam, Foley, NGT. CXR, pelvic X-ray. PRESENTATION - may present stable or in complete cardiac arrest. ER THORACOTOMY Prehospital factors predicting poor outcome in patients with trauma to the thoracic vasculature/heart - absence of vital signs (before hospital), fixed and dilated pupils, absence of cardiac rhythm, absence of motion in extremities, absence of palpable pulse, cardiopulmonary arrest. INDICATIONS FOR ER THORACOTOMY 1.) CARDIOPULMONARY ARREST SECONDARY TO PENETRATING THORACIC TRAUMA. 2.) SBP < 60 SECONDARY TO EXANGUINATING HEMORRHAGE OR PERICARDIAL TAMPNADE. CONTRAINDICATION TO ER THORACOTOMY 1.) CARDIOPULMONARY ARREST SECONDARY TO BLUNT TRAUMA OBJECTIVES OF ER THORACOTOMY - resuscitate agonal patients with penetrating thoracic trauma, evacuation of pericardial tamponade, control hemmorhage from massive cardiovascular injury, clamping of descending aorta to redistribute blood to the carotids and coronaries, control of exsanguinating thoracic vascular injuries, intracardiac massage. TECHNIQUE - left arm elevated, thorax prepped widely. Incision made on left from the sternocostal border beneath the nipple to the latissimus dorsi. Incision carried through the skin, subcutaneous tissue and serratus anterior until the intercostal muscles have been reached. Divide the intercostal muscles with scissors. Finochietto retractor is put in place and ribs separated. The left lung is elevated medially and the aorta is localized, it can be compressed against the thoracic vertebral bodies. Aorta can be cross clamped by dissecting the aorta circumferentially with index finger and thumb away from esophagus. - next observe the pericardium which is likely to be bluish and tense. Locate the phrenic nerve. The pericardial sac is opened anterior to the phrenic nerve and extended inferiorly and

superiorly. (Make a small incision in the pericardium with knife, grasp the edges with Allis clamps and elevate, divide with scissors.) - evacuate clot in the pericadial sac. Examine heart to determine the rhythm, the site of injury, and the blood volume remaining in the cardiac chambers. A flaccid, empty heart is a bad prognostic sign, as is empty coronary arteries and air in the coronary veins. TREATMENT OF INJURIES IN ER: VENTRICULAR INJURIES - place finger over hole and oversew with 2.0 prolene. ATRIAL INJURIES - control with partially occluding Satinsky clamp LARGE INJURIES - occluse with the balloon of a Foley catheter until definitive cardiorrhaphy can be performed in OR. Do not attempt to use Teflon or pledgets as it takes too much time. - may need to shock heart after repair to establish sinus rhythm with 20-50 joules directly to the heart. TECHNIQUES OF CARDIAC INJURY REPAIR INCISIONS: ATRIAL WOUNDS - control with Satinsky clamp. Close wound with running 2.0 prolene suture. Running vertical mattress with an over and over baseball stitch. VENTRICULAR WOUNDS - place finger over wound and close with simple interrupted or horizontal mattress sutures of 2.0 prolene. Often as the compromised myocardium becomes more friable, the sutures will tear through creating a larger bleeding defect. This is when biosynthetic materials such as Telfon are needed to buttress the suture line. CORONARY ARTERY INJURIES - proximal injuries will require cardiopulmonary bypass to repair. Distal third injuries should be ligated with a U stitch. In desperation, proximal injuries may be ligated, though this will cause infarction and need for aortocoronary bypass and/or IABP. ACUTE INDICATIONS FOR THORACOTOMY 1.) Cardiac tamponade. 2.) Acute deterioration - cardiac arrest (ER thoracotomy). 3.) Vascular injury at the thoracic outlet. 4.) Loss of chest wall substance. 5.) Massive air leak in chest tube. 6.) endoscopic or tracheal injury by radiography or endoscopy.

7.) Radiographic evidence of great vessel injury. 8.) Penetrating injury through the mediastinum 9.) bullet emboli to the heart or pulmonary artery. 10.) Massive (>1500 cc) or continuing (>250 cc/hr). CHOICE OF INCISIONS: 1.) LEFT ANTEROLATERAL THORACOTOMY - should be used for resuscitation under circumstances of acute deterioration or cardiac arrest. Right anterolateral thoracotomy has little use in trauma. 2.) TRANSSTERNAL ANTERIOR THORACOTOMY - an extension of the anterolateral thoracotomy across the sternum (sort of a clam shell incision) - this allows better exposure to the heart. Will need to ligate the internal mammaries. 3.) POSTERLATERAL THORACOTOMY - better access to the lower lung, esophagus and descending aorta. Also allows exposure to the proximal left subclavian artery on left. Right posterolateral thoracotomy - good exposure to the trachea, proximal esophagus or pulmonary injury. Good exposure to SVC and IVC. 4.) Book or Trapdoor incision - for exposure to left sided thoracic outlet injuries. Allows exposure to the long segment of the left subclavian artery and left common carotid. Composed of a left anterolateral thoracotomy + a median sternotomy with saw + incision above clavicle (with SCM if necessary). This incision stretches brachial plexus and is related to complications. Use only if absolutely necessary. Third interspace left anterolateral thoracotomy with a clavicular incision is a better choice for proximal control of left subclavian artery injury. 5.) MEDIAN STERNOTOMY - limited use in trauma - only for injuries to the anterior hear or proximal great vessels. Must be sure that these are the only injuries that need to be fixed. Incision of choice for ascending aorta and innominate artery injuries. # 52 - Penetrating Trauma to Base of Neck/Chest 25 y.o. male who is stabbed in the left neck in Zone I just above the clavicle. Pt brought in without external bleeding to the ER. 20 y.o. male involved in MVA with isolated injury to right chest. Arteriogram shows a pseudoaneurysm of the proximal right subclavian artery. Issues: a.) ABCs b.) What tests would you want for evaluation? c.) Angiogram shows a tapering of the distal L subclavian artery. What would you do? How would you expose the subclavian artery? how would you obtain proximal and distal control? how would you repair a laceration? What would you do if a primary repair gives you a 50% stenosis? d.) What would you do with a subclavian vein injury? e.) What are the physical findings of an inferior cord brachial plexus injury? How should this be treated? f.) Anatomically, what are the branches of the subclavian artery?

Airway = for those comatose or obtunded.

Apneic = intubate orally. Apneic with facial trauma = emergency cricothyroidotomy. Breathing but need airway control = nasotracheal intubation. Breathing but combative = rapid sequence induction with circled pressure and in-line traction. Breathing = pulmonary ventilation is necessary. Confirm by physical exam - no ventilation can occur because of poor EET placement, hemothoroax, pneumothorax, aspiration of teeth and shit, pulmonary contusion. Circulation a.k.a. shock - 14 gauge IV X 2 - avoid arm IVs especially on side of injury. If suspected innominate, subclavian injury put the IV in the leg. - draw blood for HCT and T&C (and other labs) - ABG. - UA - Adults = 2L of LR. - Peds = 20cc/kg - check CVP. - EKG. Response - those that improve and stabilize are unlikely to have ongoing bleeding and can be approached systematically. Those that do not improve or improve transiently and then deteriorate are likely to have ongoing hemorrhage and should be taken to OR. Secondary Survey - detailed physical exam. Rectal exam, Foley, NGT. CXR, pelvic X-ray, UA MECHANISM : PENETRATING - GSW, stab traversing the upper chest or the base of the neck. May also have penetrating injury beneath the clavicle. BLUNT - severe deceleration injuries. PREOPERATIVE EVALUATION: - HCT, UA, ABG, CXR, EKG. - IV ABX. ARTERIOGRAM - BLUNT TRAUMA - = strong history of mechanism and widening of the superior mediastinum more than 8 cm, loss of aortic knob, multiple left rib fractures, pleural cap or left pleural effusion indicates retrograde femoral arteriography to detect site of aortic tear. Site of injury at the ligamentum arteriosum distal to the origin of the left subclavian artery. - PENETRATING = proximity of projectile or penetration to vascular structures should prompt arteriography in cases of suspected innominate, carotid and subclavian artery only. All other great vessel injuries will usually present with hemodynamic instability and severe hemorrhage.

- for subclavian, innominate or carotid injury, arteriogram is critical prior to OR for operative planning. OPERATIVE MANAGEMENT PREP - chin to knees, supine. IN EXTREMIS WITH THORACIC OUTLET INJURY - LEFT ANTERIOR THORACOTOMY WITH EXTENSION TO THE LEFT IF NECESSARY. If the incision is extended, it should be extended to an upper interspace to enhance exposure for proximal control. Obtain proximal control. - after proximal control is obtained, if the incision has not been extended across the sternum, it can be extended via median sternotomy to the neck. NON EXTREMIS THORACIC ARTERY INJURY - HIGH LEFT ANTERIOR THORACOTOMY IN THIRD INTERSPACE. The proximal subclavian can be controlled here. The distal control is obtained via a supraclavicular incision away from the hematoma. This avoids using a supraclavicular incision for exposure and getting into hematoma without proximal control and killing the patient. ?remove clavicle - only in life saving situation, morbid procedure. Distal injuries may be approached from a supraclavicular incision away from the hematoma for proximal control. - those that cannot be repaired should have interposition Dacron graft placed. RIGHT SUBCLAVIAN ARTERY INJURY - use median sternotomy with right cervical extension for proximal control INNOMINATE ARTERY INJURY - should be approached via a median sternotomy. An extension up the right neck may be necessary. BLUNT TRAUMA - usually involves the proximal innominate artery. PENETRATING TRAUMA - usually involves the distal innominate artery. - repair using lateral arteriorrhaphy with Satinsky and running 4-0 prolene if possible for a partial circumferential tear. - if not possible, need to bypass from the ascending aorta to the distal innominate using a 10 mm Dacron bypass graft. The ascending aorta is clamped with a Satinsky clamp and the proximal anastomosis is performed. The distal innominate is dissected out and divided. The distal anastomosis from the graft to the divided distal innominate is performed end to end. Before the hematoma is entered and the injury is addressed. The injury is repaired. No anticoagulation or bypass needed. INNOMINATE VEIN INJURY - can be ligated with impunity.

PEDIATRIC TRAUMA
# 53 - Pediatric Duodenal Hematoma 10 y.o. boy fell over the handle bars of a bicycle 12 hrs ago. Initially, he had minimal pain and was brought in for persistent nausea and vomiting and midepigastric pain of modest

proportion. Physical examination shows only a moderate midpigastric tenderness. Vitals and labs are normal with exception of mildly elevated amylase. Issues: a.)Initial care and differential diagnosis. b.) Diagnostic tests of choice. c.) Assuming the UGI shows near complete obstruction of the duodenum and CT scan shows a intramural duodenal hematoma near the head of the pancreas, what is the diagnosis and treatment. d.) Is TPN indicated, if so when? e.) What is the likelihood that this will resolve without surgery? How long would you wait before intervening surgically? If you did intervene surgically, what would you do? If you created a defect in the duodenum, how would you repair it? f.) Supposing that the initial diagnosis was duodenal hematoma by UGI (Note - CT scan important for ruling out a central hematoma) and conservative treatment is chosen - what would you do if n the third postop day the patient develops fever, worsening abdominal pain and CT scan shows a central fluid collection consistent with a pancreatic injury - what would you do? How would deal with the pancreatic duct after performing a distal pancreatectomy and splenectomy? (Duodenal hematoma may cause pancreatic duct obstruction and cause a pancreatic duct leak) Potential Pitfall - duodenal defect and open pancreatic duct - both requiring Roux en Y limbs for drainage. ANSWER: DUODENAL INJURIES DIAGNOSIS - should obtain a serum amylase when injury is suspected, though this is not specific for duodenal injury. A persistently elevated amylase or rising amylase should raise suspicion of pancreatic duodenal injury. If retroperitoneal duodenal rupture is suspected, it can be confirmed by CT scan with IV and PO contrast or an UGI with Gastrograffin (or Barium if Gastrograffin is negative). INTRAOP DIAGNOSIS: bile staining of retroperitoneum, periduodenal or peripancreatic hematoma, crepitus or gas bubbles in retroperitoneum, saponification of retroperitoneal tissues. TREATMENT - 85% of injuries can be repaired with primary anastomosis. Overall morbidity and mortality is related to the severity of the injury. Mild injuries have 0% mortality and 2% fistula rate. Major injuries have 6% mortality and 10% fistula rate. DETERMINANTS OF SEVERITY MILD SEVERE Agent stab Blunt or missile

Size <75% wall >75% wall Duodenal site 3, 4 1, 2 Time from injury to surgery <24 hrs. > 24 hrs. Adjacent Injury No CBD injury CBD injury Mild injuries = primary repair Severe injuries = more complex repair EXAMPLUS GRATIAS: DUODENAL LACERATION WITH NO PANCREATIC INJURY - primary repair with no diversion. (CLASS I) DUODENAL TRANSECTION - mucosal debridement and primary repair can be used for all transection s except those involving the ampulla. - can also bring a Roux en Y limb to the proximal duodenal defect and closure of the distal duodenal defect. Of course this is more work. (CLASS II). DUODENAL INJURY WITH MINOR PANCREATIC INJURY (no injury to the main pancreatic duct) - close the duodenal defect and utilize either triple tube, duodenal diverticulization or pyloric exclusion.(CLASS IV). DUODENAL INJURY WITH MAJOR PANCREATIC INJURY - use triple tube, duodenal diverticulization or pyloric exclusion if possible. If not possible, perform Whipple. PPROTECTING PRIMARY DUODENAL REPAIR IN SEVERE INJURY 1.) Omental patch - data has not shown benefit. 2.) Serosal patch with loop of jejunum. - data has not shown benefit. 3.) Pyloric exclusion - see above 4.) Duodenal diverticulization - see above. - complications of gastric diversion is marginal ulceration at the site of the gastrojejunostomy. 5.) 3 Tubes - gastrostomy for decompression, jejunostomy proximally for decompression and distal feeding jejunostomy. ** regardless of type of diversion (3-5.), data shows that some type of diversion is beneficial and lowers mortality and fistula rate. DUODENAL HEMATOMA - INTRAMURAL DUODENAL HEMATOMA - collection of blood in the subserosal layer of the duodenal wall. Severity of the hematoma may make the viability of the duodenal wall questionable. Often affects the second and third portion of the duodenum and may extend proximally to the first or distally to the fourth. - because these often present after the trauma 24 hrs - 2 weeks, it is likely related to the shift of fluid or osmotic forces of the blood in the wall which can cause complete obstruction. - because those that present with isolated duodenal injury usually present with painless obstruction, those that present immediately after trauma (as opposed to delayed presentation) with abdominal pain and tenderness are more likely to have associated injuries as opposed to isolated duodenal hematoma. -many will obstruct within 48 hrs. from fluid shifting into the hematoma. Diagnosis made by Gastrograffin swallow (followed by Barium for delineation of detail). Coiled spring sign or stacked coin signs. - MUST BE ABLE TO RULE OUT FULL THICKNESS PERFORATION AND ASSOCIATED INJURIES, ESPECIALLY TO PANCREAS WHICH OCCURS 20%.

- start NGT suction and TPN. OK if looks obstructed. If signs of obstruction persist, repeat upper GI at 5-7 days. ADULTS-- explore at 2 weeks if conservative therapy has failed to yield results to look for contained perforation or injury to the head of the pancreas which would lead to obstruction. PEDIATRICS - trial nonoperative period of 5-10 days. If the patient has not shown at least partial resolution at 5 days or complete resolution with resumption of oral intake at 10 days. - INTRAOP - if duodenal hematoma is seen intraop, it should be explored via complete Kocher maneuver to look for perforation. This will often release a subserosal hematoma. It should be gently unroofed or evacuated to ensure that the duodenal wall is intact. DRAINAGE OF HEMATOMA - TECHNIQUE - short longitudinal incision is made over the lateral aspect of the duodenum over the hematoma. Hematoma is evacuated with irrigation and sponges. Careful not to injure the mucosa (if question of a defect, use methylene blue via NGT) - if the hemotoma extends to the fourth portion of the duodenum, the incision should be extended to evacuate the hematoma. Specific bleeding points should be ligated. Close the proximal portion of the incision with interrupted sutures, but leave the midportion open to promote drainage and prevent reformation of the hematoma. Leave drain.

# 54 - Pediatric Splenic Trauma/ Head Injury 8 y.o. male with left chest and abdominal trauma. Broken ribs 10,11,12 on left. BP=80/60 with a pulse of 130. Issues: a.) ABCs and initial fluid resuscitation (how much for a bolus) b.) Pt. stabilizes after fluid giving enough time for CT scan which shows a splenic fracture and subcapsular hematoma. Pt. then become sunstable, what do you do? Would you transfuse? If so, how much? c.) If you are forced to remove the spleen, how would you treat this patient long term (oral antibiotics) and with vaccine? d.) During the procedure the patient blows a pupil, how would you perform a Burr Hole? CRANIOCEREBRAL INJURIES A = Airway - unconscious children should be intubated. Prevents hypoxic injury on top of traumatic injury. Intubate orally if possible, otherwise cricothyroidotomy followed by tracheostomy.

Choosing ETT - can be approximated by choosing a tube that approximates the size of the nares or the fifth digit. Use uncuffed tube. Dont nasotracheally intubate. Cricothyroidotomy - use a 12 or 16 gauge angiocath and institute jet ventilation. B = Breathing - check ventilation. C = Circulation - estimated blood volume in child is 80mL/kg. (10 kg kid = 800cc total blood volume.) Hypotension is a late manifestation of shock and need to recognize other signs such as tachycardia, disorientation, tachypnea, decreased capillary filling. IVs = need 2 peripheral IVs, should be 20 gauge or larger. Preferred sites are the cephalic (antecubital fossa), greater saphenous, or external jugular. Interosseus line - can be used to children up to 6 years. Place 2-3 cm below the tibial tuberosity. Can be used as an emergency line until peripheral line can be established. FIRST BOLUS = 20 ML/KG LR if hypotensive SECOND BOLUS = 20 mL/KG LR if no response to first bolus. THIRD BOLUS = 10 mL/KG of type O/Rh (-) or type specific blood. If third bolus is not successful, take patient to OR. - CHILD IN COMA FROM TRAUMA SHOULD BE TREATED AS AN EMERGENCY AND INVESTIGATED AND EVALUATED AS IF A SPACE OCCUPYING LESION WERE PRESENT. ASSESSING NEUROLOGIC FUNCTION LEVEL OF CONSCIOUSNESS - GCS MOTOR COMMANDS = (obeys = 5, extensor response = 1) EYE OPENING = (spontaneous =5, nil = 0) VERBAL RESPONSE = (norm al = 5, nil = 0) Normal = 15 Severe Brain Injury = 8 Brain Dead = 3. - need to add pupil exam to GCS to get sufficient neuro exam. INITIAL ASSESSMENT AND TREATMENT - if abnormality on exam is noted, measure to control ICP should be instituted. 1.) HYPERVENTILATION - for a PaCO2 of 25-30 mmHg. 2.) ELEVATE THE HEAD 3.) OSMOTICS - 0.25-2.0 gm/kg (1.5 gm/kg good estimate) RADIOGRAPHIC EVALUATION - CT scan of the head is the preferred test - any child who is not normal after trauma gets a head CT. Any child with suspicion of neuro injury should have CT. If the CT is normal, a repeat is indicated for any neuro deterioration because of delayed hemtoma, swelling. - if no time for CT because of hemodynamic instability, take to OR for air ventriculogram to look for mass lesion. - if patient is deteriorating rapidly and there is no time for CT, take to OR for burr hole or craniotomy

- mass lesion diagnosed, take measures to control ICP and then take patient toOR to control mass lesion. Who gets ICP monitor - those who are unconscious before surgery and those that are in coma but do not have mass lesion. INTRACRANIAL PRESSURE NORMAL = 10 mmHg or less. ELEVATED, BUT NOT DETRIMENTAL = 10-20 mmHg. ELEVATED, NEED TREATMENT = greater than 20 mmHg. NEED ICP = lower than 25 mmHg. GUIDELINES FOR TREATMENT - if neurologic changes/deterioration occur at any ICP, it should be treated. If no changes occur at any ICP, keep ICP lower than 25 mmHg. SPECIFIC INDICATIONS EPIDURAL HEMATOMA - occur because of a tear in a branch f the middle meningeal artery in dura, appear as lens shaped mass on CT scan. Can present with lucid interval and then deteriorate or present with coma from head trauma with certain neuorlogic status and then deteriorates. Can also have small epidural which do not show mass effect or shift and these dont require surgical treatment.(This is the only situation in which an epidural does not require immediate surgery.) SUBDURAL HEMATOMA - need to separate the acute from the chronic subdural hematoma. These occur from tearing of bridging vein of sagittal sinus, direct trauma with tearing of cortical vein or tearing of sinus. - symptoms of an acute subdural are related to the parenchymal injury and the mass effect of the clot. Treatment depends on the clinical situation and the size of the clot. If the patient is thought to have severe neuro dysfunction from a clot, should have craniotomy to remove clot. However, if there is only a small rim of clot and patients neuro status is stable, it is likely that dysfunction is related to parenchymal injury and unlikely that removal of a small amount of clot will have much effect. SURGICAL DECOMPRESSION - treatment for epidurals and acute subdurals is surgical decompression as rapidly as possible. Timing is critical, later than 4 hrs. leads to decrease in outcome. Should be performed immediately in those who develop symptoms of herniation in face of maximal medical therapy and clinical evidence of a mass lesion (dilated pupil on one side) - usually controlling ICP with hyperventilation and osmotics can temporize until craniotomy can be performed. If not a temporoparietal burr hole needs to be performed. LOCATION - 1.5 - 2 finger breadths anterior and superior to the ear on the side of the mass lesion and the blown pupil.

SPLENIC TRAUMA IN PEDIATRIC AGE GROUP - GOAL OF TREATMENT IN CHILDREN IS TO SALVAGE SUFFICIENT SPLENIC TISSUE TO PRESERVE IMMUNE COMPETENCE. CLASSIFICATION OF SPLENIC INJURIESGRADE DESCRIPTION I Localized capsular disruption or subcapsular hematoma, no significant parenchymal injury. (HEMATOMA) II Single or multiple capsular or parenchymal disruption, transverse or longitudinal that do not extend to the hilar vessel. May or may not be intraparenchymal hematoma (MINOR - PARENCHYMAL INJURY) III Deep fractures, single or multiple, transverse or longitudinal, extending into hilar area and involving major segmental blood supply. (MAJOR - FRACTURE THROUGH PARNECHYMA TO HILUM) IV Shattered or fragmented spleen or spleen separated from normal vascular pedicle (SHATTERED SPLEEN) INITIAL EVALUATION AND RESUSCITIATION: A= Airway - unconscious children should be intubated. Prevents hypoxic injury on top of traumatic injury. Intubate orally if possible, otherwise cricothyroidotomy followed by tracheostomy. Choosing ETT - can be approximated by choosing a tube that approximates the size of the nares or the fifth digit. Use uncuffed tube. Dont nasotracheally intubate. Cricothyroidotomy - use a 12 or 16 gauge angiocath and institute jet ventilation. B = Breathing - check ventilation. C= Circulation - estimated blood volume in child is 80mL/kg. (10 kg kid = 800cc total blood volume.) Hypotension is a late manifestation of shock and need to recognize other signs such as tachycardia, disorientation, tachypnea, decreased capillary filling. IVs = need 2 peripheral IVs, should be 20 gauge or larger. Preferred sites are the cephalic (antecubital fossa), greater saphenous, or external jugular. Interosseus line - can be used to children up to 6 years. Place 2-3 cm below the tibial tuberosity. Can be used as an emergency line until peripheral line can be established. FIRST BOLUS = 20 ML/KG LR if hypotensive SECOND BOLUS = 20 mL/KG LR if no response to first bolus. THIRD BOLUS = 10 mL/KG of type O/Rh (-) or type specific blood. If third bolus is not successful, take patient to OR. HEMODYNAMICALLY UNSTABLE WITH NO OTHER INJURY ----------> OR. HEMODYNAMICALLY UNSTABLE WITH OTHER INURIES (PELVIS, LONG BONE FRACUTRES -------------------------------------------------------> DPL

HEMODYNAMICALLY STABLE -------------------------------------------------> CT DIAGNOSIS LABS - Hct/Hgb are often normal even in the face of arterial hypotension, not always useful. - base deficit of - 3 on ABG is indicator of ongoing hemorrhage. DPL - still useful for quick evaluation of abdominal visceral injury in the hemodynamically unstable patient. However, particularly in the pediatric population when nonoperative management of splenic injuries plays such an important role, blood in the abdomen is not necessarily an indication for laparotomy. CT Scan - offers qualitative (which organs) and quantitative (how bad) assessment of injury. TREATMENT NONOPERTIVE TREATMENT CRITERIA FOR CANDIDATE FOR NONPERATIVE TREATMENT 1.) Isolated splenic injury. 2.) Hemodynamically stable. 3.) Alert enough to allow repeated, serial physical examinations. IF A CANDIDATE - place in ICU for monitoring and obtain CT scans at 3 and 7 days. Perform serial abdominal exams. NPO and ready for OR. Monitor HCT q6hrs and vital signs. Child will likely need transfusions to maintain hemodynamic stability. Should transfuse to a Hct of 20-25%. Child remains in ICU until need for transfusion is abated. HOW MUCH BLOOD - abort for need to transfuse 50% of blood volume in first 24 hrs. Must set limit for transfusion at outset. FAILURE OF NON0PERATIVE TREATMENT - hemodynamic instability or transfusions exceeding preset limit. OPERATIVE THERAPY Position - supine Incision - upper midline. - on entering, feel the suprarenal aorta to assess arterial perfusion. If bleeding from liver and spleen, pack these. If circulatory embarrassment persists, take the spleen out. if patient stabilizes and splenic salvage can be performed, mobilize and examine the spleen. if the spleen is shattered and must be removed, the splenic artery can be temporarily compressed on the vertebral column to prevent ongoing blood loss while spleen is removed.

Mobilization - lift with right hand, place left over right with sponge and elevated with left hand to expose the lienorenal and phrenicolienal ligaments. Incise ligaments from caudad to cephalad. Right hand is then placed over left and fingers of right hand used to develop plain between the left kidney anterior surface and the posterior surface of the tail of the pancreas. Place 2 laps in LUQ. Examination - remove clots from the surface, but do not remove clots from within the edges of the fracture (this will start bleeding again.) If bleeding does recur, compress the hilar vessels with fingers or clamp. Unless the spleen is shattered or removed from its blood supply, splenic salvage via splenorrhaphy or partial splenectomy can be performed. Options for Splenic salvage 1.) Application of omentum or a topical hemostatic agent, with or without simple capsular sutures. 2.) Direct repair of splenic parenchyma and capsule with simple or mattress sutures, with or without pledgets of omentum. 3.) Suture ligation of individual splenic vessels. 4.) Partial splenctomy 5.) Capsular, parenchymal and vessel compression with application of mesh (vicryl or Marlex) 6.) Large through and through mattress sutures perpendicular to the plain of injury. POSTOPERATIVE MANAGEMENT: THROMBOCYTOSIS - reserve prophylactic anticoagulation for platelet counts in excess of 1 million. VACCINATION - Pneumovax should be given, the timing is controversial and probably unimportant. ANTIBIOTICS - should get oral PCN, especially those under the age f 2 yrs. This should continue until t hey are old enough to understand the significance of the problem and have the ability to seek medical attention for the signs and symptoms of impending infection. Antibiotics are used after this therapeutically if there are signs of impending infection, even if trivial.

# 55 - Airway Management in Head and Neck

Trauma
15 y.o. male presents with massive head and neck injuries after a MVA. Issues: a.) How would you establish the airway? b.) What would you do if an oral airway was not possible? What size tube would you use? How would you estimate the appropriate size tube. c.) Assuming the patient had a larynx fracture and had a trach placed in ER, which no longer functions, what would you do? d.) How do you deal with a laryngeal fracture? e.) Shortly after the placement of the trach, the patient has severe bleeding from the trach site. What would you do? f.) how would you repair a tracheoinnominate fistula? ANSWER:

A= Airway - unconscious children should be incubated. Prevents hypoxic injury on top of traumatic injury. Intubate orally if possible, otherwise cricothyroidotomy followed by tracheostomy. Choosing ETT - can be approximated by choosing a tube that approximates the size of the nares or the fifth digit. Use uncuffed tube. Dont nasotracheally intubate. Cricothyroidotomy - use a 12 or 16 gauge angiocath and institute jet ventilation. ABSOLUTE INDICATIONS FOR INVASIVE AIRWAY MANAGEMENT : ACUTE AIRWAY OBSTRUCTION - direct laryngeal trauma, especially when associated with fractures of the thyroid cartilage, produce submucosal hemorrhage and rapidly progressive edema. Can present with life threatening compromise of an airway when little evidence of external trauma exists. - expanding neck hematomas from arterial lacerations can cause obstruction from deviation of the larynx and from mechanical compression of the larynx. - transected airway occurs at the cricotracheal junction. The trachea retracts into the lower neck and the surrounding tissue provide a pseudo - airway. External pressure on the neck can cause this to collapse. APNEA - usually related to cerebral or spinal cord injury (above C4). HYPOXIA EXPANDING HEMATOMA OF THE NECK CRICOTHYROIDOTOMY - indicated for any patient who has the indication for intubation but the trachea cannot be intubated for some reason. Relatively contraindicated in pediatric age group (those under 12 years of age) because of the risk of subglottic stenosis. TECHNIQUE - palpate the cricothyroid membrane, stabilize larynx. Make a transverse incision over the membrane. Locate the membrane with finger and make a small incision. bluntly open with hemostat and insert a standard 8 mm outside diameter tube. TRACHEOSTOMY - indicated for patient with acute laryngeal trauma in whom placement of a tube through the cricothyroid space can contribute to the patients existing injury and in those patients under 12 yrs. in whom a cricothyroidotomy is contraindicated (relatively). would like to have the patient intubated orally first, but this is undesirable in patients who have severe laryngeal fracture. May use a percutaneous kit if available. LARYNGEAL FRACTURE - symptoms include respiratory distress, hoarseness, hemoptysis and subcutaneous emphysema. Should have anatomy restored to normal to prevent airway and speaking difficulty. Leave tracheostomy in for 4-8 days to allow swelling to diminish.

TRACHEOINNOMINATE FISTULA - may arise from erosion of cuff or tracheosotmy tube in to the underlying innominate artery. May see sentinel bleed into the trachea. Confirm diagnosis - deflating cuff will result in major hemorrhage. Temporary control - overinflate trach. cuff or digitally compress the innominate artery against the sternum through the tracheal stoma while the patient is transferred to the OR. Because of contamination, a graft should not be placed. Perform a median sternotomy with right neck extension to expose the innominate artery. Resect the involved artery and oversew the ends using nonabsorbable suture and cover suture line with thymic or mediastinal fat. Do not resect trachea. Incidence of neuro compromise with acute ligation of the in nominate artery is low.

# 56 - ACLS/Basic Protocols As preparing to do a RIH repair on 73 y.o. male with no previous medical history. Procedure has begun under local anesthesia with 1% lidocaine. As procedure begins, notified the patient has no pulse. Issues: a.) Differential diagnosis, what do you do? Explain CPR? b.) Assuming he is resuscitated promptly, do you proceed with hernia repair? c.) What kind of hernia repair would you perform and why? d.) What would be the postoperative management of this patient? # 57 - Management of ARDS Pt. in SICU one week S/P resuscitation for hemorrhagic pancreatitis. Ventilator settings are FiO2 of 40%, TV=800, rate = 12, PEEP=5. Nurse calls you with pCO2 = 32, pH=7.32, pO2=52. Issues: a.) What is your interpretation of these blood gases? Differential diagnosis for hospitalized patients with this ABG? How would you rule out P.E.? b.) What changes would you make in the ventilator settings? How would you adjust these as the patients status changes? c.) What ventilator settings would you use if the respiratory pressure was 44? what can you do to reduce barotrauma? What is the problem with elevated inspiratory pressures? d.) When would you place a Swan Ganz catheter and why? ANSWER: DIFF. DIAGNOSIS: 1.) ARDS 2.) Pulmonary Emboli 3.) Pneumonia 4.) MI with cardiac failure. 5.) Pneumothorax 6.) Sepsis 7.) Atelectasis

8.) Pleual Effusion Diagnosis: PaO2 - need to have Pa02/Fio2 ratio of less than 150 (130 in this case). pH - slightly elevated PaCO2 - mid to low 30s. CXR - though picture of ARDS will often not manifest itself for 24 hrs or more, it is useful to rule out other conditions such as pneumonia, evidence of pulmonary edema from cardiac failure, or pneumothorax. Increase Fi02 - if the Pa02 rises over 100 with increase in Fi02, this is likely due to increased dead space ventilation and will respond to only an increase in Fi02. If increased Fi02 brings only a moderate increase in Pa02, there is likely an intrapulmonary shunt (like ARDS) and more than increasing Fi02 will be needed for adequate therapy. Pulmonary artery catheter - in ARDS should show normal PCWP and normal to slightly elevated cardiac output. Those in congestive heart failure will show elevated PCWP and low cardiac output. V/Q Scan - use to rule out pulmonary emboli. CHOICE OF VENTILATION: VOLUME CONTROL - pump a preset volume of air. PRESSURE CONTROL - pump air until a preset airway pressure is reached Problem with pressure ventilation - as the chest wall compliance gets stiffer, the less volume will be delivered. IMV- ventilator provides breath at set rate to supplement the patients own spontaneous breathing (spontaneous breathing plus extra breaths) AC - whenever patient makes inspiratory effort, the ventilator provides a given volume. TREATMENT OF RESPIRATORY INSUFFICIENCY HYPOXEMIA Increase FiO2 - can help by increasing the alveolar p02, and increasing the amount of 02 that diffuses into the blood stream. However, increased 02 can be toxic. Increased FiO2 does not help in areas of the lung where there is little gas exchange because the alveoli are collapsed. -> can elevate Fi02 to.50, but only for a couple of hours. Once the Pa02 reaches 85-95, the Fi02 can be lowered. Use the lowest Fi02 possible to keep the Pa02 greater than 60. PEEP/CPAP - improves the oxygenation by decreasing the amount of intrapulmonary shunt and decreasing the ventilation perfusion mismatching. Acts to open the partially or completely collapsed alveoli (alveolar recruitment) and improve compliance to the lung.

Problems with PEEP 1.) Decreased Cardiac Output - negates the normal enhanced cardiac return with a negative intrathoracic pressure. This decrease in cardiac output can be overcome by fluid and increasing the intravascular volume, however, judging how much fluid to give requires of Swan Ganz catheter. If PEEP > 10cm H20 needed, or a fall in cardiac output/hypertension with any PEEP, place a Swan Ganz catheter. 2.) Direct Pulmonary Barotrauma - rarely seen with peak inflation pressures less than 60mmHg. VENTILATION - initially, the patient can be hypocapnic (as in this case), but eventually the patient will be come hypercapnic as ventilation fails. Can start out ventilation with high tidal volumes if desired (12-15cc/kg or twice normal), but as the disease progresses and the chest wall compliance decreases and lungs become stiffer the high tidal volumes will lead to increased airway pressures and more barotrauma. AS ARDS PROGRESSES AND BAROTRAUMA IS CAUSED BY ELEVATED AIRWAY PRESSURES, SMALLER TIDAL VOLUMES ARRE RECOMMENDED. -useful to generate a tidal volume- respiratory pressure curve to find the optimal tidal volume. Generally, a tidal volume is selected producing an airway pressure of less than 60cmH20. ALTERNATIVE VENTILATION - progressive decreases in lung compliance and increases in minute ventilation. May opt for pressure control ventilation which will reduce mean airway pressures. In addition, inverse ratio ventilation (IRV) can be used along with pressure control ventilation to keep alveoli open longer while decreasing airway pressures.

# 58 - Management of Pt. in Need of Surgery after Recent MI Cholecystectomy - 76 y.o. female 2 months S/P MI presents with cholecystitis. Rectal CA - 70 y.o. female 2 months S/P MI with near obstructing rectal CA. Issues: a.) How do you want to evaluate the patients cardiac status? b.) Assuming the cardiologist places the patient at a moderate to high risk of surgery, what will you decide to do? c.) What surgical options will allow little or no anesthesia/surgical intervention? ANSWER: ASSESSMENT OF THE CARDIAC RISK GOLDMAN CLASSIFICATION: CRITERIA POINTS HISTORY Age >70 5 MI in last 6 months 10 PHYSICAL EXAM

S3 or JVD 11 aortic Stenosis 3 EKG Rhythm other than sinus 7 >5 PVCs per minute 7 GENERAL STATUS p02< 60, pCO2 >50, K<3.0, HC03<20, 3 BUN >50 or Cr >3 or bedridden from noncardiac disease. OPERATION Intraperitoneal, intrathoracic or aortic 3 Emergency 4 ____________________________________________ 53 NYHA CLASSIFICATION CLASS POINTS RISK CLASS I 5 OR LESS 1% CLASS II 6-12 CLASS III 13 - 25 CLASS IV > 26 ONLY LIFE SAVING OPERATIONS PATIENTS WITH MI IN LAST 6 MONTHS = 30-40% CHANCE OF REINFARCTION REINFARCTION = 70% MORTALITY 25% OF THOSE UNDERGOING SURGERY AFTER HAVING HAD MI IN LAST 6 MONTHS HAVE CARDIAC DEATH. NOTE - each of these patients, age 70 or greater, MI in last 6 months, intraabdominal procedures, +/- emergency without additional assessment for labs, EKG or physical exam have scores which place them in class III or class IV - these patients should have lifesaving operations only. DIPYRDAMOLE THALLIUM - usually predict severe CAD and safer than cardiac cath. Dipyrdamole induces maximal coronary dilation, thallium redistribution following dipyrdamole indicates hypoperfusion that can be corrected by surgery. PERIOPERATIVE MANAGEMENT -

- support of the myocardium can lower the risk of infarction and is directed at lowering the myocardial oxygen demand (determinants are heart rate, contractility and wall tension). - supportive therapies of beta blockers and Ca channel blockers aim at slowing heart rate while nitrates cause ventilation which lowers the ventricular end diastolic volume. - Pulmonary artery catheter + continuous arterial monitoring = assist in avoiding episodes of hypotension and maintaining adequate 02 supply while minimizing myocardial. 02 demand. ANSWER - WHAT CAN BE DONE TO GET THE PATIENT OUT PAST 6 MONTHS FROM MI, WHEN THE RISK OF REINFARCTION GOES FROM 30-40% TO 6% AND STABILIZES. ONLY OPERATE FOR LIFE SAVING SITUATIONS WHEN NO CONSERVATIVE OR TEMPORIZING MEASURES EXIST.

# 59 - Management of Septic Patient/Pt. in Shock 72 y.o. male with CAD, history of urinary obstruction who now appears septic? Issues: a.) Pt. on floor, ill, in respiratory distress, what do you do? b.) Intubated in ICU, what do you do? Explain how a Swan Ganz catheter is placed. Antibiotics? How would you manage fluids c.) Pt. initially had high C.O. and low SVR (sepsis) but now has low C.O. - what could have happened? d.) How do you management this patient who is septic with evolving MI? Ventilator? Drips? Other parameters? e.) How would you manage decreasing urine output? ANSWER: PART #1 - SEPTIC SHOCK SEPSIS - manifested by fever, tachypnea, tachycardia, isolated end-organ dysfunction. SEPTIC SHOCK - hemodynamic instability in addition to above. EARLY SHOCK =WARM SHOCK = HYPERDYNAMIC SHOCK C.O. HIGH + SVR LOW. SVR is low even if no systemic hypotension present. Venous capacitance decreased reducing the circulating volume. Because of this decreased filling pressure, the cardiac output (which is normal or increased) is not enough for demands resulting in underperfusion. Fluid resuscitation can restore ventricular filling. - fluid resuscitation may make the cardiac output very elevated but this is appropriate for the state. No change in cardiac output with fluid resuscitation may reflect underlying cardiac disease or the hypodynamic phase. LATE SHOCK = COLD SHOCK = HYPODYNAMIC SHOCK C.O LOW+ HYPOTENSION. Oliguria and renal failure can arise. Looks like hemorrhagic shock, but with much less volume depletion. Likely to have decrease cardiac output even with normal filling pressures. May involve other factors such as underlying heart disease or inadequate resuscitation. TREATMENT OF SEPTIC SHOCK

1.) INVASIVE MONITORING - Arterial lines, Swan Ganz catheter, Foley. 2.) RESPIRATORY SUPPORT - 02, intubate if in respiratory distress. 3.) RESTORE EFFECTIVE CIRCULATING VOLUME - Isotonic crystalloid, colloid or blood products to get the PCWP to 15mmHg. - to optimize 02 carrying capacity, blood transfusion to get the Hb above 12 or 13 may help. 4.) ANTIBIOTICS - appropriate for the type of presumed infection, otherwise broad spectrum. 5.) SURGICAL INTERVENTION - should be withheld until the patient is resuscitated. 6.) INOTRPIC SUPPORT: HYPERDYNAMIC = HIGH CO+LOW SVR - Dopamine @ 4-10 mcg/kg/min. HYPODYNAMIC = LOW CO + LOW SVR - Dobutamine @ 2-10 mcg/kg/min+/- Dopamine. Norepi @ 2-4 mcg/min. if needed. PART # 2 - CARDIOGENIC SHOCK PUMP FAILURE - with left sided heart failure, fluid which cannot be handled coming from the right side of the heart will back up into the pulmonary system. - As the limits of the capacitance of the pulmonary veins are exceeded, pressures within the pulmonary vasculature rise. At 20 mmHg, pulmonary interstitial edema beings to form, at 22 - 24, alveolar pulmonary edema forms. DIAGNOSIS: - EKG - CXR - look for signs of heart failure (cardiomegaly, effusions, edema) - ABG - look for acidosis and adequacy of oxygenation - Cardiac enzymes - R/O MI. Swan Ganz catheter = indices consistent with low cardiac output (index less than 2.2), elevated SVR and elevated filling pressures (usually 18 or greater). MANAGEMENT OF CARDIOGENIC SHOCK. OXYGENATION - supplemental 02 and mechanical ventilation. Mechanical ventilation beneficial for providing 02 but also for decreasing 02 consumption and workload on heart by decreasing the work of breathing. OPTIMIZING CARDIAC SUPPORT - Preload, afterload and contractility using a Swan Ganz catheter. Preload - aim for a PCWP of 15-18 mmHg. If it is lower than this, try infusing prbcs for a Hgb>12. PCWP above 18 can be manipulated via improving contractility or intravascular volume. Contractility - Dobutamine @ 2- 10 mcg/kg/min. improves contractility without increasing rate. - Dopamine @ 4-10 mcg/kg/min. may be necessary in presence of hypotension. - ventilators (nitrates and MS04) and diuretics may be used to lower filling pressures when pressors are instituted and hypotension is no longer a concern.

Afterload - decrease in resistance to flow may augment cardiac output. - Nitroprusside @ 0.25-0.50 mcg/kg/min. may be used in episodes of hypertension or in the presence of pressors which will provide hemodynamic stability. -> find the optimal place on the Starling curve. Refractory Hypotension - IABP. - in cases where shock is determined to be cardiogenic and due to Acute Myocardial Infarction, consideration should be given to invasive angiography, angioplasty and possibly surgery if indicated.

# 60 - Oliguric Renal Failure Postop. 70 y.o. male in PACU S/P APR with low urine output, period of hypotension intraop. H/O MI 5 yrs. ago. Issues: a.)What is the differential for low urine output in this patient with low urine output? (bleeding, severe hypovolemia, transfusion reaction, ATN, MI, ureter injury, sepsis) b.) What initial interventions should you perform? c.) Initial fluid boluses has minimal effect, what interventions might you perform now? d.) What lab tests would you order? (Urine lytes, fractional excretion of sodium, renal scan) e.) Pathophysiology and treatment of renal failure. ANSWER: PATHOPHYSIOLOGY OF RENAL FAILURE = ISCHEMIC AND TOXIC INJURY ISCHEMIC INJURY - 02 deprivation leads to injury and reperfusion leads to reperfusion injury which the kidney is sensitive to. Prolonged ischemia results in swelling and disintegration of proximal tubule cells, debris collects in the tubule lumen TOXIC INJURY - include products of tissue breakdown from ischemic injury or sepsis, bacterial products, antibiotics and contrast material. CLINICAL SIGNS AND SYMPTOMS PATIENTS AT RISK FOR RENAL FAILURE = preexisting renal disease, decreased renal function because of advanced age or generalized vascular disease, those experiencing prolonged hypotension or decreased cardiac output, multisystem organ failure. HYPOTENSION/DECREASED CARDIAC OUTPUT - ischemic injury alone. BURN/CRUSH/LIMB ISCHEMIA - toxic injury alone. PERIOPERATIVE OPTIMIZATION OF RENAL FUNCTION PREOP ASSESSMENT - RENAL FUNCTION - calculate creatinine clearance in all patients over 50-60 years. - CARDIAC FUNCTION - should be carefully evaluated in any patient over 65 yrs of age. INTRAOP - titrate fluids to keep urine output at least 0.5 ml/kg/hr. (approximately 200-250cc/hr while keeping up with blood loss cc for cc). Can judge fluid status with PA catheter if available. Check periodic ABGs to assess oxygenation and ventilation.

Note - restoration of blood pressure after hemorrhage intraop or from trauma does not assure that adequate blood volume has been restored. POSTOP - usual shit. DIAGNOSIS OF ACUTE RENAL FAILURE 1.) Low Urine Output 2.) Rising BUN/Creatinine Potential Problems - Non-oliguric renal failure may present with only rising BUN and Creatinine which may not present for several days (preventing early diagnosis and treatment). Low urine sodium is present in the patient with low renal flow from hypotension as well as in the patient who is going into renal failure. STEP #1 - LOW URINE OUTPUT = less than 0.5ml/kg/hr. Place a PA catheter to assess volume status and cardiac output. If cardiac output is adequate and patient has adequate volume status, may start Dopamine at 3 micrograms per kg per min. - US - to rule out obstruction. - Hct/Hb - rule out bleeding - UA - look for urine casts or cells. STEP #2 - URINARY CREATININE, UREA AND OSMOLALITY Normal Values: Urine Creatinine/Plasma Creatinine > 2.5/1.0 Urine Urea/ Plasma Urea > 10/1 Urine Osmolality/ Plama Osmolality > 1.1/ 1.0 - when any of these are below critical levels and there is low urine output despite adequate medical therapy, the patient has impending renal failure and should be treated. - diagnosis may be more difficult in patient with nonoliguric renal failure. TREATMENT OF ACUTE RENAL FAILURE STEP #1 - using data from PA catheter, continuous arterial monitoring with ABGs, Hct/Hgb need to optimize oxygen carrying capacity. Transfuse to Hct of 30, supplement 02, pressors in addition to dopamine as needed to augment cardiac output, NTG as needed for MI, antibiotics as needed for sepsis. STEP #2 - DIURESIS - Mannitol = 6-12 grams IV - Furosemide = 20mg, 40 mg if no response in 1-2 hrs., 80 mg if no response in 1-2 hrs. - should assist in increasing renal blood flow and decreasing swelling, maintaining tubular blood flow.

STEP #3 - SUPPORTIVE CARE - Hct/Hgb = maintain at around 30. - Control serum electrolytes using fluid s administering 0.5meq/kg/day of sodium. - total daily fluid intake at 800 - 1000cc/day in addition to urine and other fluid losses. - monitoring of serum K+ - use Kayaxelate 25% in 50cc orally or enema as needed for elevated K+ (cant do either one in this case). - start TPN - renal failure is associated with increased catabolism and increased protein breakdown. - adjust medications accordingly and change nephrotoxic drugs. STEP #4 - DIALYSIS Indications: 1.) Acidosis 2.) Hyperkalemia 3.) Fluid Overload 4.) Symptomatic Uremia - bleeding, nausea

# 61 - Pulmonary Emboli in Postop Patient. 25 y.o. male, 5 days S/P MVA in which he suffered bilateral femur fractures, pelvic fractures, left hemopneumothorax and pulmonary contusion develops shortness of breath and tachypnea with respiratory rate of 42 with P=120, BP=90/60. Issues: a.) Initial resuscitation and evaluation. What is differential diagnosis (pneumonia, PE,MI, fat emboli) b.) Is a lung scan indicated when CXR shows bilateral pulmonary infiltrates? c.) When lung scan shows intermediate probability, what is the next step? d.) What do you do when the pulmonary angiogram shows an embolus to the right lower lobe? (filter) e.) What kind of filter would you use, how would you place it? f.) What would you do if prior to placement of a filter the patient develops recurrent emboli with worsening hypotension - what would you do? What are the indications and contraindications for lytic therapy? ANSWER: Differential Diagnosis: pneumonia, pulmonary contusion, fat emboli, PE, ARDS, pneumothorax, sepsis, myocardial contusion. DIAGNOSIS: LABS: - WBC - less than 15,000 will rule out pneumonia effectively. - ABG - will show hypoxemia generally (pO2 < 60 mmHg) but this will not differentiate PE from other diagnosis. Reduction in PaC02 is the most discriminating finding for pulmonary emboli. If hypoxemia and hypocarbia not present is not P.E. - EKG - primary reason is not to get an EKG diagnostic for P.E. but to rule out M.I. - CXR - also useful to rule out other diagnosis such as pneumonia, pneumothorax, esophageal perforation or CHF. Is necessary for ventilation scan.

- LUNG SCAN (V:Q) Normal - excludes P.E. LOW PROBABILITY- LOW SUSPICION - check venous duplex INTERMEDIATE PROBABILITY - check for previous cardiopulmonary disease. (-) cardiopulm. disease (-) duplex = follow. (-) cardiopulm disease (+) duplex = treat with anticoag. if poss. Cant anticoagulate (-) cardiopulm disease = pulmonary angio (+) cardiopulm disease (-) duplex = pulm. angio (+) cardiopulm. disease (+) duplex = treat with antioag. if poss. HIGH PROBABILITY - treat. - PULMONARY ANGIOGRAM - normal test excludes pulmonary emboli. TREATMENT: ANTICOAGULATION/HEPARIN - MINOR P.E. (Pa02 = 65-80, PaC02 = 30-35, tachycardic, anxious+hyperventilating, 20-30% PA occlusion). Heparin should be used at 150U/kg to achieve a PTT twice normal, unless contraindicated. Heparin should not increase bleeding risk, but with this patient s degree of injury, it would be best avoided. Contraindicated for this patient because this is not a minor PE and recent trauma. INFERIOR VENA CAVA INTERRUPTION/FILTER - MINOR P.E. Should be used in situations in which anticoagulation can not be used or in situations in which anticoagulation has failed. TECHNIQUE - either via femoral or jugular vein, under local anesthesia the jugular or femoral vein is exposed and controlled with umbilical tape and clamp. Needle is placed in vein and guidewire is passed. A venocavogram is performed (should not be performed if there is clot in the system above the insertion site - should use opposite side or the jugular vein). - sheath is passed over wire into the IVC under flouro. Carrier is passed over the sheath to the desired level. Filter deployed. THROMBOLYTIC THERAPY - MAJOR P.E. (Pa02 < 65, PaCo2 < 30, elevated CVP, hypotensive but responds to fluids) MASSIVE P.E. (PA02 < 50, PaCO2< 30, greatly elevated CVP, hypotensive that is refractory to fluids and requires pressors) or MINOR PE ON EXISTING SEVERE CARDIOPULMONARY DISEASE. USE IN THE ABSENCE OF CONTRAINDICATIONS. ABSOLUTE CONTRAINDICATIONS: Active Bleeding

Intracranial Disease Recent Eye Operation RELATIVE CONTRAINDICATIONS: Recent Operation (10 days) Recent serious trauma Postpartum (7 days) Recent external cardiac massage Biopsy at site which is inaccessible to compression. History of GI Bleed/ Gastric or Duodenal Ulcer. Pregnancy Uncontrolled Hypertension. Arterial Aneurysm al Disease. Knitted or Dacron Graft. DOSE: STREPTOKINASE BOLUS = 250,000 U over 20 minutes. INFUSION = 100,00 U per hour T- PA BOLUS = 10 mg INFUSION = 100 mg over 2 hrs. HEPARIN CONTINUED AFTER LYTIC THERAPY STOPPED. PULMONARY EMBOLECTOMY - MASSIVE EMBOLI (Pa02< 50, PaC02<30, elevated CVP and hypotension refractory to fluid requiring pressors). In patients who have collapse and suspicion of PE, should be anticoagulated with heparin and started on presssors to support blood pressure while the diagnosis is being confirmed by angio. Can use Dopamine or Isoproteronol. If the patient responds to heparin and does not need pressors, place a filter and get out. - under local anesthesia, exposed the femoral artery and vein, insert cannulas and place the patient on partial bypass, this allows for hemodynamic support while inducing general anesthesia. Median sternotomy performed and partial bypass converted to complete bypass by placing a cannula in the SVC. Incision made in the main PA and clot removed. Lungs are squeezed to mobilize distal clots proximally to main PA. Mortality - 40%. Should really only perform in those who need closed cardiac compression to maintain BP or in those who catheter embolectomy is unsuccessful.

PEDIATRICS
# 62 - Intusseception 10 month old presents to ER with a history of severe crampy abdo pain mainifested by crying loudly, pulling legs up and writhing. Passes currant jelly stool. Issues: a.) What would you do to evaluate the patient? b.) What is the height of the column of Ba? c.) BaE does not work. What would you do now?

d.) Intraop the patient has an ileocolic intusseception, how would you handle this? e.)There is a mass at the leading edge, how would you manage this? What are the margins? ANSWER: ABDOMEN - soft and nontender initially, tenderness and pain in dictate necrosis. MASS - palpable on exam 85% of time. RADIOGRAPHIC APPEARANCE: EARLY - nonspecific, perhaps empty RLQ and mass at hepatic flexure. LATE - appearance of mechanical obstruction. BaE - diagnostic with almost 100% accuracy. Also allows the possibility of therapeutic reduction for ileocolic intusseception. All children without peritonitis should have BaE for diagnosis and possible treatment. - small bowel to small bowel intusseception is harder to diagnose early because of the inability to visualize via contrast enema, usually diagnosed later when complete obstruction presents. TREATMENT: BaE Guidelines: 1.) Column of Ba should not exceed 1 meter/ 3 feet 6 inches. 2.) Do not manipulate the abdomen. 3.) Stop the enema if the column has not advanced in 10 minutes. 4.) Do not stop the enema until the Ba refluxes into the ileum, indicating that the reduction is complete. - Failure or uncertainty of hydrostatic reduction mandates surgery. If there is spontaneous reduction during preparation for surgery/induction of anesthesia, exploration to confirm this is all that is necessary. - Viable bowel - manual reduction by milking/compressing the bowel distal to the intusseception forcing the intussecepted bowel proximally. - Inability to reduce indicates necrosis at the lead point and should not be pursued. Resect with anastomosis. - perform appendectomy. Recurrence = 5% of time after hydrostatic or surgical reduction. Should be treated with repeat hydrostatic decompression. OLDER CHILDREN - the older the child, the more likely there is an anatomic leadpoint Meckels, intestinal lymphoma, cystic fibrosis, polyps, intramural bleed from HenochSchonlein purpura. Children older than 4 should have this worked up even if the hydrostatic reduction is successful.

ADULTS - associated with malignancy in 42% of cases, associated anatomic leadpoint present in 85% of cases. Spontaneous (non-postoperative intusseception) Intusseception is an indication for surgery in adults for evaluation of possible malignancy. Colocolonic intusseception should be resected without attempt as reduction. Entero-enteric intusseception in adults should be reduced and evaluated for the possibility of malignancy.

# 63 - TE Fistula Newborn becomes cyanotic every time it feeds, salivates excessively and has pneumonitis Issues: a.) What is the differential diagnosis? b.) How would you confirm the diagnosis? c.) What would contraindicate an immediate surgical correction? d.) How would you repair this TE fistula? ANSWER: ESOPHAGEAL ATRESIA WITH DISTAL TRACHEOESOPHAGEAL FISTULA - present in the vast majority of cases (85-90%). Differential Diagnosis: 1.) Esophageal atresia with distal TEF. 2.) Esophageal atresia without TEF - recurrent aspiration with feeding. 3.) H- Type Fistula without esophageal atresia. DIAGNOSIS: passage of nasogastric tube into stomach with CXR. Coiled NGT in proximal pouch and air in stomach gives diagnosis. Can be confirmed with small amount of Ba but this will increase the risk of aspiration of Ba. => important to find out preop the side of the aortic arch, which occurs on the right in 5%. Usually this can be easily seen on CXR, if not then echocardiogram or CT scan should be undertaken. IMMEDIATE SURGERY - WATERSTON A CLASSIFICATION - BIRTHWEIGHT OVER 5.5 LB. AND OTHERWISE WELL DELAYED SURGERY - WATERSTON B+C CLASSIFICATION -BIRTHWEIGHT 4-5.5 LB. AND WELL OR - BIRTHWEIGHT OVER 5.5 LB. BUT MODERATE PNEUMONITIS OR SEVERE CONGENITAL ANOMALY CONTRAINDICATION TO IMMEDIATE SURGERY = LOW BIRTH WEIGHT (< 5 LB.), SEVERE PNEUMONIA, SEVERE ASSOCIATED CONGENITAL ANOMALY (THINK VATER)

NO IMMEDIATE SURGERY -> place gastrostomy, establish proximal esophageal pouch suction, start TPN. Allow pneumonia to resolve, gain weight. SURGICAL APPROACH - posterolateral thoracotomy on the side opposite the aortic arch. 4th interspace, retropleural approach. Ligate the azygous vein (if on the right) and the TEF will lie just beneath this. Have anesthesiologist place large tube in esophagus to facilitate dissection of the proximal pouch. The distal TEF will insert into the trachea at the carina. Ligate the TEF with interrupted 5.0 silk leaving a 3 mm cuff of fistula to avoid narrowing the carina. The proximal pouch is anastomosed to the distal esophagus in 1 or 2 layers using interrupted 5.0 silk. - can extensively mobilize the upper pouch, but cannot mobilize the lower pouch because of the segmental blood supply. - 1 or 2 circular myotomies of the upper pouch can be performed for optimal length LONG GAP ATRESIA - usually occur in patients with pure atresia and no TEF. If the atresia is less than 2-3 vertebral bodies, it will be possible to perform anastomosis. if not, place gastrostomy and stretch proximal pouch for 3-6 weeks. Can confirm the shortening of the gap with radiographic studies.

#64 - Hirschprungs Disease 4 day old male with abdominal distention, no vomiting, normal rectal exam. Issues: a.) How would you make the diagnosis? b.) How would you intervene and confirm the diagnosis? c.) How would you perform the rectal biopsy? ANSWER: PRESENTATION: generally these neonates will not pass meconium in first 24-48 hrs., though this will usually only be noticed in retrospect. - will present with nonspecific signs of abdominal obstruction - bilious vomiting, abdominal distention. May present early with symptoms of abdominal obstruction (as in this case) or later in childhood into early adulthood as chronic constipation. - may present with symptoms of enterocolitis (fever, distention, bloody diarrhea) and should be treated with IV fluids, antibiotics and decompression via colostomy proximal to the transition zone. Though this presentation is unusual, the possibility of this makes the need to pursue the diagnosis aggressively important. PHYSICAL EXAM - usually normal. Occasionally, rectal exam will cause passage of stool.

DIAGN0SIS: PLAIN RADIOGRAPHS - air-filled abdomen, cannot differentiate colon and small bowel gas in a neonate. Will only indicate obstruction. Pneumatosis can be seen with entercolitis or NEC. BARIUM ENEMA - should follow plain radiographs when there is evidence distal bowel obstruction. Do not perform any enemas prior to the BaE. - insert plain catheter without balloon into rectum proximal to the internal sphincter and the barium is slowly injected with syringe. Diagnostic BaE - 1.) Transition zone - may not be present in early neonates because the colon has not had time to dilate. 2.) Retained Barium after 24 hrs. - must have histologic diagnosis prior to surgery. BIOPSY - suction biopsy safe and can be performed at bedside. Suction biopsy should be taken at least 1.5 - 3 cm proximal to the dentate line. Biopsy of mucosa and submucosa is taken - risk of perforation is very small. This specimen should have portion kept fresh for cholinesterase activity and portion taken for examination for ganglion cell s in the intramural plexus. - if diagnosis is not made on suction biopsy, then a full thickness biopsy should be taken under general anesthesia. May need to do this in older children. TREATMENT : 1.) COLOSTOMY - the colostomy should be done using a loop of normally ganglionated colon. In order to properly locate the transition zone, mapping should be done using serial frozen sections of seromuscular biopsies. May place colostomy as either a sigmoid (above transition zone) or transverse colostomy. If a transverse colostomy is placed, it is likely that a third procedure will be necessary to close this colostomy. A sigmoid colostomy can be taken down as part of a pull through procedure as a definitive two-stage procedure. 2.) PULL THROUGH PROCDEDURE - at 9 - 12 months of age. DUMAMEL - blunt retrorectal dissection is performed. Incision in the posterior wall of the aganglionic rectum. Retrorectal pullthrough performed after resection of aganglionic segment. The end of the ganglionated colon is anastomosed to the side of the aganglionic rectum. SOAVE - Resection of aganglionic segment and endorectal pull through with colorectal anastomosis above the sphincter mechanism. This leaves a long rectal sleeve that the ganglionated colon is pulled through. SWENSON - the rectum is divided at the transition zone with stapler. Extramural dissection is performed to free up the rectum. Rectum is pulled through the anus. The rectum (outside the anus) is divided just above the sphincter and anastomosis is performed. Colorectal anastomosis is performed.

#65 - Incarcerated Inguinal Hernia 8 yo male with incarcerated inguinal hernia. Issues:

a.) How will you reduce this hernia? b.) If the hernia is reduced, when would you operate? c.) If Strangulated, what would you do? d.) The bulge is red-hot, the patient has fever, and the intern reduces the hernia on the way to the OR. What do you do? ANSWER: - must be distinguished from acute hydrocoele or inguinal lymphadenitis. If the bulge has been present for more than 24 hrs without signs or radiographic evidence of intestinal obstruction, it is unlikely an incarcerated hernia. Palpable normal cord above the mass excludes an incarcerated hernia. COMPLICATIONS - incarceration leading to strangulation and necrosis. Also ischemic necrosis of the testicle from impingement of hernia onto spermatic vessels. Because of these complications, inguinal hernias should be repaired when diagnosed. Period. No minimum weight or age is needed. Operate on during hospitalization. AGE < 50 WEEKS - hospitalize over night. AGE > 50 WEEKS - same day surgery. INCARCERATION: if there is no peritonitis, attempt may be made at reduction. The patient is placed in bed with legs elevated and sedated. Gravity and sedation are often enough to reduce. If not, pressure can be placed on the testicle. If reduced, allow edema to resolve for 12 -24 hrs and then operate to repair. DIFFICULT REDUCTION - ADMIT AND OBSERVE. FAILED REDUCTION - OPERATE. CONTRALATERAL SIDE - EXPLORE OPPOSITE SIDE IN PATIENT UNDER 2, BECAUSE OF 60% CONTRALATERAL HERNIA RATE. # 66 - Congenital Diaphragmatic Hernia Know the different types and how to reduce. Associated with cardiac anomalies? TYPES OF CONGENITAL DIAPHRAGMATIC HERNIAS: 1.) FORAMEN OF BOCHDALEK 2.) FORAMEN OF MORGAGNI. 3.) EVENTRATION OF THE DIAPHRAGM 4.) TRAUMATIC HERNIATION OF DIAPHRAGM - ACUTE OR CHRONIC.

FORAMEN OF BOCHDALEK HERNIA - posterolateral defect, more often on the left side than the right. Herniation of abdominal contents into chest causes collapse of ipsilateral lung and shift of the mediastinum to the contralateral side. - can be diagnosed via chest X-ray = showing left hemithorax filled with air/fluid levels of bowel with shift of mediastinum. If this is not diagnostic, upper GI series with Barium will show small bowel in the chest. Bowel should be decompressed prior to surgery with NGT. ASSOCIATED ANOMALIES - VSD and Coarctation of the aorta are the most common, but all have been reported - preop echocardiogram are routine. Almost all will have PDA from the pulmonary hypertension and this should be allowed to close on its own after surgery. - other associated anomalies are pulmonary sequestration, GERD, CNS malformation. SURGICAL TREATMENT - transverse abdominal incision, reduce with gentle traction. Close with suture or prosthetic material for large defects. Closely examine the reduced abdominal viscera for atresias. If the abdominal contents do not fit, a silo can be created. FORAMEN OF MORGAGNI HERNIA - defect anterior retrosternum (parasternal as opposed to midline) where the superior epigastric artery transverses the diaphragm. Plain films generally show air filled sac in the mediastinum. Reduction via a transabdominal approach. These tend to present at later age.

ONCOLOGY
# 67 - Facial Melanoma Elderly female has a 1 cm black lesion on her right cheek, and clearly palpable ipsilateral neck node. Other physical examination and medical history are not pertinent. Issues: a.) What is the technique of the original biopsy? b.) Pathology report comes back malignant melanoma = 1mm depth = 1.3 mm depth = 3.0 mm depth = 3.4 mm depth = 5 mm depth what is the operative procedure of choice? c.) Describe the operative procedure in detail including what would be removed and what would be left. How large a margin would you take for the lesion? d.) What do you do as far as adjuvant therapy when a single node is found to be positive? e.) The patient is found months later to have mental status changes with obtundation. CT scan shows a single met. What do you do? What if the patient comes back with a single lung met. What do you do? d.) Explain in detail Clarks and Breslows level of melanoma. Which one is better? ANSWER: ORIGINAL BIOPSY -

EXCISIONAL BIOPSY - indicated for suspicious lesions that are not large (<1.5 cm), located on areas where amount of skin does not matter (trunk). The orientation of the biopsy long axis should be oriented toward the regional lymph node basin for re-excision purpose (extremity melanoma should be oriented on the long axis). - biopsy should have 2 mm margin of normal tissue and should include subcutaneous tissues. INCISIONAL BIOPSY - USE HERE - performed when the amount of the skin removed is critical, such as the hands, feet or face. Can also be done when and excisional biopsy will be a large undertaking. Can use scalpel or 6 mm punch biopsy to include subcutaneous tissue. Can be done in office. No disadvantages (survival or local recurrence) with this approach. DEFINITIVE EXCISION OF MELANOMA PRIMARY - must widely excise the biopsy site or tumor with margin of normal appearing skin. Since local recurrence depends more on tumor depth than it does on amount of normal skin margin, the amount of normal skin margin should be dictated by the melanoma depth. IN SITU MELANOMA - it cannot metastasize, but it can locally recur. Excise with 0.5 - 1.0cm margin. THIN MELANOMA - (<1 mm in depth) - no clear studies to indicate the minimally effective margin, so no clear answer. 1.0 cm margin. INTERMEDIATE THICKNESS AND THICK MELANOMA (any equal to or greater than 1 mm in depth) - studies show that smaller margins for thin melanomas are alright but no clear minimal guidleline for melanoma greater than 1 mm in depth or for any subgroup from 1- 4 mm. 2.0-3.0 cm margin. - with excision or re-excision, orient the long axis of the incision toward the regional lymph node basin (this allows melanoma cells in transit in lymphatics to be taken with the primary specimen.) Margins should be the radius of the short axis of the ellipse. The long axis should be at least twice the short axis, but more importantly it should be long enough that when the ellipse is primarily closed there are no dog ears. SPECIAL SITUATIONS: FACE - CAN USUALLY ONLY GET 1 CM MARGIN ON THE FACE BECAUSE OF VITAL CONTIGUOUS STRUCTURES. Must use judgement based on depth. Use margin of twice the largest diameter of the lesion. FINGERS AND TOES - Fingers - take at the middle interphalangeal joint. - Thumb - take proximal to distal joint of thumb. - Toe - amputate entire toe.

SOLE OF FOOT - avoid excision of weight bearing surfaces of t he plantar surfaces of the foot. EAR - initial wedge excision of the helix with reexcision if + for malignancy. THERAPEUTIC LYMPH NODE DISSECTION - removal of positive lymph nodes is the only chance for control of local disease or cure. HEAD AND NECK MELANOMA - melanomas anterior to the pinna of the ear metastasize to the parotid, submandibular, submental, upper jugular and posterior triangle lymph nodes. - for head and neck melanoma with positive node clinically, a MRND should be performed. - melanoma arising from the scalp or face, anterior to the pinna of the ear, above the commissure of the lip are at risk for mets to the parotid gland. Parotid nodes are contiguous with cevical lymph nodes. Melanoma can metastasize to the superficial parotid or to the nodes in the tail of the parotid gland, therefore, superficial parotidectomy should be performed. - for this melanoma of the cheek with a positive clinical node, perform a MRND and a superficial parotidectomy. Complication - pain, seroma or skin slough. Shoulder drop if you fuck up. GROIN DISSECTION FOR LOWER EXTREMITY MELANOMA - 2 lymph node basins, the superficial nodes located in the femoral triangle and the deep inguinal, obturator and iliac nodes. Palpable/suspicious groin nodes - should have radical groin dissection obtaining femoral and ilioinguinal nodes. TECHNIQUE - curving incision from 4-5 cm anteromedial to the anterior superior iliac spine over the middle of the inguinal ligament to the apex of the femoral triangle (extended transplant incision). Skin flaps are raised medially to the pubic tubercle and the adductor longus and laterally to the sartorius - dissection of deep n odes is performed by dividing the inguinal ligament to provide better exposure and to allow removal of superficial and deep nodes in continuity. I f these nodes appear normal, stop at the common iliac bifurcation. If these nodes are abnormal continue to the common iliac nodes. Non-palpable/no suspicious groin nodes - for what would be considered an ELND, proceed with superficial node dissection of the femoral triangle nodes, if one of these is found positive on frozen section, proceed with deep node dissection. Complication - lymphedema - use early elevation and compression stocking. AXILLARY LYMPH NODE DISSECTION: - should perform a complete axillary lymph node dissection of all levels (I-III), continuing dissection to include nodes beneath the pectorals minor (the pectoralis minor may be taken with this dissection) Complication - lymphedema. POPLITEAL NODE DISSECTION: - this area is usually not involved with tumor cells, so dissection of the popliteal space is not indicated unless the lesion lies directly over this area.

DEMONSTRATED IN TRANSIT METS IN EXTREMITY - isolated limb perfusion should be tried as it can offer dramatic results. This obviously is not offered for melanoma of the trunk. ADJUVANT THERAPY - no good demonstrated chemotherapy for melanoma. Some evidence of benefit using alpha interferon or IL-2 stimulated LAK cells. Recommend a clinical trial. RADIATION THERAPY - use for bone and brain mets. TREATMENT OF DISTANT METASTATIC MELANOMA - tendency to metastasize to distant subcutaneous sites, GI tract (usually small bowel). - resection is indicated for palliation of symptoms. Isolated brain mets should be treated with a combination of surgery and radiation. GI tract melanoma should be resected fro palliation of obstruction or bleeding. - isolated lung met should be treated with resection. Multiple lesions are not treated with surgery. - liver metastasis should NOT be treated with surgery because of short life expectancy. - bone = radiation. CLARKS LEVEL - determines the level of invasion by the depth of invasion into the layers of the skin. Tends to be subjective, requires multiple sections and a committed pathologist. BRESLOWS LEVEL - PRESENT STANDARD. Thickness of tumor is determined by the depth in millimeters. Less subjective. Better. # 68 - Trunk Melanoma 45 y.o. male presents with pigmented lesion that was 4 cm superior and lateral to the umbilicus that was about 1.5 cm in diameter. There is no axillary adenopathy. Issues: a.) How would you do the original biopsy? b.) The biopsy comes back as a 1.5 mm in depth in melanoma. What would you do? How can you tell which lymphatics drain this melanoma? c.) At what levels would you do a lymph node dissection? SENTINEL NODE BIOPSY - preop lymphoscintigraphy is needed to determine if more than one regional lymph node basins are draining a particular area. - Isosulfan blue dye is injected intradermally (0.5-1.0 cc) on either side of the resection scar. Wait 5-10 minutes. Incision is made over the regional lymph node basin (about 1/2 that used for complete lymph node dissection) and skin flaps are raised. The lymphatic channel

staining blue is found and traced toward the lymph node. The lymph node staining blue is removed and sent for frozen section. (+)sentinel node = 37% chance that other nodes are positive, complete node dissection is performed. (-) sentinel node = 99% assurance that remaining nodes are negative. Procedure is stopped at this time. **importance of this technique is to get around the decision for ELND, the indications for which are still controversial because subgroups of intermediate thickness melanomas which would benefit from ELND have yet to be identified. ELECTIVE LYMPH NODE DISSECTION: ONLY FOR INTERMEDIATE THICKNESS (1-4.0 MM) MELANOMA. - THIN MELANOMA (< 1.0 MM) = 95% cure rate and disease is localized. No therapeutic benefit for ELND in these patients. - INTERMEDIATE THICKNESS MELANOMA (1.0-4.0 mm) = 60% chance of harboring occult lymph node metastasis but only a 20% chance of distant mets. Potential benefit of ELND. - THICK MELANOMA (>4.0 MM) = > 60% chance of occult lymph node metastasis but > 70% chance of occult distant mets. These patients do poorly because of distant mets negating the benefit of removing positive regional nodes. PREOPERATIVE LYMPHOSCINTIGRAPHY - needed to define the regional lymph node basin for head and neck and trunk.

# 69 - Seminoma/Testicular Mass 18 y.o. male presents with a painless mass that he notices in his left testicle. Issues: a.) Differential diagnosis and preoperative work-up (U/S, AFP, CEA, beta-HCG, Ct scan) b.) Operative approach to this lesion - should it be biopsied or excised, should it be approached through the groin or the scrotal-bag? c.) Approach if several 2-3 cm nodes are seen along para-aortic chain on CT scan and pathology found to be seminoma? Similar scenario but path comes back as non-seminomatous? d.) Subsequent treatment of seminoma - adjuvant therapy? e.) What if later a single lung lesion is found, bx (+) for seminoma? Bilateral lung lesions? What if later a single lung lesion is found, bx (+) for non-seminoma? Bilateral lung lesions? ANSWER: Differential Diagnosis: 1.) Testicular torsion 2.) Epidymitis 3.) Epididymal orchitis. 4.) Hydrocoele 5.) Hematoma 6.) Hernia 7.) Spermatocele

PREOP WORKUP - ULTRASOUND - used to tell if the mass is intratesticular or extratesticular. If the mass is intratesticular, it is testiclular cancer until proven otherwise. Quick, easy way to narrow the differential. - LABS - AFP, beta - HCG, LDH Seminoma - 10% will have elevated HCG. Non-seminoma - 90% will have elevations of either or both of AFP or beta- HCG. - CXR - CT scans - chest, abdomen and pelvis. DIAGNOSIS - RADICAL ORCHIECTOMY. ONCE TISSUE IS OBTAINED AT ORCHIECTOMY, METASTATIC WORKUP IS COMPLETED AND FURTHER MANAGMENT IS DECIDED. RADICAL ORCHIECTOMY - approach through and inguinal incision and the cord is ligated high at the inguinal ring to eliminate the risk of scrotal or lymphatic seeding. STAGING = SPECIMEN + CT SCAN + TUMOR MARKERS. SPECIMEN = SEMINOMA OR NON- SEMINOMATOUS T1 = localized to testicle. T2 = extends beyond the tunica albuginea. T3 = involving the epididymis T4 = invading spermatic cord or scrotal wall. - also need to know whether there is lymphatic or vascular invasion. CT SCAN - MOST EFFECTIVE WAY OF DETERMINING RETROPERITONEAL LYMPH NODE INVOLVEMENT. Can identify para-aortic LN of less than 2 cm in size. (+) CT Scan = likely to identify metastatic disease accurately. (-) CT Scan = 25% chance of mets even with negative CT scan. TUMOR MARKERS : AFP - seen in a number of malignancies (liver, pancreas, lung, stomach). Can be produced by pure germ cell tumors (embryonal, teratoCA, yolk sac) or combined tumors, but not seen in pure seminoma. If it looks like seminoma but the AFP is elevated, it is considered nonseminomatous and should be treated as such. beta - HCG - can be produced by both seminoma and nonseminoma. POST-OP TUMOR MARKERS - because marker half life is 5-7 days for AFP and 2-3 days for HCG, persistent elevations post-op indicate residual tumor. However, normalization of

tumor markers may mask the presence of residual tumor in the retroperitoneum which is present 40% of the time. STAGING FOR SEMINOMA AND NON-SEMINOMA A CONFINED TO THE TESTICLE B1-B3 SPREAD TO THE RETROPERITONEAL NODES C METASTATIC DISEASE TREATMENT SEMINOMALOCALIZED - STAGE I or A or Min. STAGE II or B - ORCHIECTOMY + RADIATION total of 2500-3000 Gy radiation should eradicate any tumor cells in retroperitoneal nodes and give 5 yrs survival in 90-95% of cases. Radiation is directed ipsilateral iliac, para-aortic and paracaval. Do not radiate the mediastinum. ADVANCED - Advanced or Bulky STAGE II or B - STAGE III OR C - ORCHIECTOMY + CHEMOTHERAPY - Cisplatin based combination chemotherapy is best. This is defined as disease above the diaphragm, distant mets or bulky abdominal disease. If there is response to this (60-100%), there is no need for further therapy. NONSEMINOMATOUS LOCALIZED - STAGE I OR MINIMAL STAGE II - ORCHIECTOMY + RETROPERITONEAL LYMPH NODE DISSECTION - GOLD STANDARD FOR TREATMENT AS WELL AS COMPLETE STAGING - Conventional methods will understage Stage I disease 20-25% of the time. Alternatively, the patient may be followed closely with serial tumor markers, anticipating a recurrence rate of 20% - this is only for the very compliant patient. This treatment is for patients with disease localized to the testicle/cord or those with retroperitoneal lymph nodes < 5 cm. ADVANCED - Advanced or Bulky STAGE II or STAGE III - ORCHIECTOMY + INDUCTION CHEMOTHERAPY - bleomycin, etoposide and cisplatin are used as induction therapy for distant disease or bulky retropertioneal disease. Those showing a complete response to chemotherapy may be observed. Those showing a partial response may have the residual retroperitoneal disease resected via RPLND. RPLND TECHNIQUE - can be done via a midline approach for transabdominal or via a thoracoabdominal approach. The vena cava and aorta should be exposed and lymph nodes removed from the ipsilateral iliac bifurcation to the SMA along the aorta and from the caval bifurcation to the renal veins. PULMONARY METS Criteria For Resection: 1.) Presence of nodules consistent with mets. 2.) Control of primary tumor. 3.) Potential for complete resection. 4.) Sufficient pulmonary reserve for resection 5.) Absence of uncontrolled pulmonary mets.

PULMONARY METS AND GERM CELL TUMOR - chemotherapy is the primary form of treatment for both seminoma and non-seminomatous testicular CA mets. Surgical resection is reserved for those with residual disease after initial therapy when tumor markers have fallen to normal level - seminoma is seen as increased mediastinal adenopathy, nonseminoma is seen as new pulmonary nodules. BILATERAL DISEASE - not a contraindication to resection if all of the above criteria met. Can be resected via staged bilateral thoracotomies or via a median sternotomy resecting both sides at once. # 70 - Hodgkins Disease 38 y.o. woman presents with an anterior cervical triangle neck mass - otherwise asymptomatic. Issues: a.) Initial evaluation and workup of this lesion. b.) How would you biopsy this lesion? What if FNA is not diagnostic? c.) What are the characteristic cells of Hodgkins Disease? What do they look like? d.) What are the 4 types of Hodgkins? Which is the most common? e.) What is the workup of Hodgkins Disease? When is a staging lap indicated? What tissues are sampled during a staging lap? What are indications for chemo/XRT? Potential Trap - general anesthesia for biopsy of cervical node with possibility of Hodgkins disease and mediastinal compression on the trachea. HODGKINS DISEASE - characterized by single or multiple painless lymph node enlargements. May be associated with fever, night sweats, weight loss. DIAGNOSIS: -excluding benign primary thyroid tumors, 80% of neck masses in adults are malignant, of which 80 % will be metastatic. 80% of metastatic neck masses arise from above the clavicle. INITIAL EVALUATION AND EXAMINATION: EXAM - if the mass moves on swallowing, it is likely under the strap muscles and is likely a thyroid primary. - use otoscope, tongue depressor, nasal speculum, glove and mirrors to examine the oral cavity (glove and tongue depressor and otoscope), oropharynx, hypopharynx, larynx and nasopharynx (nasal speculum and mirrors for indirect laryngoscopy).

- also examine groin and axilla (adenopathy associated with Hodgkins disease), breasts (breast CA), abdomen and rectal exam (GI tract CA), respiratory tract (lung CA). BIOPSY - FNA Positive Results: Metastatic thyroid - definitive surgery. Lymphoma - remove entire node to confirm and establish cell type. Branchial Cleft cyst - definitive surgery. INCONCLUSIVE FNA ----> OPEN BIOPSY - important to perform a complete excisional biopsy as the stromal architecture is important in diagnosis. Present for formalin fixation. Reed - Sternberg Cells - large, multinucleated cells. Diagnostic of Hodgkins Disease. TYPES OF HODGKINS DISEASE: 1.) Nodular Sclerosis 2.) Lymphocyte Predominance 3.) Mixed Cellularity 4.) Lymphocyte Depletion DIAGNOSTIC WORKUP OF HODKINS DISEASE: 1.) Histologic Diagnosis 2.) CT Scan of Chest and Abdomen 3.) Lymphangiogram - only if initial CT Scan of abdomen is negative. 4.) CBC with Differential and Platelet Count. STAGING OF HODGKINS DISEASE: I - involvement of single lymph node area. II - 2 or more lymph node regions on the same side of the diaphragm or involvement of single extralymphatic organ with lymph nodes on same side of diaphragm. III - involvement of lymph node basins on both sides of the diaphragm or involvement of extralymphatic organ. (lymph nodes or spleen) IV - disseminated or multifocal involvement of one or more extralymphatic organs (bone marrow, lung, liver, skin, GI tract). A - no constitutional symptoms. B - weight loss of 10% of body weight in 6 months, unexplained fever of 38 degrees, night sweats. ROLE OF STAGING LAPAROTOMY - should be used for all IA an IIA stages in which RT will be used as exclusive form of therapy. If RT is considered (no contraindications), then the extent of disease must be determined by staging lap. 25% - downstage (worse disease than anticipated) 15% - upstage (better than anticipated.)

STAGING LAPAROTOMY - splenic hilar, celiac, porta hepatis, mesenteric, peripancreatic, para-aortic, paracaval, iliac nodes. Spleen. Core and wedge biopsies of both lobes of the liver. Iliac crest bone marrow biopsy. +/- oophoropexy. CONTRAINDICATIONS TO STAGING LAPAROTOMY - if radiation is not a treatment option, patients with B symptoms, bone marrow involvement, extranodal involvement, mediastinal involvement, and factors making general anesthesia a risk. Large Mediastinal Mass - should be treated with chemo and radiation, so treatment will not change based on staging lap. So dont do it. Beware of general anesthesia in patient with large mediastinal mass. Do biopsy under local. NONHODGKINS LYMPHOMA - most common type is diffuse histocytic lymphoma (DHL), and is treated with chemotherapy. Do not do staging lap. # 71 - Sarcoma 55 y.o. male presents with a mass on his anterior thigh. On physical examination it is poorly defined but approximately 5 cm in diameter. Issues: a.) What further evaluation is required? (Radiologic evaluation) b.) What type of biopsy is required and how is it performed? Would you get frozen section? What is the grade? c.) How does one perform definitive dissection assuming that this proves to be a sarcoma? d.) What type of margins, etc? e.) Role of postoperative radiation therapy? Role of adjuvant chemotherapy? What is the recurrence rate if clear margins are obtained and postop XRT used? ANSWER: STAGING = BASED ON HISTOLOGY AND CLINICAL (SIZE, NODAL STATUS AND METS) INFO. G1 = WELL DIFFERENTIATED G2 = MODERATELY DIFFERENTIATED. G3 = POORLY DIFFERENTIATED. STAGE I = any well differentiated tumor, no nodes, no mets. (A= < 5cm, B= > 5 cm) STAGE II = any moderately differentiated tumor, no nodes, no mets

(A=< 5cm, B = > 5 cm) STAGE III = any poorly differentiated tumor, no nodes, no mets. (A= > 5cm, B = > 5 cm) STAGE IV = any tumor with either nodes and/or mets. (A= nodes without mets, B= mets with or without nodes) RADIOGRAPHIC EVALUATION: CT SCAN - OF MASS AND CHEST - for low grade lesions, the chance of chest mets is only 15%, so chest X-ray is all that is necessary. For high grade lesions, chest CT is necessary. - for retroperitoneal sarcomas, CT scan of the abdomen should be obtained to rule out metastatic disease of the liver, the most likely site of mets in this case. MRI - can be used in place of CT scan to evaluate the primary lesion to visualize the relationship of the lesion to bone, muscle and vascular structures. May also use both as the studies complement each other. Nodal Mets - poorly assessed by CT or MRI, but not important because the chance f nodal mets in sarcoma is low(4%). The prognostic significance of nodal met is the same as a met in the case of sarcoma (both Stage IV). BIOPSY - incisional biopsy is performed in almost all cases- the exception is small tumors (<35 cm.) which have low risk of malignancy and in which excisional biopsy can be performed without burning bridges for subsequent treatment in case sarcoma is diagnosed. TECHNICAL POINTS - for extremity sarcoma, the incision should be placed along the long axis of the extremity. For retroperitoneal sarcoma, the incision should be placed in such a way that it can be easily excised with resection if sarcoma is diagnosed. The biopsy of the mass should be placed over the portion of the mass that is closest to the surface. Do not disturb tumor planes around tumor, minimize dissection. - sarcoma are often surrounded by a pseudocapsule which is composed of compressed tissue and tumor. Do not enucleate from pseudocapsule as you will leave tumor cells behind. - send frozen section, not for definitive diagnosis, but to get confirmation of adequate tissue and malignancy and to make certain that you have not biopsied the pseudocapsule. - careful hemostasis to prevent hematoma, as tumor cells will disseminate in hematoma. Do not use drains. - FNA will not yield appropriate tissue for tissue type and histologic grade and should not be used. - core needle samples are acceptable if several are provided. - send tissue fresh, get a frozen. SURGERY - KEY FOR LOWER EXTREMITY SARCOMA IS WIDE LOCAL RESECTION FOR LIMB SPARING PROCEDURE. - tumor should not be entered during dissection. If the tumor was small and removed by excisional biopsy (<3-5 cm) and was found to be a malignant sarcoma, they should be operated on for a wide local resection.

- ideal margin is 3-5 cm in all directions. When this is not possible, at least one uninvolved fascial plane should be taken. ? Nervous/Vascular Involvement - should be sacrificed for high grade/poorly differentiated sarcoma, but dissected free of tumor for low grade lesions. I f these structures are taken, the vascular structures can be reconstructed. Resection of nerves can leave a compromised but functional extremity. TUMOR < 10 CM, RESECTABLE ON PREOP STUDIES --> RESECT + RADIATION TUMOR < 10 CM, FIXED SARCOMA, NONRESECTABLE --> RADIATION + CHEMOTHERAPY PREOP FOR LIMB SPARING TREATMENT. TUMOR INVOLVING BONE OR RECURRING AFTER LIMB SPARING ->AMPUTATE Negative margins are necessary. POSTOPERATIVE RADIATION AFTER WIDE LOCAL RESECTION FOR LOWER EXTREMITY SARCOMA. 1.) Low grade sarcoma, small with desirable clean, 3-5 cm margin -> can observe without radiation. 2.) Low grade sarcoma with either clean but close or questionable margins because of proximity to bone, vascular structure or nervous tissue --> RADIATION. 3.) Any intermediate or high grade lesion, regardless of margin ---> RADIATION. Nota Bene - radiation lowers local recurrence but does not improve survival. POSTOPERATIVE CHEMOTHERAPY AFTER WIDE LOCAL RESECTION FOR LOWER EXTREMITY SARCOMA. - since 40% of patients who die of systemic, hematogenous spread, adjuvant chemotherapy makes sense. However, current combinations only slightly improve disease free interval but do not improve survival. Routine adjuvant chemo is not recommended, but should enroll in clinical trial. FOLLOW UP - annual CT scan of primary site and chest for first 5 year for high grade lesions. Low grade lesions should have CT scans of local site. LOCAL RECURRENCE - RECURRENCE RATE FOR EXTREMITY SARCOMA AFTER RESECTION = 15% - for small tumors that were treated with local resection and no radiation, resection and multimodality therapy should be offered. For those previously treated with resection and radiation, radical amputation should be performed at one joint level above lesion. METASTATIC DISEASE

- patients with multiple lesions are still candiate for resection, though those with 3 or less do best. If these lesions are seen before resection of primary, obtain tissue of lung lesion for diagnosis, then plan a staged procedure for resection of primary followed by lung lesions. - if lesions are found on follow-up, resect if possible. Bilateral lesions should be treated with staged bilateral thoracotomy or median sternotomy. SPECIAL SITUATIONS: Distal Extremity, involving neurovascular structure, bone, joint - likely amputate. Proximity to Pelvis - consider hemipelvectomy. Popliteal or antecubital fossa tumor - amputate. Hand - some for of local amputation is recommended. Retroperitoneal Sarcoma - most are very large at presentation (10-20 cm in diameter) and intermediate to poorly differentiated. 79% involve other organs and will need to be resected with specimen. Postop radiotherapy is recommended for these lesions, but has not shown the success of radiation in extremity sarcoma because of difficulty in directing high doses to the right area along with poorer tissue tolerances. Radiate anyway. # 72 - Incidental Ovarian Mass during Upper Abdo Surgery While doing a cholecystectmy you palpate an ovarian mass. Issues: a.) How would you evaluate the mass. b.) At what size would you remove the mass? ANSWER: CYSTIC MASSES: - may be benign (functional cysts) or malignant (mixed cyst) - in general, when an ovarian mass is present, whether cystic or solid, it is removed regardless of size because the only way to exclude cancer is to pathologically examine the entire mass. The exceptions to this rule are (1) the functional ovarian cyst (which presents in women of reproductive age.) and small (< 5cm) asymptomatic, unilateral cysts in postmenopausal women. Functional Cysts - most are corpus luteal cysts. These cysts are unilateral, unilocular and mobile. These cysts will usually resolve over 3-4 menstrual cycles, and can be induced to resolve by treating with oral birth control pills if desired. Cysts larger than 10 cm are less likely to resolve spontaneously. - other functional ovarian masses are theca lutein cysts, pregnancy luteoma, polycystic ovaries. -see Greenfield, p. 2014, Fig. 116-6. NON-CYSTIC OVARIAN TUMORS: - may be benign (serous cystadenoma, mucinous cystadenoma, cystadenofibroma, Brenner tumor), malignant serous cystadenocarcinoma, mucinous cystadenocarcinoma, clear cell CA, endometroid CA, undifferentiated tumor.

Ovarian Malignancy - 80-90% of epithelial origin. Usually present between 50 and 60 yrs of age. 5 yr. survival ranges from 85% for disease localized to the ovary to 15% for advanced disease. Most present with disease outside of the ovary, so early detection and treatment could have impact on survival. - tendency for malignant cells to exfoliate, and are transported via peritoneal fluid flow in clockwise fashion, affecting the right side more often than left. SURGICAL PRINCIPLES: REMOVAL OF MASS (FOR COMPLETE RESECTION OR DEBULKING) AND COMPLETE STAGING. 1.) Removal of Tumor - complete removal is essential, even if only for debulking. Resect any colon or small bowel associated with tumor. Classical debulking resection for biopsy proven cancer includes removal of the uterus, cervix, ovaries, fallopian tubes. Problem - Woman of Child Bearing Age i.) is the tumor malignant or benign? - do not perform a complete debulking operation on a woman of child bearing age based on frozen section. Remove tube and ovary, biopsy contralateral ovary and complete staging. If benign, done. If malignant, all staging done and decision can be made whether pt. is candidate for conservative therapy. ii.) can conservative resection be performed? - if tumor is unilateral, confined to ovary, fully encapsulated, favorable histology, then a unilateral salpingooophorectomy, contralateral ovary wedge biopsy, omental biopsy, liver capsule and right hemidiaphragm. If contralateral ovary biopsy is positive, then a second operation to remove other ovary and uterus must be done. 2.) Staging - biopsy right paracolic gutter, liver capsule, right hemidiaphragm (tendency for peritoneal spread on right because of clockwise rotation of peritoneal fluid), the omentum, paraaortic nodes and any other suspicious areas. Ascites should be collected and sent for cytology. If no ascites is present, irrigate peritoneum with 50 cc of saline along peritoneum, liver and right diaphragm and send for cytology. - tumor must be assessed, resected and staged. Close this incision and make a midline. If the patient is being explored for appendicitis, extend this for exploration. Other situations: 1.) Explore and find abdomen caked with tumor - patient will need debulking operation which may include resection of small bowel and colon. Make sure the patient has had a bowel prep.

# 73 - Evaluation of Mass/Cold Nodule 40y.o. female with cold nodule in the right superior thyroid. PMHx and family history are all negative. Physical exam is negative except for the lesion. Issues: a.) Preop labs, imaging and possible FNA of this lesion.

b.) FNA comes back as follicular cells with suspicion for malignancy. What is you operative procedure? Frozen section shows follicular CA. c.) Definitive procedure for follicular CA? For papillary CA? d.) Describe in detail the surgical technique of thyroidectomy. e.) What do you do if there are also a few palpable nodes? f.) You are called to the bedside and the patient is obtunded (Ca is 6). The patient is cyanotic with a massive hematoma. What do you do? g.) What are the possible complications of total thyroidectomy? h.) How would you evaluate for mets postop? ANSWER: History - although negative - should focus primarily on history of radiation exposure and family history of thyroid cancer or MEN syndromes (II). Exam - negative - but are concerned primarily with thyroid nodule and lymph nodes. Labs: TFTs (most euthyroid), calcitonin level. FNA - TECHNIQUE - patient in the supine position. Nodule site is prepped with alcohol. 27 gauge needle is placed in the lesion and plunger pulled back. With plunger pulled back. Place on slide and fix. Cystic lesions should be aspirated. Large lesions suspicious for anaplastic CA or lymphoma should have larger specimens taken via a TruCut biopsy. Results - suggestive of malignancy, suspicious for malignancy, benign or inadequate specimen. FNA is extremely accurate for diagnosing medullary or papillary CA, but is difficult to diagnose follicular CA because of criteria for angioinvasion which cannot be diagnosed. TREATMENT 1.) Neck exploration. 2.) Exploration and evaluation of both thyroid lobes. 3.) Lobectomy on affected side and isthmusectomy. 4.) Frozen section on lesion. 5.) Benign ----> close. 6.) Malignant... TREATMENT OF FOLLICULAR CA OF THYROID - the pathologic criteria for distinguishing malignant from benign is the presence of capsular or angioinvasion of the follicular cells (these features cannot be seen on FNA, only the follicular cells can be seen.) Hurthle Cell CA - Hurthle cells are oncocytes, cells that are derived from follicular cells, that appear as large bizarre, polygonal shaped cells with hyperchromatic nuclei. Hurthle cells may be present in Hurthle cell adenomas or in Hurthle cell carcinoma. Hurthel cell Carcinoma is differentiated from a benign adenoma by capsular or angioinvasion of Hurthle cells (as differentiating benign from malignant follicular cell tumor.) - these tumors (both Hurthle cell and follicular CA) tend to have distant mets via hematogenous spread to lung, bones and brain rather than to the lymph nodes via lymphatic spread. OPTIMAL SURGICAL TREATMENT - TOTAL THYROIDECTOMY.

- at minimum, a near total thyroidectomy should be performed. POST OPERATIVE CARE - total or near total thyroidectomy decreases the sink effect of normal thyroid uptake of I-131. Postoperative I-131 scanning should be performed for detection of metastasis. If mets are detected, therapeutic I-131 can be used. I-131 treatment does not work as well for Hurthle cells because of their poor ability to take up iodine, but should be used anyway. TREATMENT OF PAPILLARY CA OF THE THYROID. - RECURRENCES FOR PAPILLARY CANCER TEND TO OCCUR AT THE EXTREMES OF AGE, 40% under 20 yrs. and 30% over 59 yrs. MIXED PAPILLARY - FOLLICULAR CARCINOMA = should be treated as a pure papillary carcinoma. PAPILLARY CA - CLASSIFICATION 1.) MINIMAL OR OCCULT - measure less than 1.5 cm, are usually discovered either incidentally during thyroid resection for other benign condition or after diagnosis of lymph node met as a lateral aberrant thyroid rest. Extremely rare to die of a minimal papillary thyroid cancer. 2.) INTRATHYROID - measure greater than 1.5 cm but are still isolated to the thyroid gland and show no extension outside of the thyroid. 3.) EXTRATHYROIDAL - invade the thyroid capsule and grow into contiguous structures. ** multicentricity is common in papillary cancer. Cervical mets are also common in papillary cancer, seen in up to 40%. Extranodal mets are seen in 10%. LOBECTOMY + ISTHMUS - proponents state that the survival results are the same as a total lobectomy while the complication rates are lower in the case of minimal thyroid cancer (small and without extension outside of the capsule). A papillary cancer in the contraleateral lobe diagnosed later can easily be removed. - the need to remove the remaining thyroid for the purposes of using diagnostic or therapeutic I131 is unnecessary since the course of this cancer is so benign that I-131 is really not needed. TOTAL LOBECTOMY - effective at eliminating multicentric bilateral tumor foci and preventing contralateral recurrence. Allow I-131 to be used for both diagnostic and therapeutic reasons. Also allows the use of thyroglobulin as a marker for recurrence. Should be done by experienced thyroid surgeon to decrease incidence of complications. RECOMMENDATIONS FOR TREATING PAPILLARY THYROID CA:

1.) INCIDENTAL MINIMAL THYROID CA - subtotal thyroidectomy. Should be placed on TSH thyroid hormone suppression and followed closely. 2.) OCCULT MINIMAL PAPILLARY CA WITH LATERAL ABERRANT THYROID REST - should undergo total or near total thyroidectomy with lymph node dissection so that the patient can be treated with radioactive I-131 therapy. 3.) INTRATHYROID (> 1.5 CM) OR EXTRATHYROID PAPILLARY CA - should undergo total or subtotal throidectomy. LYMPH NODE METASTASIS - first mets go to peritracheal nodes and enbloc dissection of these nodes should be taken with the initial operation. Modified radical neck dissection should be performed. POST OPERATIVE CARE - serum calcium and phosphorus drawn at 5 hrs and the following morning. If papillary Ca is confirmed by path, do not give thyroxine postop, hold until the patient is slightly hypothyroid and the TSH rises to 35. Perform I-131 scan at about 2 weeks. If mets are seen or there is residual thyroid tissue left over, proceed with I-131 ablation. The patient is then treated with TSH suppressive doses of thyroxine. - follow every 6 months for first 2 years. - if the thyroglobulin level rises, thyroxine is held to allow TSH to rise so that I-131 can be used for ablation. LOBECTOMY AND SUBTOTAL THYROIDECTOMY - TECHNIQUE - supine position, roll between shoulders, occiput padded. Tranverse 1.5-2.0 cm incision 2 finger breadths above the sternal notch. Divide skin, subQ fat and platysma. Superficial veins are ligated. Superior and inferior skin flaps are raised superficial to the level of the strap muscles. Linea alba divided between the strap muscles. The anterior/superficial surface of the thyroid is dissected off of the posterior strap muscles using blunt dissection. This dissection is carried laterally to separate the lateral thyroid from the carotid sheath. Middle thyroid vein is found and ligated. The thyroid lobe is rotated medially exposing the superior and inferior pole vessels. - grasp the thyroid just below the vessels with a Kocher clamp and retract inferiorly. Identify the superior laryngeal nerve if possible and ligate the superior pole vessels flush with the thyroid gland using 4-0 silk. (This technique will lower the risk of injury to the superior laryngeal nerve. The gland can now be retracted even further medially and the recurrent laryngeal nerve can be found in the tracheoesophageal grove beneath the inferior thyroid artery. Inferior thyroide vein branches are taken. The nerve is identified and separated from the thyroid gland. Once the nerve is freed, the small vessels can be taken. Bleeding at this point is controlled with thrombin and gelfoam and pressure. Caution is taken to avoid damaging blood supply to the parathyroids and minimizing over dissection of the parathyroids. If the parathyroid is identified and not part of the diseased thyroid, it should be left as is. If the parthyroid is devascularized, it may be diced into 2-3 mm pieces and placed in the forearm or pocket of sternocleidomastoid. Mark with vascular clip. The thyroid attached to the pretracheal fascia is divided with cautery. Send the lobe for frozen. SUBTOTAL THYROIDECTOMY - lobectomy on one side and mobilization of the thyroid from the trachea on the opposite side. The course of the recurrent laryngeal nerve is identified along with the parathyroid glands. Place hemostats on mobilized thyroid to leave 1 cm rim of tissue. Avoid parathyroids and recurrent laryngeal nerve. leave the inferior thyroid artery to lower the risk to the recurrent laryngeal n.

TOTAL THYROIDECTOMY - repeat steps for thyroid lobectomy on opposite side. COMPLICATIONS OF TOTAL THYROIDECTOMY 1.) DAMAGE TO RECURRENT LARYNGEAL N. - it is possible to damage a non-recurrent laryngeal nerve, which can only occur on right. 2.) DAMAGE TO PARATHYROIDS - should check phosphorus level 5-8 hrs. postop, if low there is little concern. If the level is rising (> 4.5), there is concern about permanent hypocalcemia. Have Ca gluconate at bedside. 3.) DAMAGE TO SUPERIOR LARYNGEAL NERVE. 4.) HEMORRHAGE - with loss of airway. Evacuate at bedside if necessary. # 74 Medullary Carcinoma of the Thyroid (MTC) Issues: a.) How would you work up a patient with a suspected MCT prior to resection? What is your surgical resection of choice? How would you evaluate the family members? ANSWER: MCT - arise from C cells or parafollicular cells which produce calcitonin. In their more aggressive form, they show evidence of lymphatic or vascular invasion early, metastasizing to perithyroid and paratracheal nodes to nodes of the jugular chain and upper mediastinal nodes. Familial Syndromes - 75% of cases of MCT are sporadic and 25% are associated with either MEN 2a, MEN 2b or familial non-MEN MCT. - those with MEN- 2A can also develop parathyroid hyperplasia and pheochromocytoma. While MEN 2B can often be diagnosed by multiple mucosal neuromas, the outward phenotype of MEN -2A cannot be detected. The only way MEN-2A can be detected is by the presence of endocrine neoplasia. - diagnosis of MEN-2A and MCT can be made by screening for calcitonin levels. Pheochromocytomas may be detected by measurement of urinary catecholamines and metanephrines. Hyperparathyroidism is detected by measuring serum calcium levels - typically those with palpable disease will typically present with nodal mets. - nonfamilial cases are usually unifocal and unilateral, MCT associated with familial syndromes are usually multicentric and bilateral.

INITIAL DIAGNOSIS - palpable mass - FNA cytology can be done but thyroid lobectomy with histologic diagnosis. All patients suspected of MCT should have CEA levels and stimulated calcitonin levels for screening. Serum calcium should be checked to rule out parathyroid hyperplasia. 24 hr. urine collection should be done for VMA, catecholamines and metanephrines. Even those with sporadic MCT should have screening.

STIMULATED CALCITONIN - infuse Calcium= 2mg/kg/min. followed immediately by Pentagastrin 0.5 microgram/kg/5 sec. with blood draws at 1,2,3, and 5 minutes. SURGICAL TREATMENT TOTAL THYROIDECTOMY and CENTAL NODE DISSECTION - from the hyoid bone superiorly to the innominate vessels inferiorly. Laterally, the nodes and thyroid tissues from the carotid sheath to the carotid sheath. Those with large masses in the thyroid (> 2cm) or palpable paratracheal or jugular nodes should have a modified radical neck dissection on that side. Generally when MCT is resected along with central nodes, the parathyroids must be taken to clear the specimen. these parathyroids should be minced and autotransplanted into the forearm. RECURRENCE/PERSISTENT ELEVATIONS OF CALCITONIN - many have persistent elevations of calcitonin after adequate resection with presence of nodal mets. Recommend observation in absence of clinical disease. ?RADIATION - I-131 is not effective because C cells are not derived from follicular cells and will not concentrate iodine. External beam radiation is not beneficial. ?CHEMOTHERAPY - mixed results but not shown to be effective. SCREENING FOR MCT - CALCITONIN LEVELS # 75 - Insulinoma 14 y.o. female with behavioral changes over the past several months, manifested by psychotic episode s and blackouts. Issues: a.) What is the differential diagnosis? b.) How would you work up this lesion to confirm the diagnosis? c.) Assuming this is an insulinoma, what is the chance that this is chance that this is malignant? an adenoma? diffuse benign islet cell hyperplasia/nesidioblastosis? d.) What are the localization modalities and when are they indicated? What do you do if you are not able to localize the lesion preoperatively? What are intraoperative methods of tumor localization? What if no tumor can be localized? How can you confirm diagnosis of hyperplasia? What do you do if the patient has hyperplasia and has responded to diazoxide? What do you do if the patient has not responded to diazoxide and has hyperplasia?

ANSWER: DIFFERENTIAL DIAGNOSIS: Factitious Insulin Administration - usually occurs in typically whacked out chick. Hepatic Insufficiency Mesothelioma Functional Hypoglycemia Insulinoma Nesidioblastosis - hyperplasia of the Islet cells in pancreas, typically in infants. DIAGNOSIS: based on symptomatic hypoglycemia and hyperinsulinism. SUPERVISED FAST AND SERIAL SERUM GLUCOSE AND INSULIN LEVELS - patient placed in hospital with heplock in place. Fast for up to 72 hrs. Draw blood for insulin and glucose measurements before fast starts and every 6 hrs. When neuroglycopenia (symptoms suggesting hypoglycemia) develops, glucose, insulin, C-peptide and proinsulin levels are drawn. All patients will develop symptoms of hypoglycemia in 72 hrs., those without insulinoma will not. RESULTS: GLUCOSE - all will have glucose levels 45-50. INSULIN - all will have elevated levels for degree of hypoglycemia (>5) C-Peptide - elevated level indicates that the tumor is producing insulin and it is not being given from surreptitious source. Proinsulin - insulin precursor - produced in excess by insulinoma. ?Borderline levels = can use provocative agents such as Calcium, tolbutamide and glucagon to stimulate insulin levels, but theist tests are not used routinely for diagnosis. CONTROLLING HYPOGLYCEMIA - ability to control the hypoglycemia dictates the importance of removing the tumor with the first operation. I f the tumor has not been localized well prior to surgery, but the symptoms are well controlled with diet and medications, a less aggressive operation may be performed. however, if the tumor has not been localized well and the symptoms are not under control, the need to remove the tumor with the first operation is critical and a more aggressive approach must be taken with the first operation.

DIAZOXIDE - can be used to control the insulinoma syndrome, by dilating resistance vessels of the islets and decreasing insulin secretion. Must be stopped 1 week before surgery because of the risk of severe hypotension intraop DIET - should be used as primary form of treatment for symptoms. larger and more frequent meals. PATHOLOGY: MALIGNANT - 10% BENIGN - 90% (10% associated with MEN I and will have multiple tumors and hyperplasia, 80 (+) % associated with adenomas) PREOPERATIVE RADIOGRAPHIC LOCALIZATION: PROBLEM - tumors are small, like gastriinomas, and difficult to localize. Even with invasive imaging techniques, localization of tumors is only about 50%. However, unlike gastrinomas (in which most of the tumors can be localized within the gastrinoma triangle) insulinomas are uniformly distributed throughout the head, body and tail. -> even if tumor cannot be localized, it is important to try to isolate the lesion to one portion of the gland so that surgical excision can be performed to remove the tumor without have to do a blind total or subtotal pancreatectomy. IMAGING MODALITIES 1.) CT, US, MRI - because tumors are so small (< 2cm), it is unlikely that any of these modalities will be able to localize the tumor. However, one of these should be done to make sure there is not a large tumor or numerous mets. 2.) SELECTIVE CELIAC ANGIO - sensitivity was thought to be high, but studies show it is only about 50%. 3.) PORTAL VENOUS SAMPLING - for insulin - it does not localize the tumor, it does suggest which part of the pancreas contains the tumor. Most sensitive modality (82%). 4.) CALCIUM ANGIOGRAPHY - variation of secretin angio used for gastrinomas but uses Ca as the provocative agent. In this modality the calcium is injected into each selectively catheterized artery and plasma samples are collected from the hepatic veins for determination of insulin concentration. LIMITED DATA, BUT BEST METHOD FOR DIAGNOSING INSULINOMA AND LOCALIZATION OF PORTION OF PANCREAS. 5.) ENDOSCOPIC ULTRASOUND - using a high frequency transducer, tumors as small as one cm can be localized. SEQUENCE OF TESTS:

CT SCAN WITH IV CONTRAST - INVASIVE LOCALIZATION STUDIES - USE EITHER PROTAL VENOUS SAMPLING OR CALCIUM ARTERIOGRAPHY.(provide correct localization in 80-100% of patients.) PREOP PREPATION - give pneumovax, bowel prep and periop antibiotics EXPLORATION FOR INSULINOMA - enter abdomen through a bilateral subcostal incision and place self retaining retractors. Dont expect to find tumor outside the pancreas be cause of the low risk of malignancy. Mobilize the splenic and hepatic flexures of the colon. Divide the gastrocolic ligament to expose the body and tail of pancreas. Wide, complete Kocher maneuver to examine the posterior head of the pancreas. Mobilize the entire inferior border of the pancreas to be able to palpate the body and tail of the gland between the thumb and forefinger. May need to take short gastric vessels and mobilize the spleen to adequately assess the tail of the pancreas. POTENTIAL PROBLEM - if the tumor is not localized at this point, excessive biopsy of the pancreas is likely to lead to bleeding, fistulas and abscesses. VISUALIZATION - tumors are reddish brown. PALPATION - generally sensitive method, but can easily miss tumors in the head because of the thickness of the gland here. INTRAOP ULTRASOUND - can not only localize tumor but also give anatomic detail of the relationship of the tumor to the SMV, portal vein, etc. RESECTION/REMOVAL - if possible (as in the likely case of the benign tumor) the mass is enucleated and sent to path for frozen identification of an islet cell tumor. LOCALIZED ADENOMA - ENUCLEATE AND CLOSE LOCALIZED ADENOMA WITH MEN SUBTOTAL PANCREATECTOMY - because of the high rate of multiple tumors. LOCALIZED CARCINOMA - SUBTOTAL PANCREATECTOMY (FOR TUMORS IN THE BODY OR TAIL) OR WHIPPLE (FOR TUMORS OF THE HEAD) LOCALIZED CARCINOMA WITH METASTASIS - RESECT FOR PALLIATION. UTILIZE MEDICAL THERAPY (DIAZOXIDE) NONLOCALIZED WITH GOOD CONTROL OF HYPOGLYEMIA - may opt for following and allowing the tumor to grow and plan for resection at second operation. May biopsy the pancreas to rule out the rare conditions of beta cell hyperplasia or adult nesidioblastosis. Blind subtotal pancreatectomy has only 50% chance of removing the tumor.

NONLOCALIZED WITH POOR CONTROL OF HYPOGLYCEMIA - SUBTOTAL PANCREATECTOMY. BETA CELL HYPERPLASIA/NESIDIOBLASTOSIS - SUBTOTAL PANCREATECTOMY.

Scenario # 76 - Aldosteronoma 38 y.o. female with h/o SVT and negative PMHx. BP = 200/110 and found on routine blood work to have a potassium of 2.0. Issues: a.) Assuming that the patient has hyperaldosteronism, how can you rule out secondary causes? b.) What tests will confirm the diagnosis of aldosteronoma? c.) Assuming the patient has an aldosteronoma, how will you differentiate hyperplasia from an adenoma? How will you localize the tumor? What are treatment options? ANSWER: PRIMARY HYPERALDOSTERONISM - results from excess secretion of aldosterone from the zona glomerulosa of the adrenal cortex. Caused by an adrenocortical adenoma (60%) or from adrenocortical hyperplasia (40%). SECONDARY HYPERLDOSTERONISM - cirrhosis, pituitary adenoma, renal artery stenosis, etc. -important to differentiate primary from secondary causes of hyperaldosteronism and adrenal cortical hyperplasia from adenoma because adenoma is amenable to surgical resection and hyperplasia is not treated with surgery. DIAGNOSIS OF HYPERALDOSTERONISM - Most effective test is to see nonsuppressibility of aldosterone secretion during a period of salt loading. TECHNIQUE - infuse 2000 cc of Normal Saline over 4 hrs. If the plasma aldosterone level is not suppressed to less than 10ng%, it is likely primary hyperaldosteronism. The presence of hypokalemia and suppressed renin activity add further support to this diagnosis. CAPTOPRIL CHALLENGE - captopril will replace NaCl infusion as a means of lowering the renin level and ultimately the aldosterone level by inhibiting the angiotensin converting enzyme. DIFFERENTIATING ADENOMA FROM HYPERPLASIA - can differentiate adenoma from hyperplasia by postural changes. UPRIGHT POSTURE > 4HRS -> no change, decreased aldosterone = ADENOMA UPRIGHT POSTURE > 4 HRS-> increased aldosterone =HYPERPLASIA

CT SCAN - can reliably localize adenomas > 1.5 cm. ADRENAL VEIN RENIN SAMPLING - 100% EFFECTIVE at differentiating adenoma from hyperplasia and also localizes side. This method is invasive and should be used only in the event that preop tests suggest an adenoma but the CT scan fails to localize adenoma to one side. TREATMENT OF PRIMARY HYPERALDOSTERONISM - UNILATERAL ADRENALECTOMY FOR ADENOMA AND SPIRONOLACTONE FOR HYPERPLASIA. SURGICAL APPROACH - FLANK INCISION # 77 - Pheochromocytoma 28 y.o. female swimmer gets headache after workouts. BP = 220/120 after workout. Normally the patient has a BP of 130/80. CT scan shows a right adrenal mass. Pt is referred to you. Issues: a.) What is the differential diagnosis, i.e. surgically correctable causes of hypertension? b.) How would you workup this patient? what are the localization modalities? When is an MIBG scan indicated? c.) How would you prepare this person for the operating room? d.) Describe how you would approach this person surgically? What type of incision? Unilateral or bilateral exploration? How would you approach the mass? How would you improve the exposure on the right or left? how would you control blood pressure intraoperatively? What if you remove the mass and the patients BP does not improve? ANSWER:

DIFFERENTIAL DIAGN0SIS: 1.) PHEOCHROMOCYTOMA 2.) COARCTATION OF THE AORTA 3.) RENAL ARTERY STENOSIS 4.) ALDOSTERONOMA DIAGNOSIS - most effective means is by measuring the catecholamine levels and levels of metabolic products in the urine.

24 hr. Urine Collection - for either catecholamines or metanephrines. Levels elevated above normal limit for each test will yield a sensitivity of 95%. The two tests combined will yield a sensitivity of 98%. Addition of urinary VMA can yield almost 100% sensitivity. Important for collection to be started close to an episode of symptoms or documented hypertension as those who have pheochromocytoma and are episodic secretors may not demonstrate elevated levels if the collection is not started at the time of documented HTN/symptoms. Urine vs. Plasma measurements - collection of urine over 24 hrs. smoothes out all the variations, peaks and troughs of catecholamine secretion which occur naturally. Plasma measurements capture only one point in time and make it difficult to differentiate a normal surge of catecholamine from a minimally secreting pheochromocytoma. LOCALIZATION STUDIES CT SCAN - can reliably detect tumors of 1 cm in size 80% of time and pick up tumors 3-4 cm 100% of time. Problem is that it cannot differentiate benign from malignant tumors and functional from non-functional tumors. Overall, CT scan will localize 96% of pheochromocytomas. MIBG SCAN - should be used if CT scan does not show adrenal mass or if there is concern about metastasis. Useful for the diagnosis of mets or in localizing an extraadrenal pheochromocytoma. Nota Bene - use of IV contrast in CT scan or through the use of arteriography can precipitate a hypertensive/hypercatechol crisis in those with pheochromocytoma and should be avoided. Contrast may be used if the unenhanced CT does not show tumor, but only after the patient has been adequately alpha adrenergically blocked. PREOPERATIVE PREPARATION: 1.) ALPHA BLOCKADE - PHENOXYBENZAMINE - MUST BE USED FIRST - start with 20 mg per day and increase by 10 mg per day increments until the BP is controlled with patient in the erect and supine position. This should be done such that the drug is on board 710 days before surgery. This drug is stopped at midnight prior to surgery. Slow progressive alpha blockade will allow volume expansion over the week preop and will obviate need for fluid and prevent hypotension intraop. 2.) BETA BLOCKADE - PROPRANOLOL - START 48 HRS. BEFORE SURGERY. - start at 30-40 mg per day. Last dose is given 1 hr. preop with sip of water. INTRAOP BP CONTROL MONITORING - at least central line and arterial line. PA catheter should be used selectively in those with cardiac history and all over 60 yr. of age. BP CONTROL - PHENTOLAMINE AND NITROPRUSSIDE - should be used intraop for BP control. Should use lidocaine and propranolol for arrhythmias. ANESTHESIA - use enflurane instead of halothane be cause of arrhythmogenic properties. SURGERY ?POSTERIOR APPROACH - should be avoided because it necessitates too much manipulation of the tumor and gives poor visualization of the adrenal vein, both things needed in removing a pheochromocytoma. INCISION - LONG MIDLINE OR BILATERAL SUBCOSTAL - place self retaining retractor. EXPOSURE

- RIGHT ADRENAL - take down the hepatic flexure of the colon and Kocherize the duodenum. Use left hand to retract the right kidney caudad. Divide Gerotas fascia without detaching adrenal gland. The liver can be mobilized by dividing the right coronary ligament and reflecting the liver superiorly and to the midline. The right adrenal vein is broad and short and often enters the IVC posteriorly. Hemoclips may be used here to avoid avulsing the vein. - LEFT ADRENAL - divide the gastrocolic omentum to enter the lesser sac. Divide the peritoneal covering of the inferior border of the pancreas and divide the fatty tissue for superior rotation of the pancreas. The spleen can be mobilized completely and developing plane between the posterior aspect of the pancreas and the anterior surface of Gerotas fascia. The left adrenal vein is longer and narrower as it enters the left renal vein. This should be tied. - RESECTION - after ligation of the vascuar supply, the entire adrenal and margin of adjacent fat should be removed. Avoid rupture of the capsule as this can predispose to recurrence. HYPOTENSION AFTER RESECTION - expected. If there is persistent hypertension after the adrenal is removed, search should be made for second pheochromocytoma in the abdomen (contralateral adrenal gland or in the organ of Zuckerkandl.) REMEMBER - 10% ARE BILATERAL - 10% ARE EXTRA-ADRENAL - 10 % ARE MULTICENTRIC BILATERAL PHEOCHROMOCYTOMA - necessitates bilateral adrenalectomy PALLIATION - since mortality with these tumors is related more to excess catecholamine secretion and less to tumor spread, attempts should be made to resect as much tumor as possible in the case of metastatic disease. This may make the hypercatecholamine syndrome easier to control with phenoxybenzamine. POSTOPERATIVE CARE - patient should remain in ICU for 24 hrs for BP monitoring. Catecholamine levels should be checked before the patient leaves hospital. Most patients will become normotensive, but 25-30% will remain hypertensive postop because of development of essential hypertension or renovascular changes. MEDICAL THERAPY - for those who cannot tolerate surgery or those who have disease that cannot be resected, continuous alpha and beta blockade with propranolol and phenoxybenzamine. # 78 - Incidentaloma 50 y.o. white male is evaluated for right hypochondriac pain with an ultrasound. Normal gallbladder, but a 4 cm mass in the right adrenal gland. No symptoms.

Issues: a.) What is the differential diagnosis of this mass. b.) What tests would you want to order to narrow your differential diagnosis? Would you biopsy the lesion if you were to decide to follow this lesion? c.) Assuming all tests come back negative, at what size would you operate on an incidentaloma? d.) How would you approach this? (Posterior, flank, transabdominal). ANSWER:

DIFFERENTIAL DIAGN0SIS: 1.) Adrenal Cortical Adenoma 2.) Adrenocortical Carcinoma 3.) Pheochomocytoma 4.)Ganglioneuroma 5.) Cyst 6.) Organized Hemorrhage or Fibrosis 7.) Myelolipoma 8.) Adenolipoma 9.) Metastasis from nonadrenal primary (e.g. lung) LABORATORY TESTS METASTASIS FROM MALIGNANCY - stool for occult blood - CXR - Mammogram (in woman) HORMONAL TESTING (in asymptomatic patient) - serum potassium (rule out aldosteronoma) - if this is decreased, then a 24 hr. collection for aldosterone should be done. If this is found to be elevated, then a serum renin should be performed. Hypokalemia, elevated aldosterone in 24 urine collection and decreased serum renin confirms aldosteronoma. - 24 hr. urine collection for catecholamines, VMA and metanephrines. (rule out pheo)- this alone will generally diagnose pheo. - 24 hr. urine collection for 17- hydroxysteroids and 17-ketosteroids (rule out Cushings disease) - this can be followed by a Dexamethasone suppression test if these levels are elevated in 24 hr. urine collection. High Dexamethose should act on pituitary to decrease 17hydroxycorticosteroid levels in urine after administration. 17-hydroxycorticosteroid levels will remain elevated in patient with autonomously functioning adrenal tumor. This test should be confirmed with a decreased serum ACTH level. - elevated 17- ketosteroid levels will generally demonstrate some signs of feminization or masculinization but should be confirmed with serum estrogen and testosterone levels. CYSTIC ADRENAL MASS - a cystic adrenal mass measuring 3-6 cm b y CT and having a thick wall should be percutaneously biopsied. Fluid from this cystic structure should be sent for cytology. Malignant cytology necessitates resection. ADRENAL MASS IN PATIENT WITH HISTORY OF MALIGNANCY - tend to come from melanoma, renal cell CA, breast, lung and gastric CA. In patients with history of malignancy, the mass should be confirmed by percutaneous needle biopsy. Before

undergoing percutaneous biopsy, the patient should be screened with urinary catecholmines, VMA and metanephrines to avoid a hypertensive crisis if this turns out to be a pheochromocytoma. - while a percutaneous needle biopsy can distinguish a metastasis from a primary adrenal tumor (malignant or benign), it cannot differentiate a malignant from a benign adrenal tumor. MANAGEMENT OF INCIDENTAL ADRENAL MASS CRITERIA FOR RESECTION: 1.) AGE < 50 AND MASS 3 CM OR LARGER. 2.) MASS 3-6 CM WITH SUSPICIOUS CT CHARACTERISTICS 3.) RESECT ALL MASSES 5 CM OR GREATER - because of greater risk of malignancy. 4.) AGGRESSIVE APPROACH - ANY GREATER THAN 3 CM. CRITERIA FOR OBSERVATION: 1.) AGE OF 50 YRS. OR OLDER AND MASS 3-5 CM. 2.) ALL MASSES LESS THAN 3 CM AND NOT HORMONALLY ACTIVE. OBSERVATION - CT Scan every 3 months for first year and annually thereafter. Along with CT scan, patient should have serum potassium, 24 hr urine collection for VMA, metanehrines and catecholamines along with 17- ketosteroid and17- hydroxycorticosteroids. SURGICAL APPROACH - transabdominal approach is best to be able to examine the abdomen for evidence of other primary tumor (making the adrenal tumor a met) or for staging the abdomen for mets if this adrenal mass turns out to be an adrenocortical carcinoma. - see algorithm in Surgical Clinics, June 95, p. 508. # 79 - Severe Hypercalcemia 50 y.o. male presents with lethargy progressing to unresponsiveness. Issues: a.) What is your initial ER workup to determine the cause of his unresponsiveness? (cbc, Ca= 16.5, sodium = 145, pH=7.2, glucose =110) b.) What are the possible causes of hypercalcemia? What other lab tests would you want to narrow the differential? (PTH level, Cl-/PO42- ratio) What radiographic tests would help narrow the differential diagnosis? The patient has a right neck mass on physical examination. c.) What is the medical management of this patient? What is the tentative diagnosis and how would you plan to treat this? What are the percentages of this being an adenoma? hyperplasia/ multiple adenomas? parathyroid CA?

d.) What is the surgical approach to this problem? Incision, site of exploration, chance of more than one adenoma? Where would you look for ectopic parathyroid glands? What would you do if three normal glands are located? Would you perform a median sternotomy if the adenoma has not been found? How would you monitor the patients Ca++ level postop? ANSWER: INITIAL WORKUP: 1.) CBC 2.) ELECTROYTES WITH GLUCOSE 3.) LFTS WITH AMMONIA AND CALCIUM 4.) ABG 5.) HEAD CT CAUSES OF HYPERCALCEMIA: 1.) HYPERPARATHYROIDISM - parathyroid adenoma, parathyroid hyperplasia, parathyroid carcinoma. 2.) MALIGNANCY/DIFFUSE BONY METASTASIS 3.) MULTIPLE MYELOMA 4.) THIAZIDE DIURETICS 5.) HYPERTHYROIDISM 6.) MILK-ALKALI SYNDROME - those using milk and antacids for treatment of ulcer. 7.) SARCOIDOSIS 8.) PROLONGED IMMOBILIZATION WORKUP OF HYPERCALCEMIA: 1.) PRIMARY HYPERPARATHYROIDISM - Calcium, albumin, phosphate, chloride, Parathormone Level Calcium, albumin - need albumin to calculate the actual Ca as patients with slightly elevated calcium may have very low albumin. Alternatively, obtain ionized calcium. Chloride/Phosphorus Ratio - greater than 33 is virtually diagnostic of primary hyperparathyroidism. Parathormone level - combination of elevated parathormone level with elevated Ca is almost diagnostic of primary hyperparathyroidism. HAND X-RAYS - show the subperiosteal bone resorption in the middle and terminal phalanges of digits. BONE DENSITOMETRY 2.) MALIGNANCY/DIFFUSE BONE METS- in cases of diffuse metastatic malignancy causing hypercalcemia, the phophate level will not be depressed as in primary hyperparathyroidism. BONE SCAN CXR LOCALIZATION OF PARATHYROID TUMOR

- in the hands of an experienced endocrine surgeon, the success rate with initial neck operation is 95%. However, it is sometimes difficult to differentiate single gland from multiple gland disease and to distinguish a normal from an abnormal gland. Preoperative localization should definitely be performed in cases of recurrent or persistent hyperparathyroidism after an unsuccessful first operation. MODALITIES: US/CT SCAN - sensitivities of 50-60% because tumors are small. THALLIUM/TECHNITIUM - Thallium taken up by thyroid and parathyroid, Tech taken up by thyroid. Subtract the 2 images. Useful for inferior pole and mediastinal tumors. DSA - sensitivities of 50-60%. Parathyroid Venous Sampling - may be difficult in case of previous surgery as the veins may have been ligated. Has sensitivity of 80%. Should try SSA or DSA prior to attempting parathyroid venous sampling. TREATMENT OF SEVERE HYPERCALCEMIA - though surgery for severe hypercalcemia is the ultimate therapy, it is unwise to proceed to neck exploration until the calcium level has decreased. - Admit to ICU for monitoring. Place central line. Place Foley catheter FLUID - Normal Saline - 1L every 3- 4 hrs. (250-300cc/hr.) DIURETIC - Lasix - 100 mg/hr. ---> want to ensure a urine output of at least 100 cc/hr. May need to fluid resuscitate prior to administering Lasix. Careful of pulmonary edema in GOMERS. CALCITONIN - 1-5 units/kg body weight per day IV - weak agent but works quickly. MITHRAMYCIN - 25 micrograms/kg body weight - useful in malignancy but can cause thrombotic disorders and liver damage/failure. PROBLEM - elevated calcium levels from hyperplasia or benign adenoma will often decrease with medical therapy but those with hypercalcemia from tumor (parathyroid or other ectopic parathyroid secreting malignancy) often will not respond to medical therapy. CAUSES OF PRIMARY HYPERPARATHYROIDISM: 1.) ADENOMA - 85% 2.) HYPERPLASIA - 14% 3.) CARCINOMA - 1% SURGICAL THERAPY FOR PRIMARY HYPERPARATHYROIDISM: INITIAL OPERATION:

PREOP PREPARATION - assess vocal cord function preop. Standard collar incision is made. Subcutaneous fat and platysma are divided. Superficial veins are ligated. Tissue plane superiorly and inferiorly created. Strap muscles divided through the linea alba. The posterior strap muscles dissected from the anterior surface of the thyroid through loose areolar tissue. Middle thyroid veins are ligated. The thyroid lobes are mobilized and rotated medially. The recurrent laryngeal nerve on each side is visualized. ADENOMA - REMOVE, OTHER GLANDS NOT DISTURBED. MICROADENOMA - REMOVE, BIOPSY GLAND ON SAME SIDE - to make sure that this gland is not an adenoma or hyperplastic gland. HYPERPLASIA - REMOVE ALL GLANDS AND AUTOTRANSPLANTATION. 2 ENLARGED GLANDS + 2 NORMAL GLANDS - remove the enlarged glands and leave the normal glands. 3 ENLARGED GLANDS + 1 NORMAL GLAND - remove enlarged glands and leave the normal gland. ONLY 3 GLANDS IDENTIFIED: SUPERIOR GLAND ECTOPIC SITES - within thyroid gland, retroesophageal (posterior mediastinum), carotid sheath, superior thyroid pedicles. INFERIOR GLAND ECTOPIC SITES - inferior pole of thyroid, within the thymus, within the thyroid, superior/anterior/posterior mediastinum. 3 NORMAL GLANDS + NO GLAND IN ECTOPIC SITES - perform thyroid lobectomy on side of missing gland. GLAND IN MEDIASTINUM 1.) Suspected Gland in the Mediastinum - can mobilize the thyrothymic ligament through the cervical incision to pull the thymic remnant into the incision for inspection. 2.) Median Sternotomy - because of high morbidity associated with median sternotomy, the following criteria should be met: I.) previous neck exploration by experienced surgeon. II.) preop studies showing suspicious area in chest/mediastinum. PARATHYROID CARCINOMA - rare. Usually have very high Calcium and palpable neck mass. Perform en bloc resection of carcinoma and ipsilateral thyroid lobectomy. perform radical neck dissection for any palpable nodes. POSTOPERATIVE HYPOCALCEMIA - expect the Ca level to drop by 1-2 per day until it levels off. - need to check for symptoms at least 2X per day Chvosteks - tap facial nerve anterior to the ear. Trousseauss - carpal spasm after inflating BP cuff above systolic pressure for 2-3 minutes. - check calcium twice per day for first three days. Symptoms or Ca < 7.5 - give 2-3 amps 10% Ca Gluconate (Ca chloride will burn the veins) no faster than 2ml per minute. This will likely be transient hypocalcemia that will resolve when the other glands become reactivated - no further therapy is needed.

Persistent Hypocalcemia - start oral calcium at 1 gram of calcium carbonate per day. When the patient is asymptomatic and the Ca level has stabilized, the patient can be discharged. The oral calcium can be stopped after several weeks. It is preferable not to add vit. D therapy unless permanent hypocalcemia is suspected or if an autotransplant into the forearm has been performed. If this doesnt work, add vit D. Permanent Hypocalcemia - vit. D 50,000 - 100,000 units per day along with 2 gr of Ca Gluconate. Check Ca and phosphorus levels 2X/month.

VASCULAR
# 79 - Presentation of Ruptured AAA Massively obese 55 yr. male presents to the ER with a protuberant abdomen, HR= 140 and BP=60/palp. Issues: a.) Initial therapy and diagnosis including how do you establish venous access in this patient. b.) Assuming he remains hypotensive after fluid administration and he is too large for traditional imaging modalities, what do you do? c.) Assuming a midline exploration shows a midline lower retroperitoneal hematoma, what is the most likely diagnosis and how do you manage it? d.) Diagnosis is a ruptured AAA, how do you operatively manage this problem immediately and definitively? ANSWER: ABCs A= Airway - if obtunded and unresponsive - intubate. However, muscle relaxants, narcotics or sedatives should not be used for induction until the patient is prepped and draped and incision is about to be made. B= 02 - 5-10L per face mask. C= Circulation. Infuse 2L LR through quickly placed Swan Ganz Introducer. Type and cross for 10U prbcs, 10U platelets and 10U FFP. Do not try to push fluids to achieve normotension before the aortic cross clamp as this may overcome the tamponade effect of the retroperitoneal blood. - place Foley catheter. - place art line in upper extremity if capabilities exist and this can be done quickly. - give prophylactic antibiotics.(Vanco 1 gr. IV).

TRIAD: ABDOMINAL/BACK PAIN, HYPOTENSION AND PULSATILE ABDOMINAL MASS - Ruptured AAA until proven otherwise. Physical Findings - pulsatile abdominal mass may or may not be present. Flank ecchymosis is a reliable finding of retroperitoneal bleeding but often a late finding. OR - the place to diagnose a ruptured AAA, especially with severe hypotension. CT SCAN - only if : hemodynamically stable and if doubt exists about the diagnosis. Obviously not indicated in this case. OR - Hold Induction of General Anesthesia until the patient is prepped and draped and incision is ready to be made. Have anesthesiologist place Swan Ganz catheter/IJ Swan Ganz Intro prior to induction of anesthesia. Prep - Neck to Knees - make certain that the left chest is adequately prepped in case a thoracotomy is necessary for cross clamping above the diaphragm because of hypotension in an infrarenal aorta or upon finding a juxtarenal or suprarenal aneurysm. Incision - large midline PROXIMAL CONTROL OF AORTA INTRAPERITONEAL RUPTURE OF AORTA - occlude with finger and place a Foley catheter with 30 cc balloon in hole and inflate balloon. This can temporize until proximal neck can be dissected out. REASONABLY STABLE - gain initial control at the diaphragm. Divide the left triangular ligament of the liver and rotate the left lateral segment medially. Palpate the NGT and sweep the esophagus to the right. Palpate the pulsatile aorta. Divide the peritoneal covering over the pulsatile aorta and identify the crural fibers. Dissect the crural fibers away from the aorta and place clamp tips against vertebral bodies. - deterioration to profound shock - alternatively can occlude the aorta here by compressing the aorta against the vertebral bodies using a fist or metal retractor. Problem is that this compression is not as secure or complete as the cross clamp. PROFOUND SHOCK - Left Thoracotomy - assists in quick cross clamp and control while dissecting out aorta and preventing injury to renal arteries, IVC, pancreas PREOPERATIVE CARDIAC ARREST - Left Thoracotomy - allows access for cardiac massage. PREVIOUS UPPER ABDOMINAL SURGERY - Left Thoracotomy - for obvious reasons. EXPOSE NECK AND PLACE INFRARENAL CROSS CLAMP - eviscerate the transverse colon and small bowel and pull the duodenum to the right. Divide the peritoneal covering along the Ligament of Treitz. The retroperitoneal hemorrhage will usually dissect the IVC away from the neck, but care must be taken to avoid injuring the IVC, lumbars or left renal vein when placing the cross clamp. - if the left renal vein injured during dissection or obscures the neck, it should be ligated.

- once the infrarenal clamp is placed, let of the suprarenal clamp slowly and expect a period of hypotension. Advise the anesthesiologist prior to removing clamp to allow time to prepare the patient with fluid bolus. DISTAL CONTROL - identify and clamp at the iliacs. If the area will have dissection obscured by hematoma, pinch each iliac and gain control from within the aorta using large Fogarty catheters or Foley catheters. OPEN ANEURYSM - open and remove mural thrombus. Gain control of iliacs from inside if this has not already been done. Oversew the IMA and any lumbars. GRAFT - TUBE GRAFT IF POSSIBLE - small iliac aneurysm are ignored. Place a bifurcated graft only for (1) large iliac aneurysm (2) severely calcified aortic bifurcation (3) severe iliac occlusive disease. BACKBLEED ILIACS - if backbleeding is poor, pass Fogarty catheters and flush with dilate heparin. CHECK DISTAL PULSES - check the femorals for palpable pulses and the pedal arteries for dopplerable signals. If pedal pulses are absent, consider a distal thromboembolectomy - base decision on the condition of the patient. - close aneurysm sac and check the sigmoid colon for viability. SPECIAL CONSIDERATIONS JUXTARENAL/SUPRARENAL AAA - most of these are juxtarenal rather than suprarenal. In this case, the clamp should be placed above the renals and the proximal anastomosis performed incorporating the lower margins of the renal artery ostia into the anastomosis if necessary. Once the anastomosis is completed, it is tested and the clamp is replaced on the graft below the anastomosis to allow perfusion of the renal arteries. - for a true suprarenal AAA, perform a medial rotation by mobilizing the spleen, pancreas, Left colon and left kidney toward the midline. The celiac, SMA, right renal artery are taken on a large patch and anatomosed to the graft. The left renal artery orifice is anastomosed to the graft separately. AORTOCAVAL FISTULA - rare as hell. Present with pain, mechanical abdominal bruit, hematuria (from venous bladder congestion), and CHF. Will not present with exanguinating hemorrhage. Once this is diagnosed, do not attempt to dissect out fistula. Occlude the vena cava with sponge sticks. Place proximal and distal clamps. Open the aneurym sac and close the defect in the cava from within the aortic sac using large bites.

AORTODUODENAL FISTULA - also rare as hell. Present with upper GI bleed and occasionally with pain. Unlike a secondary aortoduodenal fistula in which the previously placed graft is exposed to the duodenum, the risk of infection is low. - primarily close the duodenal defect and repair AAA with Dacron graft. COMPLICATIONS OF RUPTURED AAA REPAIR COMPLICATIONS COMMON WITH EMERGENCY OPERATION OR AAA REPAIR: - M.I. - Hemorrhage - Renal Failure - Respiratory Failure. COMPLICATIONS SPECIFICALLY OF RUPTURED AAA: - Ischemic Colitis - chances of this complication are lowered if the left colon is carefully examined after AAA repair. If the colon is left at the end of the operation must be vigilant and aggressive in the evaluation of DIARRHEA, SEPSIS, ACIDOSIS. -> FULL THICKNESS NECROSIS - resect and end colostomy. -> DUSKY MUCOSA - antibiotics, gut rest, optimize cardiac output and observe. - Spinal Cord Ischemia - paraplegia, fucked.

# 80 - Elective AAA with Hx of Colon CA. 65 y.o. male with history of transverse colectomy for colon cancer 5 yrs. ago. Ultrasound shows 5 cm AAA. Issues: a.) How would you evaluate this patient preoperatively? (Cardiac workup and colon workup). Radiographic evaluation? (make sure and have a CT scan to differentiate a suprarenal aneurysm or a mycotic aneurysm). b.) Anticipating the marginal artery has been disrupted with the previous colon resection, would you reimplant the IMA?

# 82 - AAA/Reimplantation of IMA/Ischemic colitis 79 y.o. man is found on routine exam to have a 5 cm AAA. HTN controlled with meds, very active, otherwise healthy. Issues: a.) How would you evaluate this AAA? At what size would you operate on his AAA? b.) how would you evaluate his cardiac status? How would you explain to this patient the risks of operative repair vs the risk of rupture? c.) What are the indications for reimplanting the IMA? If the IMA is not reimplanted, or is reimplanted and subsequently thromboses and the patient develops ischemic colitis - how would you workup this patients ischemic colitis? How would you treat this? If you explore the patient and find normal bowel (sigmoidoscopy shows edematous, boggy bowel with dusky gray mucosa) - what do you do? Disease that is not transmural will likely heal and should not be resected. ANSWER:

PREOPERATIVE EVALUATION: PERTINENT HISTORY - chest pain, shortness of breath, exercise tolerance, symptoms of claudication and ability to have erections. EXAM - palpable pulses in lower extremities, esp. palpable femoral pulses, rectal. LABS - cbc, lytes, BUN, Cr, PT/PTT, LFTs(if indicated) CXR EKG ABIs (if indicated) PFTs - if clinical evidence of COPD. Carotid Dopplers if evidence of cerebrovascular disease. Only symptomatic carotid disease should be operated on prior to AAA. CT Scan of Abdo/Pelvis - to identify an inflammatory aneurysm or the presence of a horseshoe kidney. Angiogram - if evidence of iliac disease/occlusion, suspicion of renovascular hypertension or history of visceral ischemia symptoms. Exercise Thallium - especially with history of chest pain/angina, previous MI, SOB, no exercise (because of disability, disabling claudication, severe SOB, etc) or evidence of previous MI on EKG. Those with reperfusion abnormality should have coronary angio. If coronary angio shows disease amenable to bypass, CABG should be performed prior to attempted AAA repair. AAA repair should follow CABG by 6-8 weeks. Severe Uncorrectable CAD - those with Ejection Fraction < 20%, and those with unreconstructible CAD should have AAA repair only for large AAA. RISK OF RUPTURE - in an asymptomatic aneurysm, main issue is whether the risk of rupture exceeds the risk of elective operation. Factors to be considered include (1) size, type and location of the aneurysm (2) comorbid diseases and risk factors. SIZE AND TYPE OF ANEURYSM - 5 year risk of rupture exceeds the risk of operation once the AAA reaches 6 cm. Because of the 20% risk rupture of small AAA, the recommendation is to repair AAA once it reaches 5 cm. - Aneurysms of 4 cm or less should be followed with CT scan every 6 months. When progressive enlargement occurs or there is a complication (such as embolization), operation is recommended. ARTERIAL ANATOMY - the blood supply to the left colon is mainly through branches of the IMA. However, the IMA is frequently occluded or thrombosed in patients with AAA. When IMA is occluded, collateral circulation is provided through collateral vessels from the SMA (via the meandering mesenteric artery and the marginal artery of Drummond, which has been interrupted in this case from the transverse colectomy) and from the hypogastric (internal iliac) arteries. COLLATERAL PATHWAY FROM SMA ->IMA -

1.) Marginal Artery of Drummond - LESS IMPORTANT- from SMA to the left branch of the middle colic artery via the marginal artery of Drummond at the splenic flexure of the colon to the left colic branch of the IMA.(peripherally located) 2.) Large Meandering Mesenteric Artery - MORE IMPORTANT - is located to the left of an infrarenal aorta and originates near the SMA at the middle colic origin and connecting the IMA circulation at the left colic artery origin. This anatomical variant is found in 2/3 or normal people and can dilate up to 3 mm in a patient who develops compromise of the IMA. Alternatively, if the SMA becomes stenotic, the IMA may feed much of the SMA circulation (entire midgut) through the meandering artery. (centrally located). It is unlikely that the Meandering Artery has been compromised during a transverse colectomy. HYPOGASTRIC ARTERIES - middle and inferior rectal branches of the hypogastric may communicate with the superior rectal branch of the IMA. Patients at Increased Risk for Postoperative Intestinal Ischemia - (1) History of Visceral Ischemia Symptoms. (2) Previous Colon Resection (3) Emergent Aneurysm Repair. Preoperative Arteriogram - if done, should focus on (1) patency of IMA (2) patency or stenosis in SMA (3) Presence of Meandering Mesenteric Artery as well the direction of flow. If there is flow from the SMA to the IMA, it is alright to ligate the IMA. If there is flow from the IMA to the SMA, ligation if the IMA will result in infarction of the small and large bowel. If there is flow through the MMA from the IMA to the SMA, will need to reimplant the IMA and revascularize the SMA. The mere presence of a meandering artery does not provide enough information to justify the ligation of the IMA (4) the presence of hypogastric flow. If both are occluded, the rectal blood flow is coming from the IMA or the SMA through the marginal or meandering artery. If this is the case, will want to revascularize one hypogastric artery. INTRAOPERATIVE EXAMINATION OF THE IMA - if there is no bleeding (occluded) or brisk pulsatile backbleeding (adequate descending colon collateral flow), the IMA can be ligated. - if there is sluggish, nonpulsatile flow, the IMA should be surrounded with a Pottsed vascular tie for temporary occlusion which will not damage the artery. Need to assess adequacy of collateral flow. INTRAOPERATIVE ASSESSMENT OF COLLATERAL FLOW - examination of colon serosa is not considered an adequate indicator of collateral blood flow. 2 METHODS: 1.) DOPPLER - doppler signal at the antimesenteric border during temporary IMA occlusion indicates inadequate collateral flow. 2.) IMA stump Pressure - > 40 mmHg or ratio >0.4 can be used as indicators or collateral flow and allow safe ligation of IMA. ISCHEMIC COLITIS COMPLICATING AAA REPAIR PATHOPHYSIOLOGY - colonic hypoperfusion is usually the result of ligation of a hemodynamically important IMA. May also be due to marginal mesenteric artery with an interruption of hypogastric collateral flow during the aortic cross clamp. Colonic ischemia is

the most common ischemic complication though the small bowel may be compromised if the IMA is ligated and there is a stenosis in the SMA (this is rare). 3 CLASSES OF ISCHEMIC COLITIS BASED ON DEPTH OF PROCESS: 1.) MUCOSAL - 20% of time. Mild and resolves. 2.) MUSCULARIS - 20% of time. Does not cause perforation but causes fibrosis and eventual stricture. 3.) TRANSMURAL - 60% of time. Gangrene and perforation result. PROBLEM WITH ISCHEMIC COLITIS - difficult to diagnose postoperatively, so evaluation should be aggressive, including mandatory colonoscopy in high risk patients. SYMPTOMS - bloody or watery diarrhea, unexplained fever, leukocytosis, abdominal pain, abdominal tenderness, increased fluid requirements, occult sepsis and metabolic acidosis. Presence of any of these mandates evaluation. Findings - mucosal ulceration should be treated with antibiotics, bowel rest, hyperalimentation, optimization of cardiac output (if this is a contributing factor) and serial colonoscopy. Exam - abdominal findings on physical exam imply a transmural process, making exploration and treatment mandatory SURGICAL TREATMENT OF ISCHEMIC COLITIS - immediate measures to minimize contamination and possible infection of graft. Evaluate the colon visually and using Doppler exam. COLON ISCHEMIA - resect compromised colon, end colostomy, Hartmanns or mucus fistula. RECTAL ISCEMIA - debride involved rectum and irrigate. Provide drainage. SMALL BOWEL ISCHEMIA - debride frankly necrotic or perforated small bowel. Revascularize the SMA, with resection of necrotic bowel. Plan second look operation.

# 81 - Elective Large AAA with Colon CA 70 y.o. male with constricting (not obstructing) sigmoid cancer and asymptomatic 8 cm AAA. Issues: a.) Which one do you repair first? b.) Describe sigmoidectomy. c.) Describe AAA repair. Pitfall - if choosing a retroperitoneal repair of AAA, be prepared to describe everything and defend control of the right iliac as well as treatment of iliac aneurysm. d.) If forced to operate on the AAA because of leakage, how will you deal with the colon CA? Pitfall - if dealing with both at once, be prepared to deal with a graft infection. Alternatively - Pt. with an 8 cm AAA and splenic flexure tumor with heme positive stools.

Issues: a.) Which do you operate first? ANSWER: BOWEL TUMORS AND ANEURYMS - performing both procedures concomitantly is much more likely to be accompanied with life threatening complications and should not be performed. TUMORS DIAGNOSED PREOP - THE SIZE OF THE ANEURYSM AND THE DEGREE OF OBSTRUCTION SHOULD BE CONSIDERED WHEN DECIDING ON WHICH SHOULD BE MANAGED FIRST. RESECT BOWEL FIRST - AAA is less than 5 cm diameter. REPAIR AAA FIRST - if AAA is > 5 cm diameter or symptomatic. TUMORS DIAGNOSED INTRAOP - tumors should be left alone unless they are obstructing or pre-obstructing. Obstructing or Preobstructing - if bowel prep is done, conventional colon resection is performed. If the bowel has only been prepped with enemas and there is there is an obstructing tumor of the jejunum or right colon, can resect. If the lesion is anywhere else and a complete bowel prep has not been done, close and prep the bowel. Nonobstructing Tumor - leave alone, perform AAA and return for colon resection in 3weeks or later if postop course is slow. Obstucting lesion and symptomatic AAA - repair AAA, close retroperitoneum, perform proximal diverting colostomy to be opened later or exteriorize tumor and resect later with double barrel colostomy. # 83 - Chronic Mesenteric Ischemia 60 y.o. female w/o sig. PMHx presents with post-prandial pain, diarrhea, and 10 lb. weight loss. Issues: a.) Differential diagnosis. b.) How would you work this up? c.) What would you expect to see on the visceral arteriogram? d.) How would you approach this? What kind of conduit would you use? Which vessel (s) would you revascularize and how would you place the graft? ANSWER: - Chronic Mesenteric Ischemia results from inadequate perfusion of the midgut during periods of increased oxygen demand. 02 demands of bowel increase significantly in the postprandial period because of rises in motility, secretion and absorption. - Chronic Mesenteric Ischemia is differentiated from Acute non occlusive mesenteric Ischemia in which the mesenteric arteries undergo severe vasoconstriction from vasoactive medications (norepi) or by decreased cardiac output from MI, hypovolemia or arrhythmia.

- Hypoxic injury is manifested by ischemic visceral pain and abnormlities in absorption and motility. Pain is similar to myocardial ischemia or angina. - Atherosclerotic involvement of large arteries is almost always the cause, but it can be related to vasculitis such as Buergers or polyarteritis. Differential Diagnosis 1.) Chronic cholecystitis 2.) Pancreatic CA 3.) Gatric CA 4.) Colon CA. 5.) PUD WORK - UP: DIAGNOSIS - No specific reliable test exists, diagnosis must be based on clinical symptoms (must have weight loss, should have pain) and angiographic evidence showing occlusion of the splanchnic arteries and to a greater degree the exclusion of other gastrointestinal causes. ANGIO - stenosis or occlusion of one or more of the major visceral vessels on angio does not itself establish the diagnosis of arterial insufficiency. Likely have stenosis/occlusion of at least two of the three arteries. The presence of prominent collaterals not only indicates major artery stenosis but also indicates a chronic process. Likely show a large meandering mesenteric artery. INDICATIONS FOR SURGERY - (+) diagnosis = pain with (+) angio findings. Fear of impending thrombosis is not an indication. The only indication for repair in the absence of pain is patient undergoing aortic reconstruction (AAA or occlusive disease) in which a preop angio shows a stenosis/occlusion of the SMA or celiac with large meandering artery and patent IMA. TREATMENT - limited success with transluminal angioplasty, but not much experience or data - may be used in the case of sick patient who cannot tolerate an operation. Preop Preparation - usually require a period of TPN because of the long period of malnutrition. Albumin, PT/PTT should be checked preop. SURGICAL TREATMENT OPTIONS: REIMPLANTATION - transect the artery distal to the occlusion and reimplant it in the aorta below the site of the artery takeoff. Technically difficult because of the severe disease of the aorta and the long segment of an SMA/celiac stenosis. Should only be done when aorta is being reconstructed and revascularization is being done preventitively (see above).

ENDARTERECTOMY - can be done transarterially or transaortically. Problem with transarterial approach is getting the portion of the disease at the most proximal aspect. Problem with the transaortic approach is need to cross clamp in the supra celiac position and the inherent risk of renal ischemia with cross clamp. BYPASS - from the aorta or preferably iliac to the SMA distal to the site of occlusion is the procedure of choice. Conduit of choice - reversed autologous saphenous vein. PTFE or knitted Dacron can also be used if the saphenous vein is unavailable or is less than 5 mm diameter. There is controversy over the best site of origin of the graft - debate is over the chance of kinking or twisting from the small bowel mesentery if the suprarenal position is used vs. the enhanced rate of atherosclerosis in the infrarenal aorta if this is used as the site of anastomosis. TECHNIQUE - CELIAC - retract the stomach inferiorly and the liver superiorly to place the lesser omentum on stretch. The gastrohepatic peritoneum is incised (superiorly/cephalad) and the gastrocolic ligament is incised (inferiorly/caudad) opening the lesser omentum. The celiac axis is exposed by sharply dissecting the sympathetic fibers. Celiac axis is the confluence of the hepatic, splenic and left gastic - these branches must be dissected out to select the most appropriate site for anastomosis. Tunnel - lift the pancreas forward and pass a finger through the retropancreatic fascia anterior to the left renal vein into the lesser sac. Once the tunnel is created, a clamp is passed through the tunnel and graft is passed. - SMA - expose freeing the vessel from the root of the mesentery - primary branches to the jejunum are isolated. Area of the SMA proximal to the major branching points are selected as the site for the proximal anastomosis. Area proximal to this is likely occluded.

# 84 - Carotid Disease 60 y.o. hypertensive, moderately obese attorney who smokes 1.5 packs of cigarettes per day is referred with an asymptomatic right carotid bruit. Issues: a.) What do you do if the duplex shows a 40-50% smooth stenosis when the patient is entirely asymptomatic? How often would you follow the patient? b.) Assuming that a repeat duplex shows an 80% smooth stenosis still without symptoms, what do you do? The patient refuses surgery at this point. What if the patient had 40% ulcerated plaque without symptoms? What about a 40% stenosis with ulceration and symptoms? c.) The patient returns in 1 month having had a bland stroke with complete resolution of symptoms within one week. What would you advise him? d.) On cardiac workup the patient is found to have a reversible defect on stress thallium. What would you advise re: cardiac vs. carotid surgery? During wait for cardiac surgery the patient has RIND. What would you recommend? e.) How precisely would you perform a carotid endarterectomy? Type of anesthesia, EEG monitoring, use of shunt, use of patch? f.) Patient becomes hypotensive during procedure, what is the most likely cause and what is your treatment?

g.) Postoperatively, the patient develops extreme hypertension in PACU. What is your differential diagnosis. How would you rule new stroke? How would you treat this patient? What doses would you use? ANSWER: Indications for Carotid Endarterectomy (CEA) in the assymptomatic patient: - Stenosis of ICA of 75-80%. - Asymptomatic Contralateral Carotid Stenosis Opposite Symptomatic Carotid - contralateral lesions of less than 75% can be followed clinically. Lesions greater than 75% should be considered for prophylactic endarterectomy. - Asymptomatic Carotid Stenosis is Patient Undergoing Surgical Procedure - risk of stroke is 0.3% in patients about to undergo general surgical procedure, 1% in patients about to undergo vascular procedures and 2-4% in patients about to undergo aortic reconstruction or CABG/valve. In cases of peripheral or visceral vascular procedures accompanied by significant carotid stenosis, the symptomatic lesion should be repaired first. If both are asymptomatic, the carotid is done first (but it is unusual for either a peripheral or visceral vascular procedure to considered if it is asymptomatic) Considerations for Carotid Stenosis and AAA Repair - if both lesions meet indications for repair (AAA=5-6 cm and ICA=75% or greater), treat the symptomatic lesion first. If the patient has bilateral asymptomatic critical lesions and an asymptomatic AAA, fix the carotid first. If asymptomatic unilateral carotid stenosis and asymptomatic AAA (both meeting criteria for repair), can treat either first. Only if the AAA is symptomatic and carotid has crescendo TIAs should simultaneous procedures be considered. Consideration for Carotid Stenosis and Coronary Revascularization Risk of Stroke during CABG = 2-4%. Risk of MI during CEA = 0.5- 2.5%, though it is acknowledged that MI rate is higher among those with known cardiac history or symptomatic coronary disease. It is unclear whether the risk of MI, stroke and death is greater for a staged procedure or a combined procedure. Probably makes more sense to protect a patient with an asymptomatic carotid stenosis from having a heart attack than a patient with heart disease from having a stroke. - several risk factors place a patient at risk for MI during or after a CABG - Q-waves, exertional angina, recent MI, CHF, diabetes, age > 70. These patients should be evaluated for cardiac disease the same way as the patient undergoing a AAA repair. EVALUATION : HISTORY AND PHYSICAL EXAM - for NYHA Classification EKG Exercise/ Dipyrdamole Thallium - if any of the above risk factors present. (+) THALLIUM ---> CORONARY ANGIO.

(+) CORONARY ANGIOGRAM (1) Lesion Amenable to PTA - should be performed and followed CEA with same or later hospital admission. (2) Lesion Not Amenable to PTA, Stable Angina + Bilateral Asymptomatic/Unilateral Symptomatic Carotid Stenosis = perform CEA under local or under general with intensive invasive intraop cardiac monitoring. Follow with staged CABG in 4-6 weeks. Only situation in which carotid done first. (3) Lesion Not Amenable to PTA, Stable Angina + Unilateral Asymptomatic Carotid Stenosis = could perform CEA under local but makes more sense to perform CABG first, followed by staged CEA. (4) Lesion Not Amenable to PTA, Unstable Angina + Unilateral Asymptomatic - perform CABG first followed by staged CEA in 4-6 weeks. (5) Lesion Not Amenable to PTA, Unstable Angina + Unilateral Symptomatic or Bilateral Asymptomatic = combined procedure. (6) Lesion Not Amenable to PTA, Presence of Left Main Disease or Poor LV function combined procedure. (7) Severe coronary disease with poor EF not amenable to CABG/PTA + symptomatic carotid stenosis = CEA under local. (8) Severe coronary disease with poor EF not amenable to CABG/PTA + asymptomatic carotid stenosis = medical therapy. TECHNIQUE = can perform median sternotomy at the same time as the neck dissection or perform the CEA while harvesting the saphenous vein for the CABG. the cardiac surgical team is ready, though this is never necessary. INDICATIONS FOR CEA IN PATIENT WITH ULCERATED PLAQUE Classification of Ulcerated Carotid Plaque A = small ulcer B = large obvious excavation C = multiple cavities or cavernous appearance on angio. Ulcer - most common cause of symptoms is embolization from the carotid bifurcation, which can occur from ulcers. SYMPTOMATIC CAROTID PLAQUE --> CEA ASYMPTOMATIC CAROTID PLAQUE - patient with asymptomatic carotid plaque is initially evaluated by noninvasive Doppler. The Doppler will provide 2 pieces of information - the degree of stenosis and the size of the ulcerated plaque. The degree of stenosis should be dealt with the same way as a smooth carotid lesion. (80% or greater stenosis --> CEA, less than 80% ---> medical therapy and serial duplex)

A Ulcer - small ulcer - treat the same way as a smooth stenosis. Treat with medical (antiplatelet therapy). Follow with duplex Doppler every 6-12 mos. B Ulcer or C Ulcer - the presence of a large ulcer should be confirmed and further evaluated with angiography. This angio will give definitive definition of the class of the ulcer and also a definitive degree of stenosis. >80% stenosis ----> CEA C Ulcer ---> CEA B Ulcer - unclear, can be treated with CEA or antiplatelet therapy and serial Dopplers every 612 months. MANAGEMENT OF EVOLVING STROKE WITH CAROTID STENOSIS: Evolving Stroke - temporal progression of neurological signs which may wax and wane but do not return to normal. Crescendo TIAs return to normal in between. Initial Evaluation of Patient with Evolving Stroke - BP control, 02, CT Scan - to rule out hemorrhage (infarct may or may not show early), ?heparin, noninvasive studies (if not already done). - hemorrhage, hemorrhagic infarct or AVM rule out surgical candidate. - those showing no infarct on CT scan are the best candidates. Neuro Deficit - those who are surgical candidates should have mild or moderate neuro deficit (less than complete hemiparesis). Complete hemiparesis should not have surgery. Anatomic Evaluation - those who do not have infarction CT and those with mild neuro deficit and noninvasive studies showing critical stenosis should have angio (MRA would be preferred as the risk of stroke related to procedure is lower.) - preop evaluation = cardiac evaluation is undertaken to dictate the type of anesthetic management needed. There is no time for intensive investigation because PTA/coronary revasc. is not an option. Operation - select group of patients who would have a critical stenosis with an acute thrombosis or a patient with an ulcerated plaque and an acute thrombosis. The aim of the operation is to remove the site of thrombosis/embolization while improving blood supply. If angio shows good flow round lesion or complete occlusion, do not operate. MANAGEMENT OF FIXED STROKE RELATED TO CAROTID STENOSIS Stroke - fixed neuro deficit that has not cleared in 30 days. - 40% of those having stroke will have new stroke in next 3-5 years when given medical therapy. The risk of stroke if treated with surgical therapy is about 0-3% per year (9-15% at 3-5 yrs.) PATIENT SELECTION - should be a patient who has minimal neuro deficit or has recovered completely from stroke. Those with dense hemiplegia and those with complete recovery

from stroke are the best candidates for surgery. Those with dense hemiplegia and decreased mentation have little to prevent by surgery. EVALUATION - should have full cardiac evaluation (echocardiogram, Holter, EKG) to rule out a stroke from a cardiac embolic source. Patient should also have noninvasive studies (duplex Doppler) and CT Scan to document the extensiveness of intracranial disease. Usual cardiac workup. Angio or MRA. Minimal Plaque - if no other source can be found, CEA is justified only if the patient develops new symptoms. Moderate to Severe Stenosis - (>75-80%) - should have CEA after 4-6 week delay. Opt for slightly earlier repair if the lesion is preocclusive. Total occlusion - CEA if contralateral stenosis is greater than 50%. RECOGNITION OF STROKE FOLLOWING CEA Mechanisms of Stroke Following Carotid Endarterectomy 1.) Ischemia during clamping - can usually be circumvented by the use of shunting. Those at highest risk for ischemic stroke related to clamping are those with contralateral stenosis. 2.) Carotid Thrombosis at Operative Site - symptoms can vary from focal findings to generalized global ischemia. Complete occlusion of the ICA postop is usually due to thrombosis at the operative site and usually die to easily correctable causes (intimal flaps, kinking of artery, buckling of posterior wall after too small a patch for angioplasty.) 3.) Embolization - can occur either at the time of cross clamp, after clamp is released or hours to days postop. Deficits are usually transient, and reexploration of the neck is not indicated. 4.) Intracerebral hemorrhage - usually occurs in the first and tenth day. Course is progressive and usually fatal. Craniotomy if indicated. MANAGEMENT Neuro Deficit - thrombosis must be suspected. Focal Defect, resolving - usually from emboli that occurred intraop. Recurring Focal Deficit - usually from recurrent emboli, should be reexplored. INDICATIONS FOR EMERGENT REEXPLORATION: 1.) Stroke or TIA and Duplex is positive. 2.) Severe stroke on awakening from anesthesia whether duplex is positive or negative. - if no duplex Doppler is negative, return to OR. STROKE AND DUPLEX DOPPLER NEGATIVE ---> ANGIO STROKE AND DUPLEX DOPPLER, ANGIO OR NECK EXPLORATION NEGATIVE ------> CT Scan to rule out hemorrhage. INTRAOP REEXPLORATION - expose operative site and examine artery and closure. If the artery is open and has normal pulsatile flow, perform and intraop angio with 20 gauge needle. If the angio is positive or the ICA/CCA is nonpulsatile, the patient heparinized and vessels clamped and shunt placed. Arteriotomy opened and patient reexplored. Close with patch angioplasty. Perform completion arteriogram. BLOOD PRESSURE CONTROL AFTER CEA;

Cause - Unknown. Risk Factors - preop hypertension, ICA stenosis over 75%. Neuro Deficit - since it is impossible to differentiate whether hypertension causes a neuro deficit or the neuro deficit causes the hypertension, one must assess the latter. Subgroup whose first evidence of neuro deficit was the development of postop hypertension. Patients who develop postop hypertension should be evaluated for thrombosis/emboli related to technical factors as above. Medical Treatment of Postop Hypertension - treat systolic greater than 200 mmHg with Nitroprusside. Keep SBP between 160-200 mmHg. CAROTID ENDARTERECTOMY TECHNIQUE EEG - leads placed before surgery, intraop EEG monitoring. POSITION - shoulder roll, head on cushioned donut, neck not hyperextended. INCISION/DISSECTION - along anterior border of SCM. Divide the platysma down to the SCM, divide the cervical fascia at the anterior border of the SCM and retract laterally. Divide the tissue down to the IJ. Ligate the facial vein. Dissect along the medial surface of the IJ. Retract the IJ laterally. **vagus nerve lies beneath the CCA but then runs lateral to the ICA. ** hypoglossal nerve runs across the field, usually across the external carotid artery. CAROTID DISSECTION - the carotid bulb is injected with 3-5 cc 1% lido without epi. The common carotid is dissected circumferentially and encircled with an umbilical tape. Likewise the ICA and ECA are dissected circumferentially and encircled with vessel loops. Palpate the artery, feel for the plaque. Free the ICA up at least 1 cm beyond the distal aspect of the plaque. HEPARINIZE - 3500-5000 Units of heparin. After 2-3 minutes, the ECA is clamped with a soft bulldog. The CCA is then clamped with an angled vascular clamp. The ICA is clamped with a soft bulldog clamp and arteriotomy is made from the CCA to the ICA above the plaque. (+)EEG - loss of beta activity, loss of amplitude, emergence of slow wave activity. (+) STUMP PRESSURE - less than 40-50 mmHg. SHUNTING - if indicated - shunt is placed in the distal ICA above plaque, and allowed to backflow from the ICA to fill the shunt. The shunt is then placed in the CCA and the angled vascular clamp is removed. The shunt is put in place and umbilical tapes snugged down. Use 10 Fr. ENDARTERECTOMY - plane is between the diseased intima and media through internal elastic lamina. Get nice tapered, feathered end. Tack down if necessary. Irrigate CLOSURE - close primarily unless the primary closure will narrow luminal diameter. Close with 6.0 prolene.

IMAGING - should use intraop duplex or angio.

# 85 - Cold/Ischemic Lower Extremity S/P MI 70 y.o. female in the E.R. discharged 3 weeks after an uncomplicated hospitalization for an acute MI presents with 6 hr. history of cold, numb left leg. Issues: a.) Initial evaluation, history and physical examination b.) How do you decide if there is neurological compromise? c.) How would you evaluate this patient prior to going to OR? d.) Anesthetic ? Procedure? Fasciotomy? What size fogarty catheter would you use? ANSWER: - in 85% of cases, the clinical presentation and associated disease allow differentiation between acute emboli (which do not require extensive arterial reconstruction) and thrombosis of existing disease (which require extensive reconstruction). HISTORY - key to differentiating embolus from thrombosis (and planning operative intervention) is the presence of a history of symptoms such as claudication. - the sudden onset and the history of recent MI, recent arrhythmia, valve dysfunction or replacement are the keys to history of embolus. EXAMINATION - pulselessness (absent Doppler signals), pallor (lack of capillary refill), paresthesias and paralysis. Paresthesia - need to evaluate sensation and proprioception. Motor - note and record the level of motor impairment. Embolus - almost always found at the arterial bifurcation just above the site where the Doppler signal disappears. ANGIOGRAPHY - commonly done and can provide definitive diagnosis prior to surgery. However, in deciding whether to proceed with angio, one must weigh the certainly of the diagnosis vs. the potential for further neurologic compromise/negative outcome if angio is done. PREOPERATIVE MANAGEMENT - HEPARIN - done to prevent both the propagation of the clot(treatment) and further embolization (prophylaxis). Start with bolus of 5000 units BOLUS and start drip at 500 U per hour (aim for PTT = 2.5 times normal). ISCHEMIA TIME - no specific time interval is predictive of a functional outcome or serves as a indication or contraindication for surgical therapy. More important is the neuro status of the limb at the time of intervention and the ability to perform a successful embolectomy. PREOP NEURO STATUS - Motor - loss of dorsiflexion is the first sign. Extreme dorsiflexion weakness and absence of dorsiflexion is a sign that ischemia is advanced. Sensory - numbness is usually present to some degree in all patients and is not a particularly ominous sign. loss of motor and sensory function is a contraindication to surgery in a patient with recent MI. May be a better candidate for thrombolytics. even in presence of neuro dysfunction in an otherwise healthy patient, embolectomy in presence of severe neuro compromise may alter the level of amp.

ANESTHETIC - local anesthesia with anesthesiologist present PREP - nipples to toes, so that limb or abdomen may be explored if necessary. EXPOSURE - the involved vessels are dissect out and controlled. For embolus at the distal common femoral, the SFA and profunda are dissected out along with the common femoral. The area of the bifurcation is palpated to find the proximal portion of the artery that has a pulse. Assuming the patient is already heparinized, the vessels are clamped. if the diagnosis is clear, a transverse arteriotomy is made. If there is some question, a longitudinal arteriotomy is made (and can be used as such if a bypass graft is necessary). EMBOLECTOMY Fogarty Catheters: AortoIliac = size 5-6 Femoral/Popliteal = size 3-4 Tibial = 2 Technique - even is aortiliac thrombus is suspected, the distal femoral vessels are done first. The incision is made and femoral vessels are exposed and controlled. The vessels are clamped. No3-4 catheter is passed distally to the point of mild resistance, balloon inflated and catheter pulled back. The deep femoral system is examined with a #3 catheter to 25 cm. Check for backbleeding (though this is no guarantee all clot is removed). Flush with 25-50 cc of dilute hep. flush. - if the iliacs and aorta are to be examined, a #6 catheter is passed proximally into the aorta from the CFA. The catheter is withdrawn and repeated if there is not return of forceful, pulsatile blood from the CFA. If there is high suspicion of aortic clot, this procedure must be repeated on the opposite side. - if there is problem extracting distal tibial emboli, should cut down over the tibial vessels and directly thread the catheter into the tibial vessels. - close the arteriotomy with simple interrupted 6.0 prolene sutures. Get a completion angiogram. FASCIOTOMY - should be considered in the patient with a severely ischemic extremity (consider preop neuro exam). Can be done at the time of embolectomy or 6-12 hrs later if the diagnosis is in question. Compartment pressures above 30 mmHg is an indication for fasciotomy. LONG TERM ANTICOAGULATION - continue heparin until therapeutic c on coumadin. Attempt is made to delineate the underlying cause (hypercoaguable state, arrhythmia, etc.) In this case, with a self limiting cause (MI), continue for 6-12 months.

# 86 - Cold/Ischemic Lower Extremity S/P AortoBifem. 70 y.o. male S/P aortobifem for occlusive disease and are called to see because of cool, pale R leg. a.) What do you do? Groin embolectomy performed and angio shows good runoff? Fasciotomy? b.) Next day patient develops cold right leg again. (Angio showed occluded SFA) What do you do? c.) 3 weeks postop the patient develops a wound infection. What do you do? ANSWER: DIFFERENTIAL DIAGNOSIS: 1.) OUTFLOW PROBLEM - GRAFT LIMB CLOTTED I.) Kinking of graft as it passes through tunnel. II.) Narrowing of the anastomosis. III.) Poor outflow (extensive femoral popliteal disease) iv.) Emboli at anastomosis or in immediate outflow. 2.) OUTFLOW ALRIGHT - GRAFT LIMB PATENT I.) Shock II.) Spasm - unlikely III.) Distal Emboli - to tibial vessels IV.) Compartment Syndrome. PHYSICAL EXAMINATION - should focus on the level of occlusion and whether the graft is open. Should also try to determine the neuro status of the extremity - the sensory and motor exam. Focus on 4 Ps for exam (pallor, parethesia, paralysis and pulselessness). If compartments are not soft, perform compartment pressure measurements. PLAN - OR ANESTHESIA - general, in case it turns out there is extensive fem/pop disease and the patient needs a fem - pop. PROCEDURE - Embolectomy and Intraop Angiogram - see above for technique GRAFT OPEN, EMBOLECTOMY, CLEAN ANGIOGRAM ---> likely from distal embolus (macroemboli or microemboli), spasm, or compartment syndrome (which will already be ruled out by measuring pressures). Macroemboli - perform embolectomy, get intraop angiogram. Microemboli - treat with anticoagulation Compartment Syndrome - Fasciotomy Shock - resuscitate. GRAFT NOT OPEN, EMBOLECTOMY, CLEAN ANGIOGRAM --> suspect either the anastomosis or kinking of graft in tunnel. Revise anastomosis. If revision of the anastomosis does not work, revise that limb of the graft from the aorta to the groin.

GRAFT NOT OPEN, EMBOLECTOMY, CLEAN ANGIOGRAM -->consider problem with inflow, problem at aortic anastomosis. GRAFT NOT OPEN, EMBOLECTOMY, ANGIOGRAM SHOWS FEMORAL OCCLUSION --> fem - pop bypass.

Other
# 87 - Lung Lesion 64 y.o. woman with new 2 cm lesion in RUL on CXR. Issues: a.) How would you workup this patient? How would you attempt to obtain a tissue diagnosis? How would you evaluate for resectability? b.) What would you do with mediastinal adenopathy? Describe mediastinoscopy. c.) What would you suspect with development of a pleural effusion with pneumothorax on POD #10 (CT out on POD#4). ANSWER: TNM DEFINITIONS AND STAGING Tis - Carcinoma in situ T1 = tumor 3 cm or less and surrounded by lung not invading main bronchus or pleura. T2 = tumor 3 cm or more or any tumor that invades pleura or has associated post obstructive pneumonitis, must be 2 cm from carina. T3= tumor of any size invading the chest wall, diaphragm, mediastinum (but not contents) or pericardium or any tumor involving the main bronchus or within 2 cm of the carina. T4= tumor of any size invading the contents of the mediastinum, trachea, esophagus, vertebral bodies, carina or presence of malignant pleural effusion N0 = no nodes N1= mets to peribronchial nodes or ipsilateral hilar nodes. N2= mets to ipsilateral mediastinal or subcarinal nodes N3= ipsilateral supraclavicular nodes or any contralateral nodes. STAGE 0 = Tis STAGE I = tumor of any size without invasion of chest wall, mediastinum (or contents), or pericardium. May invade pleura or be associated with post obstructive pneumonitis. No nodes or mets = T1 or T2 N0 M0

STAGE II = tumor of any size without invasion of chest wall, mediastinum (or contents), or pericardium. May invade pleura or be associated with post obstructive pneumonitis. (+) nodes in ipsilateral hilar or peribronchial region. No mets. = T1 or T2 N1 M0 STAGE III = tumor of any size invading the chest wall, diaphragm, mediastinum (but not contents) or pericardium or any tumor involving the main bronchus or within 2 cm or the carina. Mets to either ipsilateral peribronchial, hilar, subcarinal, mediastinal nodes. No mets. = T3 N1 or N2 M0 STAGE IV = any tumor with mets, regardless of nodes or level of local invasion. = any T any N M1 SELECTION OF PATIENT FOR SURGERY STAGE IV = CHEMOTHERAPY STAGE III = CHEMOTHERAPY + RADIATION. Surgery may used in some cases. STAGE I AND II = SURGERY. COIN LESION - solitary pulmonary nodule - malignant less than 1% of time in patient under 30 but malignant in 50% over age 50. WORKUP = need to determine whether or not the mass is malignant and if it is the location of the tumor, presence of invasion into other structures, presence and location of nodal mets, presence and location of mets. ASYMPTOMATIC NODULE: STEP #1 - Obtain old X-rays - those that have not changed in 2 years are benign. STEP#2 - Examine X-rays with radiologist - if appearance of benign granuloma (calcified), it is likely benign and can be followed. STEP #3 - CT SCAN OF CHEST (through upper abdomen to examine adrenals) - should be done if the old x-rays are not available or if the new X-rays show a change. STEP #4 - ROUTINE LABS (CBC, chem-7, Ca++, alk phos, LDH). STEP #5 - SPUTUM CYTOLOGY STEP #6 - BRONCHOSCOPY -with brushings and biopsy if possible. STEP # 7 - BIOPSY - biopsy can be obtained via transbronchial biopsy for central lesions. If the diagnostic workup is still negative and the lesion is peripheral, may opt for percutaneous biopsy. However, it is recognized that a negative result will not rule out cancer. If the patient is a good surgical candidate, should proceed to thoracotomy (or thoracoscopy) for diagnosis and definitive treatment if the lesion is malignant. - once this workup has been completed, it is necessary to determine if there is presence of N2 (ipsilateral mediastinal or subcarinal nodes) or N3 (contralateral nodes or ipsilateral supraclavicular nodes) disease. Presence of N2 or N3 disease will make this IIIa or IIIb disease and will require that chemotherapy and radiation as the primary modes of therapy rather than surgery. MEDIASTINAL NODES - those nodes less than 1 cm are considered benign, nodes greater than 1.5 cm are often malignant. However, mediastinal nodes can be enlarged from infection related to postobstructive pneumonitis or chronic lung disease. MEDIASTINOSCOPY - allows the ability to sample central tumors, left and right paratracheal nodes, right tracheobraonchial nodes and subcarinal nodes.

TECHNIQUE - 2cm transverse incision in the base of the neck. The strap muscles are separated to expose the trachea. Peritreacheal fascia is separated and elevated. Blunt digital dissection allows separation of the mediastinum from the trachea. Allows visualization of the paratracheal and subcarinal nodes as well as the azygous vein and the pulmonary artery. Dissect the node using blunt dissection to make sure that a vascular structure is not biopsied. Aspirate before biopsy. Main complication is bleeding from major vascular structures. DETERMINATION OF ABILITY TO RESECT: PFTS Initial FEV1> 2L - allows any resection including pneumonectomy Postresection FEV1>800CC - single most useful parameter because of presence of chonic lung disease. - if initial FEV1 < 2L, get ventilation - perfusion scan to determine how much the area to be resected contributes to the overall lung function ABGS pC02 - serve mainly to identify the patient who is at extremely high risk for any form of pulmonary resection because of C02 resection. pC02>50mmHg - not a candidate for resection. p02 - may simply reflect the presence of a shunt or ventilation/perfusion mismatch caused by tumor and is not helpful in selecting patient for resection. However, p02< 50 mm Hg is generally indicative of severe restrictive disease and should be considered a contraindication to resection. FEV1/FVC< 50 - connotes high surgical risk MMV - minute ventilation volume > 50% predicted value should tolerate pneumonectomy. - if all other factors speak against resection, can check functional status by the ability to tolerate walking up a flight of stairs. # 88 - Testicular Torsion 10 y.o. boy with very tender and painful testicle. issues: a.) Differential diagnosis. Initial test (UA) b.) UA is normal. c.) What, if any, preop workup is necessary? d.) Describe the operative approach.

ANSWER: DIFFERENTIAL DIAGNOSIS: 1.) Testicular Torsion 2.) Epididymitis 3.) Acute Orchitis 4.) Strangulated Hernia 5.) Traumatic Hematocoele or Hydrocoele 6.) Torsion of testicular appendix - Torsion occurs much more likely than epididymitis in this age group, since children of this age are rarely sexually active. The exception is the child that for some reason requires catheterization or has anomalous linkage between the vas deferens and urinary tract. - torsion is predisposed by abnormal suspension of the testicle (of which there are many configurations). 50-60% of patients will have the same abnormality of testis suspension on the contralateral side. It is important when taking history in this patient to look for history of intermittent torsion which resolved spontaneously on the contralateral side. If such a history is (+), a contralateral orchiopexy is indicated. UA- can be sent to rule out epididymitis. However, the presence of a positive UA and epididymitis does not rule out testicular torsion. The downside of misdiagnosing a testicular torsion is that the testicle will necrose. Duration of Torsion - presence of torsion for longer than 6-12 hrs. minimizes the chance and salvage. Ischemia time of less than 12 hrs. is associated with 90% salvage rate. Further Testing - not indicated. Both ultrasound and nuclear medicine scan can be used to determine if there is blood flow to the testicle but may not be available or timely in this situation where only prompt surgical correction can prevent loss of the testicle. - manual detorsion of the testicle in the ER can temporize the situation and change it from an emergent to an elective procedure. TECHNIQUE - if diagnosis is certain, use a scrotal incision. If diagnosis is in doubt, or if there is concern about the presence of a concomitant hernia, use a groin incision. - frankly necrotic testicle is removed. Ischemic testicle is detorted and observed for viability. If the testis is viable, perform an orchiopexy by securing the testicle to the dartos muscle with a minimum of 2 opposing nonabsorable sutures.

# 89 - Necrotizing Fasciitis 24 hrs. S/P APR pt. with fever and obtundation with abdominal wall bulla and grayish wound drainage. Issues: a.) What is your initial evaluation? b.) Gr. stain shows gram pos. rods. What antibiotics would you start? What would be your treatment plan? c.) How much tissue would you remove?

d.) What would you do if you had to remove so much muscle that there was a defect you could not reapproximate? e.) What is the pathophysiologic mechanism of C. perfringens? NECROTIZING FASCIITIS - PREDISPOSING FACTORS: 1.) Impairment of Immune system (steroids, diabetes, alcoholism, malignancy). 2.) Compromised fascial blood supply. 3.) Microorganisms which can proliferate within area. - those who are immunocompromised, have decubitus or stasis ulcers, closure of heavily contaminated wounds or closure of traumatic wounds. Clostridial Perfringens - spectrum of infection ranges from simple contamination without toxicity to cellulitis to clostridial myonecrosis and gas gangrene. This sort of infection does not rely on the synergistic action of a polymicrobial infection for its virulence. Mechanism - clostridium can produce large amounts of volatile fatty acids, which generate gas in tissues. An alpha toxin is also produced, which results in platelet damage, hemolysis and capillary damage, destroying tissues and providing further medium for infection. Polymicrobial Infection - some infections are polymicrobial in nature and contain gram positives (Staph and strep), gram negative rods and anaerobes. These infections appear to be synergistic - the mixture of bacteria is needed for such an infection to spread in this way. DIAGNOSIS: Appearance and Local Exploration of Wound - skin discoloration or necrosis, blebs, drainage of thin, watery, grayish, foul smelling fluid; subcutaneous crepitus) Gram Stain =Clostridial Infection= large gram positive rods with few if any wbcs. - few wbcs because of the rate of infection leaves little time for wbcs to accumulate and the infection may have damaged the local blood supply CT Scan - may give insight as to the level of involvement but should only be used to assist in operative planning as a negative CT Scan does not preclude prompt surgical intervention. In a clear cut case, CT scan is not warranted as it will delay operative intervention. ANTIBIOTICS- because the wound will likely contain several organisms the antibiotic regimen should be broad based with action against gram positives, gram negatives and anaerobes. However, because of concern in all such cases of Clostridial infection, high doses of PCN G are recommended. If patient has PCN allergy, imipenim or Flagyl may be used. In addition to PCN G, Flagyl or clinda (for anaerobes) and either a aminoglycoside or monbactam (for gram negative) may be used. Alternatively, Imipenim, timentin and Unasyn can be combined to give broad coverage.

SURGICAL INTERVENTION PREOP PREP. - Fluid resuscitation, arterial line, PA catheter (if septic shock is present), Foley (if not already) DEBRIDEMENT - debride all necrotic tissue. In necrotizing fasciitis, this usually involves the skin, subQ and fascia but spares the muscle. In clostridial myonecrosis, the muscle may vary from being pale and watery to lacking contractility on stimulation to black and necrotic. - in necrotizing fasciitis, resect al involved fascia, sub Q and skin. - in clostridial myonecrosis, if the process is early a fasciotomy may be enough to decompress the fascial compartment and decompress the muscle. This way the muscle can be decompressed and inspected. Decision is made on resection. Muscle viability - contracts when stimulated by bovie, bleeds when incised. Key - resection must be beyond the borders of the infection back to normal viable appearing tissue. Must plan to return the patient to the OR in 24 hrs. for reinspection. HYPERBARIC OXYGEN - physiologic basis is the exposure of the clostridia to absolute 02 tension of 3 atmospheres inhibits production of alpha toxin. Can be used either as an adjunct to surgery or as primary therapy after fasciotomy and exposure of the wound in hopes of limiting infection and subsequent debridement and disability. Controversial, but appropriate surgical therapy with appropriate antibiotics is the preferred therapy for this patient.

# 90 - Burn 47 y.o. male with 60% TBSA combination 2nd and 3rd degree burns over the upper body from smoking in bed, sparing only lower body Issues: a.) ABCs (focus on A with history of burn in closed area) b.) What form of respiratory evaluation and treatment/support would you offer? c.) Pt. hypoxemic, what additional support would you institute? Pt. still hypoxemic, what else can you do? d.) Fluid resuscitation according to Parkland Formula for day 1 and 2. What kind of fluid? e.) How would you treat the wounds? f.) When would you excise? What would you graft first? g.) What would do for nutrition? h.) Can the patient survive with this injury? ANSWER: CONSIDERATIONS FOR SMOKE INHALATION- injury is rarely related to heat as much as it is related to (1) hypoxia (2) damage to tracheobronchial epithelial cells from toxic components in the smoke. Hypoxia - in fire, pt. may be inspiring oxygen fraction as low as 10%. Carbon Monoxide - CO binds preferentially to Hgb. In a short period, the arterial 02 tension is normal because CO has only replaced 02 in rbcs. -> consequence of these factors alone is impaired 02 delivery and tissue oxygenation. Earliest signs of hypoxia are signs of impaired cerebral oxygenation - disorientation and lethargy.

Heat Injury - rare for heat injury to cause extensive damage to the upper or lower respiratory tract. Even if this does occur, the full impact is not seen for 12-18 hrs. after fluid resuscitation when full airway and pulmonary edema sets in. Chemical Injury - usually much more serious than heat injury, results in injury to both the lower and upper airways. Laryngospasm may occur early as a defense mechanism in the conscious patient. Early symptoms consist of early wheezing and bronchospasm from smooth muscle constriction of injured airways. Increased airway pressures after burn will cause increased mucosal edema. Loss of Chest Wall Compliance - most evident in case of a circumferential burn of the upper body/chest wall. Result is diminished intrathoracic volume, significant ventilation/perfusion mismatch, atelectasis, hypoventilation. Anything (hypoxia, sedation, pain, hypovolemia) compromising respiratory effort will accentuate lung dysfunction. This will get worse as the patient is fluid resuscitated and chest wall edema worsens. INITIAL EVALUATION AND TREATMENT OF INHALATION INJURYpatient with deep facial burns, oral burns or upper airway edema from inhalation injury requires early intubation. Expectant management may lead to emergency intubation in situation where there is laryngospasm and edema and may make oral intubation very difficult. Patients with chemical burns (closed space) and impaired gas exchange should be intubated for pulmonary toilet and institution of positive pressure ventilation. Chest wall escharotomy should be considered if appropriate burns exist. (If escharotomy done, should use curvilinear incisions extending cephalad to caudad along the midclavicular to anterior axillary line. Third incision should be made just beneath the costal margin. Can perform with scalpel or cautery without anesthesia. - do not use racemic epi or steroids. See Figure 11-5 and 11-6 in Greenfield for algorithm on airway and ventilation management. - see also 287 - 289 in Gold. MANAGEMENT OF BURN WOUND RULE OF NINES: HEAD =9% EACH ARM = 9% FRONT TORSO = 18% BACK TORSO = 18% EACH LEG = 18%. Antibiotics - not needed initially because the burn wound is avascular and intravenous antibiotics will not penetrate areas needed. May also contribute to the selection of resistant organisms.

TOPICAL AGENTS : SILVADENE - primarily directed against gram negative rods with some anti- fungal activity. Since it doesnt penetrate burn s as well as some other agents, it should be used to prevent infection rather than treated proven infection. COMPLICATIONS: LEUKOPENIA - due to transient bone marrow suppression. the effect is usually transient and the wbc will dip to 2000-3000 before returning to normal even if therapy is continued. SULFAMYLON - broad spectrum antibiotic, but especially effective against gram negative rods. Also some antifungal activity. Because of better penetration into burn eschar, this is better for treating established, proven infection rather than for prevention. COMPLICATIONS: CARBONIC ANHYDRASE INHIBITOR- systemic absorption can lead to metabolic acidosis and pulmonary insufficiency. FLUID RESUSCITATION 1ST 24 HRS. 2ND 24 HRS. BROOKES = LR 2 ML/KG/% BURN D5W TO KEEP UP U.O. (50% first 8 Hrs, 50% 2nd 16 hrs.) PARKLAND = LR 4 ML/KG/% BURN D5W TO KEEP UP U.O. NUTRITION - should be begun on the second to the fifth day after burn. Should be given via enteral route starting at the 4-5 day after the burn, if the GI tract is functional. CALORIC NEED - Curreris Formula - 25 kcal/kg + 40 kcal/each 1% BSA burn. - problem - for enteral formulas, iso-osmotic formulas at 1-2 kcal/ml means that 115cc/hr are needed for a 70kg man with a 60% burn. Hyperosmotic formulas often result in diarrhea. For burns over 50%, the iso-osmotic enteral formula must be supplemented with a parenteral protein formula through a peripheral or central vein. PROTEIN NEED - 2 gr./kg body weight NITROGEN NEED - 3gr./kg body weight. # 91 - Incarcerated Groin Hernia 26 y.o. obese female presents with complaints of nausea and vomiting, pain in RLQ and low grade fever. Issues: a.) What do you look for on history and physical exam (hint) b.) Pt. is too obese to adequately assess groins. What would you do for the remainder of the workup? c.) How would you prepare this patient for the operating room? d.) How would you approach this patient in the operating room? In the OR you find an incarcerated inguinal hernia which you are unable to reduce. What would you do to assist in

the reduction of this hernia? How would you prevent showering of metabolic toxins from this ischemic bowel? e.) The bowel is dusky and black. What would you do? f.) How would you repair the hernia? Would you use mesh? g.) If the patient had a history of CHF, what sort of anesthesia would you use? ANSWER: INCARCERATED HERNIA - incarceration is most likely when a narrow hernia neck is present, such as the femoral canal, obturator canal, internal inguinal ring, umbilical ring and around stoma. Closed Reduction Problem #1- closed reduction may reduce a strangulated segment of bowel. Problem # 2 - if the hernia reduced en masse, the constricting neck may be reduced and continue to cause obstruction. Operative Approach - should focus on exposure and control of the hernia sac, its contents and the constricting neck so that the incarcerated segment can be examined. Approach #1- suprainguinal incision - satisfactory but risks include premature reduction and loss of control of potentially strangulated bowel segment. Approach #2 - preperitoneal approach - provides access and control, allows exposure for preperitoneal repair and ability to resect bowel. Approach #3 - transabdominal approach - provides ability to resect bowel easily but can make hernia repair somewhat difficult. Releasing Incarceration - may require that the hernia defect be enlarged. This may make it more difficult to close and may require mesh for defect closure. Mesh for repair may be risky in patients who requires resection and especially risky in those with peritonitis. May use absorbable vicryl mesh as a temporizing measure when mesh is needed to close a defect and prevent recurrent herniation. HERNIA ANATOMY NERVES: ILIOHYPOGASTRIC NERVE - lies beneath the external oblique aponeurosis and dives into the subcutaneous fat a cm above the external inguinal ring. Lies medial in the inguinal canal above the conjoined tendon. Gives sensory to the suprapubic skin. ILIOINGUINAL NERVE - lies along the anterior surface of the spermatic cord and exits at the external inguinal ring. Gives sensory to the pubic region and the upper scrotum and the labia major.

GENITOFEMORAL NERVE - the genital branch perforates the transversalis fascia and transversus abdominis muscle inferior to the internal inguinal ring. Provides sensory to the scrotum and inner thigh INDIRECT INGUINAL HERNIA - lateral to epigastric vessels - forms because of persistence of processus vaginalis. Hernias found at the superficial inguinal ring are covered by (1) skin and sub Q (2) external oblique aponeurosis (3) cremasteric muscle fibers and (4) attenuated transversalis fascia. Surgical Correction - exposure is attained by incising the external oblique aponeurosis. Incision through the cremasteric fibers and internal spermatic fascia allows the hernia sac to be dissected out and isolated. The sac is opened and the intestinal contents are reduced. The sac is closed at the deep inguinal ring. The ring is closed by approximating the conjoint tendon to the shelving edge of the inguinal (Pouparts) ligament - Bassini repair. DIRECT INGUINAL HERNIA - medial to the epigastric vessels in Hesselbachs triangle. Borders of Hesselbachs Triangle - laterally by the inferior epigastric vessels, medially by the lateral border of the rectus abdominis and inferiorly by the inguinal ligament. This area is composed entirely of transversalis fascia can present at the superficial inguinal ring but does not present in the inguinal canal. Covered by (1) skin and subcutaneous tissue (2) external oblique aponeurosis (3) attenuated transversalis fascia. Surgical Correction - incision is made in the attenuated transversalis fascia and the peritoneal hernia sac is reduced. The edges of the incised tranversalis fascia are held in place using preplaced stay sutures. These stay sutures are tied down after the hernia is reduced. The conjoint tendon is then sutured to the shelving edge of the inguinal ligament (Bassini repair).

S-ar putea să vă placă și